Download as pdf or txt
Download as pdf or txt
You are on page 1of 112

Samenvatting rechtssociologie

2020-2021

geschreven door

StudentjeRechten

De website voor het Kopen en Verkopen van je Samenvattingen

Op Stuvia vind je de beste samenvattingen, notities en ander studiemateriaal. Voor alle toetsen,
examens en cursussen. Bekijk het aanbod op Stuvia.

www.stuvia.com

Gedownload door: ayoubelamri99 | ayoubelamri99@outlook.be Wil jij €76 per


Dit document is auteursrechtelijk beschermd, het verspreiden van dit document is strafbaar. maand verdienen?
Stuvia - Koop en Verkoop de Beste Samenvattingen

Sociologie en rechtssociologie.............................................................................................5
Wat is Rechtssociologie?...............................................................................................................5
Law in books vs laws in action..........................................................................................................................5
Instrumenteel en expressief recht (evenwicht tussen beide)..........................................................................6
Sturingsvraagstukken.......................................................................................................................................6

Tweerichtingsverkeer recht + samenleving....................................................................................6


Sociale productie recht.....................................................................................................................................6
Sociale werking van het recht..........................................................................................................................8

Afstandelijk, betrekken, kritisch, constructief................................................................................9


Juridisering........................................................................................................................................................9
Factoren variatie juridisering............................................................................................................................9
Historische ontwikkeling van de rechtssociologie.........................................................................................10
4 Invloeden:....................................................................................................................................................10
Karakteristieken van de rechtssociologie.......................................................................................................11

Hoorcollege 4: Regelgeving...............................................................................................12
Inleiding.......................................................................................................................................12
Deregulering als Maatschappijhervorming (HUYSE).....................................................................13
Totale Regeldruk.............................................................................................................................................13
Overregulering................................................................................................................................................14
Vormen van Deregulering..............................................................................................................................15
Maatschappelijke probleemstelling gepreciseerd.........................................................................................17
Regulering en Instrumentalisering van het Recht..........................................................................................17
Twee vormen van Juridisch Rationeel Denken..............................................................................................18
Instrumentalisering van Recht.......................................................................................................................19
Grenzen en Risico’s aan Deregulering (Huyse)...............................................................................................19

Wetgevingspraktijken (HULS)......................................................................................................21
Sociale productie van wetten:........................................................................................................................21
Wetgeving als product van nationale politieke besluitvorming....................................................................21
Semi soevereine staat....................................................................................................................................21
Kritiek op de Wetgever...................................................................................................................................22

Welke rol spelen ambtenaren achter de schermen wetgevingsproces........................................23


Inleiding..........................................................................................................................................................23
Modellen van het wetgevingsproces.............................................................................................................24

Wetgeving op de ambtelijke werkvloer.......................................................................................28


Onderhandelend wetgeven.........................................................................................................29
Definitie HOEKEMA.........................................................................................................................................29
Begrippen en hun betekenis...........................................................................................................................30
Hebben partijen altijd vrije keuze om te handelen?......................................................................................30
Is de belangenafweging feitelijk het laatste woord?.....................................................................................30
Conclusies.......................................................................................................................................................31

Handhaving.......................................................................................................................32
Handhaving als Thema van Rechtssociologie...............................................................................32
Noorse wet op huishoudelijk personeel 1949 door Vilhelm Aubert.............................................32
Symboolwetgeving......................................................................................................................33

Gedownload door: ayoubelamri99 | ayoubelamri99@outlook.be Wil jij €76 per


Dit document is auteursrechtelijk beschermd, het verspreiden van dit document is strafbaar. maand verdienen?
Stuvia - Koop en Verkoop de Beste Samenvattingen

(= vooral symbolisch op papier, maar weinig effect in de realiteit → kritiek)...............................33


Sociale functies van wetgeving.......................................................................................................................33
Effecten van Wetten.......................................................................................................................................33

Handhaafbaarheidstoetsen & handleidingen...............................................................................41


Tafel van Elf....................................................................................................................................................41

Publieke Handhaving...................................................................................................................44
Toezicht op handhaving wetten.....................................................................................................................44
Vier stijlen van regeltoepassing......................................................................................................................45
Soort regulering..............................................................................................................................................46
De politie als handhaver.................................................................................................................................46
Vervormde spiegels tussen politie en gemeenschap.....................................................................................46

Private Handhaving (= handhaving door privé-instanties)...........................................................50


Formele Geschillenbeslechting..........................................................................................51
1) Civiele rechtspraak in België....................................................................................................51
Vraag Perspectief............................................................................................................................................51

2. Rechters werk in Theorie en Praktijk........................................................................................54


In het spoor van Durkheim ’s Functionalisme................................................................................................54
Theorie van sociale systemen........................................................................................................................55
Legitimiteit......................................................................................................................................................56

Politicologische analyse van rechtbanken door Martin Shapiro...................................................59


Straffen als sociale praktijk...............................................................................................60
Benadering van het rechtspreken (= straftoemeting)...................................................................60
PROBLEEMSTELLING (BEYENS, 2000)...........................................................................................60
WETTELIJK KADER........................................................................................................................61
Hoe straf gaan bepalen?.................................................................................................................................61

Straftoemetingsonderzoek..........................................................................................................61
Buitenwettelijke factoren...............................................................................................................................62

Strafdoelstellingen.......................................................................................................................62
Persoon van de rechter................................................................................................................63
Context........................................................................................................................................63
Straftoemetingspraktijk...............................................................................................................63
Onderzoeksvragen.......................................................................................................................64
Bevindingen onderzoek...............................................................................................................64
Persoon van de dader.....................................................................................................................................64
Daderbeeld en stereotypering.......................................................................................................................64
Rechter als informatieverwerker...................................................................................................................65
Interpretatie dader en straf (discours)...........................................................................................................65

Gevangenisstraf – alternatieve straf............................................................................................65


Straftoemetingscultuur................................................................................................................65
Structurele context: interne factoren..........................................................................................67

Gedownload door: ayoubelamri99 | ayoubelamri99@outlook.be Wil jij €76 per


Dit document is auteursrechtelijk beschermd, het verspreiden van dit document is strafbaar. maand verdienen?
Stuvia - Koop en Verkoop de Beste Samenvattingen

Hedendaagse penale context.......................................................................................................67


Conclusie empirisch onderzoek (Beyens, 2000)............................................................................67
Algemene conclusie.....................................................................................................................67
Rechter tussen rede en gevoel.......................................................................................................................67
Rechtspreken als betekende partij.................................................................................................................68
Straftoemetingscultuur..................................................................................................................................68
Gelijkheid versus dispariteit...........................................................................................................................68

Straffen is meer dan het bestrijden van criminaliteit...................................................................68


Maatschappelijk aspecten van de penaliteit................................................................................69
Penologie en Penitentiair recht......................................................................................................................69

(Enkele belangrijke vragen om mee te nemen)............................................................................69


Informele Geschillenbeslechting........................................................................................70
ADR (Alternative dispute solution)..............................................................................................70
Model Wouters & van Loon.........................................................................................................70
Geschilbeslechtings-delta (2004, 2014)........................................................................................70
Functioneel perspectief...............................................................................................................70
Functionele Benadering...............................................................................................................71
Achtergrond ADR.........................................................................................................................71
Onvrede met de Rechtsgang..........................................................................................................................71

Grondslagen................................................................................................................................71
Informalisering & access to justice.................................................................................................................71
Ontlasten gerechtelijk systeem......................................................................................................................72
Filosofie van Participatie................................................................................................................................72

Informele Geschillenbeslechting..................................................................................................73
Onderhandeling..............................................................................................................................................73
Bemiddeling....................................................................................................................................................73
Ombudsman...................................................................................................................................................73
Arbitrage.........................................................................................................................................................74
Minnelijke Schikking (Bij Rechtbank).............................................................................................................74

Uitdagingen voor ADR.................................................................................................................74


Proceduralisering............................................................................................................................................74
Institutionalisering..........................................................................................................................................75
Professionalisering.........................................................................................................................................75

ADR en Recht 2 Wijzen van Conflictbeslechting...........................................................................75


Maatschappelijke trends als achtergrond......................................................................................................75

Internationaal Vergelijkend Perspectief van Rechtspraak & ADR (Van Aeken)............................76


Internationaal Vergelijkend Perspectief.........................................................................................................76

Culturele verklaring.....................................................................................................................76
Conclusie.....................................................................................................................................77
Juridische Beroepen...........................................................................................................78
De mensen van het recht.............................................................................................................78

Gedownload door: ayoubelamri99 | ayoubelamri99@outlook.be Wil jij €76 per


Dit document is auteursrechtelijk beschermd, het verspreiden van dit document is strafbaar. maand verdienen?
Stuvia - Koop en Verkoop de Beste Samenvattingen

Juridisch beroep.............................................................................................................................................78
Juridische beroepen in België een algemeen overzicht.................................................................................78
Studenten en afgestudeerden in rechten aan de universiteit.......................................................................79

Recht in het Bedrijfsleven............................................................................................................83


Bedrijfsjuristen...............................................................................................................................................83
Onderzoeksontwerp.......................................................................................................................................84
Onderzoeksontwerp.......................................................................................................................................84

Conclusies....................................................................................................................................89
Bedrijfscultuur & Juridisch Bewustzijn...........................................................................................................89

Legitimiteit van Recht. Burger & Justitie............................................................................90


Wat weten en vinden burgers van recht......................................................................................90
Knowledge & Opinion about Law...................................................................................................................90
Rechtsgevoel en Rechtsbewustzijn................................................................................................................91
Benadering Amerikaans en Europees............................................................................................................92
Rechtsvervreemding.......................................................................................................................................93
Legitimiteit in rechtssociologisch onderzoek.................................................................................................94
Van Beleving naar Naleving van Recht...........................................................................................................95

Publieke Opinie & Recht..............................................................................................................95


Legitimiteit van de Rechter (Huls)..................................................................................................................95
Hoogconjunctuur van vertrouwen.................................................................................................................96
Omslag Jaren 90: Kritiek.................................................................................................................................96
Aanvaarding van de Rechtsnorm (Van Houtte)..............................................................................................96

Justitie onder de Loep: De publieke opinie ten aanzien van justitie.............................................97


Kort overzicht bestaande onderzoeken in BE – kloof recht en burgers.........................................................97
Methodologie van het onderzoek..................................................................................................................98
Nationale resultaten van de eerste juridische barometer...........................................................................100
Conclusies.....................................................................................................................................................102
Het nut van Opinie onderzoek.....................................................................................................................102

Recht in de risicomaatschappij........................................................................................104
Recht in de maatschappij – NICK HULS......................................................................................104
Leven in de risico maatschappij...................................................................................................................104
Recht in de risicomaatschappij.....................................................................................................................105

Vande Walle – conflicthandeling bij collectieve schade: op zoek naar herstel...........................108


opkomst slachtofferschap............................................................................................................................108

Gedownload door: ayoubelamri99 | ayoubelamri99@outlook.be Wil jij €76 per


Dit document is auteursrechtelijk beschermd, het verspreiden van dit document is strafbaar. maand verdienen?
Stuvia - Koop en Verkoop de Beste Samenvattingen

Sociologie en rechtssociologie
Praktische zaken:

 Schriftelijk examen met meerkeuzevragen met een aantal open vragen.

Wat is Rechtssociologie?
Wat is recht?

 Verschillende antwoorden, hangt af aan wie je de vraag stelt


o Juristen: Geheel van regels die neerslag vinden in de bronnen van het recht, deze
regels vinden toepassing in de praktijk van rechtspraak en rechtsleer.

o Filosofen (natuurrecht): recht verwijst naar ideaalbeeld als systeem van waarden en
principes die het sociale leven moeten structureren.

o Sociologen: Recht als sociaal fenomeen.


 Nauw: maatschappelijke controle uitgeoefend door systematische
aanwending van macht.

 Breed: geheel van collectieve gedragingen en normen die het sociaal leven
structureren.

Law in books vs laws in action

 Law in the Books: juridisch perspectief, probleem oplossen binnen een set spelregels.
o Juristen, jurist: juridische puzzel oplossen, theorie recht
 Law in action: recht zoals het bestaat in de praktijk. (veel breder perspectief).
o functie juridische instituties, structureren R, personen R, rechtssociologen

Law in the books Law in action

Wetteksten Totstandkoming
Vonnissen Functioneren juridische instanties
Bevoegdheden Effecten
Argumenten Praktijken
Oplossingen Problemen

Gedownload door: ayoubelamri99 | ayoubelamri99@outlook.be Wil jij €76 per


Dit document is auteursrechtelijk beschermd, het verspreiden van dit document is strafbaar. maand verdienen?
Stuvia - Koop en Verkoop de Beste Samenvattingen

Instrumenteel en expressief recht (evenwicht tussen beide)


 Instrumenteel recht
o Dienende taak, instrument inzetten door wetgever (doel is wet bereiken)
o Bindende regels tot stand brengen, conflict oplossen

 Expressief recht
o Waarden SL
o Rechtsbescherming individu

Sturingsvraagstukken
Samenleving → complexer → nieuwe onoverzichtelijkheid

 Recht belangrijk: niet zomaar goed, vanzelfsprekend


 Alternatieven voorwaarde
 Juristen verantwoorden

Tweerichtingsverkeer recht + samenleving


Rechtssociologie
DEFINITIE: Empirische discipline waarin het recht wordt opgevat als een sociaal verschijnsel, als
product van menselijke interactie. (Empirisch = het veld in gaan, ervaring)
 Mate waarin juridische constructies betekenis krijgen
 Zoeken achterliggende context
Sociale productie (invloed SL recht) ↔ sociale werking (invloed recht SL)
Sociale productie recht
 Sociale productie van het recht: Maatschappij beïnvloed het recht
= Alle spelers (factoren en actoren) voor de totstandkoming van het recht.
o Interne Formele bronnen v/h recht (wetgeving, algemene rechtsbeginselen, ..)
o Maatschappelijke bronnen van het recht

Politiek
1) Politiek (relatie tussen recht en politiek)

Recht is product van bestaande politieke besluitvormingsprocessen in een land.

 Publieke machtsfactoren: politieke partijen, ambtenaren, bestuurders


 Private machtsfactoren: Lobby bedrijfsleven, vakbonden, milieuorganisaties
 Internationale politiek: spelers die een invloed hebben op wat wij kunnen beslissen

→ Sociaal economische verhoudingen vastgelegd in sociaal economische raad (SER)

Gedownload door: ayoubelamri99 | ayoubelamri99@outlook.be Wil jij €76 per


Dit document is auteursrechtelijk beschermd, het verspreiden van dit document is strafbaar. maand verdienen?
Stuvia - Koop en Verkoop de Beste Samenvattingen

Economische – technologische ontwikkelingen


2) Economisch-technologische ontwikkelingen

De ondernemingsgewijze productie en technologische innovatie zijn sterk bepalend voor de


inrichting van het rechtssysteem.
 Kapitalistische productiewijze: contract, eigendom, concurrentie
 Wetenschap en technologie: DNA-technieken, coldcase
 Rampen

Juridische professionals

3) Juridische professionals

 Rechtspraak: kijken wat de rechters doen in de praktijk/ hoe is de RB georganiseerd?


o Abstracte regels betekent na toepassing rechters
o hoe functioneert rechterlijke macht

 Advocatuur: verzakelijking en internationalisering

 Ambtenaren: rol van ambtenaren op het recht


o Nieuwe regering maken
o Onderhandelen met pastor bedrijven (zielenzorg)
o Over toelaatbare kosten krediet

Rechtsopvatting burgers

4) Rechtsopvattingen burgers

Welke kennis hebben burgers over het recht? Wat zijn hun ideeën over hoe het recht functioneert in
de maatschappij? Hebben zij vertrouwen in het recht?

 Iedereen geacht de wet te kennen


o Kennis recht?
o Vertrouwen recht bij conflicten
o Opvattingen functioneren instituties ontstaan

 Actieve invloed uitoefenen: invloed recht op burger


o opeisen van rechten
o verkiezingen
o demonstraties

Gedownload door: ayoubelamri99 | ayoubelamri99@outlook.be Wil jij €76 per


Dit document is auteursrechtelijk beschermd, het verspreiden van dit document is strafbaar. maand verdienen?
Stuvia - Koop en Verkoop de Beste Samenvattingen

Rechtscultuur

 Verschillen in rechtscultuur in verschillende landen


o Landen verschillen in machtsverschillen
o Vaste culturele patronen van een land hun weerslag in rechtscultuur
o Kijken juridische sector (aanbodzijde: hoeveel D bieden ze aan)

Rechtspluralisme

 Komst grote groepen → cultuur diverser


o anders denken over recht, rechtvaardigheid in verschillende groepen van SL

Sociale werking van het recht

 Sociale werking van het recht: Recht beïnvloed de maatschappij

Effecten van het recht in de SL (bedoeld en onbedoeld).

 Bestrijden assumpties: “iedereen kent de wet” / “iedereen gehoorzaamt de wet”

Anti-instrumentalisme

Sociologische analyse gaat de normatieve veronderstellingen van wetgeving na en vergelijkt deze


met sociale werking.

o Mobiliseringstheorieën: “op welke manier beroepen sociale groepen op rechtsregels


om hun belangen te helpen?”

o Handhavingstheorieën: “Wat zijn de typen van handhaving die door ordehandhavers


kunnen worden gehanteerd?”

Wetten: vaak andere gevolgen dan door de wetgever beoogt, gebruik door mensen om andere
redenen.

Hoe bereikt recht de burger?


 Via BS (niemand leest)
 Via intermediaire organisaties → regels vanuit eigen belang doorgeven
o Burger krijgt gekleurde, strategisch, gefilterde info
o VB: werkgever, vakbond, sportorganisatie

 Via filterende instituties met eigen interpretatie van boodschap van de wetgever.
o Media, TV, kranten, Internet
o Intermediaire organisatie: werkgever, vakbond, sportorganisatie, …

Gedownload door: ayoubelamri99 | ayoubelamri99@outlook.be Wil jij €76 per


Dit document is auteursrechtelijk beschermd, het verspreiden van dit document is strafbaar. maand verdienen?
Stuvia - Koop en Verkoop de Beste Samenvattingen

Mensen leven in sociale verbanden


 Mens = individu
 Functioneren in verschillende organisaties
o Gebonden aan (in)formele regels
o hoge mate autonomie
o beperkte mate invloed door OH

Semiautonome sociale velden


Semi-autonome velden

= Een laag dat tussen de wetgever en de burger zit. Bemoeilijken centrale sturing van OH + basis voor
maatschappelijk draagvlak voor handhaving recht.

 Eigen doelen
 Formuleren + handhaven eigen formele en informele normen voor leden.
 Hoge mate van autonomie + beperkte invloed van de OH.
 VB: studentencorps, tuchtrecht en extern recht

Bottum up, top down


o Bottom-up perspectief: Sociale werking vanuit maatschappij
o Top-down perspectief: sociale werking vanuit bestuurder

Afstandelijk, betrekken, kritisch, constructief


Juridisering
Toenemende rol van recht in de maatschappij. Relatie met opkomst en uitbouw van:

 Rechtstaat: acties van de staat gebonden aan het recht.


 Verzorgingsstaat: regulering van het welzijn van de burgers. (VB: mensenrechten)

Factoren variatie juridisering


 Cultuur: andere wijzen om met conflicten om te gaan
o Japan vs VS: Japan heeft schaamte cultuur.
 Economische ontwikkeling
o ontwikkelingslanden VS ontwikkelende landen
 Juridische infrastructuur
o Toegang tot recht, omvang rechtshulp

Gedownload door: ayoubelamri99 | ayoubelamri99@outlook.be Wil jij €76 per


Dit document is auteursrechtelijk beschermd, het verspreiden van dit document is strafbaar. maand verdienen?
Stuvia - Koop en Verkoop de Beste Samenvattingen

Historische ontwikkeling van de rechtssociologie


 Natuurrecht VS rechtspositivisme
o Natuurrecht = het recht zoals dat ons van nature is gegeven

 Uitgerust met de natuurlijke reden


 Principes die voor iedereen gelden
 VB: universele verklaring voor de rechten van de mens

o Rechtspositivisme = recht wordt bepaald door het juridische systeem waar je je op


dat moment in bevindt.

 Positieve recht, nadruk: formele geldigheid van rechtsregels


 Verbreken band tussen recht en moraal

4 Invloeden:

1) Kritische rechtswetenschap
Start Duitsland 19e E, positivisme is heel sterk ontwikkeld.

Jhering: gaat kritiek geven vanuit het Recht.


 Moeten kijken naar de DOELEN van het R.
 Niet zozeer kijken naar het systeem van het recht.
Ehrlich:
 Begin 20e E.
Zwaartepunt van de rechtsontwikkeling ligt niet in wetgeving/ rechtspraak MAAR in de maatschappij.
We moeten naar het levende recht (= alle normen die in die SL bestaan en waar het recht ook deel
gaat van uitmaken en die de maatschappij zullen sturen) gaan kijken om het recht goed op waarde te
kunnen schatten.
Slaat ook over naar de VS en die nemen deze ideeën over.

2) Ideologiekritiek
Karl Marx: Kritiek op de Klassenbasis van het recht (relatie tussen recht en macht).

Recht is een soort afspiegeling van de politieke machtsverhoudingen in de maatschappij, volgens


Marx te maken met de klasse (groep). De klasse die in een bepaalde fase van de geschiedenis aan de
macht is, is ook de groep die voor de maatschappij het recht zal bepalen.

Recht gaat uit van een idee van gelijkheid, Marx zegt dat dit een soort van ideaal is, en dat er een
grote ongelijkheid is in de maatschappij.

10

Gedownload door: ayoubelamri99 | ayoubelamri99@outlook.be Wil jij €76 per


Dit document is auteursrechtelijk beschermd, het verspreiden van dit document is strafbaar. maand verdienen?
Stuvia - Koop en Verkoop de Beste Samenvattingen

3) Algemene sociologie
Durkheim + Weber = grondleggers van de sociologie.

Zien het recht als een heel belangrijk fenomeen in de maatschappij.

 Durkheim ziet het recht als een soort spiegel van hoe de SL georganiseerd is.
 Weber vindt dat het recht vooral zegt hoe een maatschappij zich wil organiseren.
o Op basis van: rationalisering van besluitvorming + bureaucratisering.

4) Naoorlogse sociologie
Sociologie wordt steeds meer verfijnd, er komen betere methodes in het veld. Men gaat zich
specialiseren, men wilt geen uitleg geven over de hele maatschappij maar bijvoorbeeld over een
specifiek thema.

Karakteristieken van de rechtssociologie


1) Effectuering van regels en rechten:
- toets regels aan hun gebruikswaarde/ werkt de regel gezien het doel?
2) Rechtsverhoudingen en machtsverhoudingen
3) Recht en organisatie (link met semi-autonome velden)
4) Recht en sociale verandering
- Law Lag argument: recht zou achterlopen bij de maatschappelijke ontwikkeling. Regels zijn
verouderd.

11

Gedownload door: ayoubelamri99 | ayoubelamri99@outlook.be Wil jij €76 per


Dit document is auteursrechtelijk beschermd, het verspreiden van dit document is strafbaar. maand verdienen?
Stuvia - Koop en Verkoop de Beste Samenvattingen

Hoorcollege 4: Regelgeving
Inleiding
Traditionele rechtssociologische benadering: Law in the Books (+ recht) ↔ Law in action (feitelijke
werking in de SL). Het leidde tot pragmatische methoden om wetgeving beter te doen aansluiten op
de samenleving.

1952: Noorse hoogleraar Vilhelm Aubert aandacht aan effectiviteit wetten. Wet op huishoudelijk
personeel 1948 hij wou de toestand van huispersoneel verbeteren, maar er was ineffectiviteit door
het ontbreken van handhavingsmachines. Instrumenten wetgever houden geen rekening met
intrinsieke complexiteit van samenleving. Rechtssociologen benaderen wetgeving vanuit
verschillende perspectieven.

1) 1980 verzorgingsstaat : overheid einde maken aan legal explosion of wetgevingstsunami


2) Sociale werking regels
3) Symbolische werking van regelgeving en communicatiebenadering

Complexe ontwikkelingen: nieuwe intellectuele en methodologische uitdagingen die vaak in


mekaar integreren.

 Globalisering, nationalisering van de rechtsorde, private normstelling, technologische


ontwikkelingen, ontwikkeling regulatory state, scepsis over representatieve democratische
wetgevingsproces

De wet en het recht:


 Wet als bron van het recht
 Outscope (afspeelt buiten interne logica rechtssysteem)
 Inscope (werking en impact wet)
 In Engels is law zowel ‘wet’ (loi) als ‘recht’ (droit)
o Bij ons verschillende betekenissen, in het Engels maar 1 woord
o Geeft aan hoe belangrijk die rol van de wet voor het recht is

→ DEREGULERING ALS MAATSCHAPPIJHERVORMING : EEN RECHTSOCIOLOGISCH PROBLEEM

12

Gedownload door: ayoubelamri99 | ayoubelamri99@outlook.be Wil jij €76 per


Dit document is auteursrechtelijk beschermd, het verspreiden van dit document is strafbaar. maand verdienen?
Stuvia - Koop en Verkoop de Beste Samenvattingen

Deregulering als Maatschappijhervorming (HUYSE)


Huyse zegt:

 Perceptie dat er in Westerse landen (België) sprake is van:


o Toegenomen Regeldruk
o Overregulering

DAAROM: Pleidooien voor Deregulering

Totale Regeldruk

 Moeilijk kwantitatief op te sporen.


o het regulerend gewicht verschilt van regel tot regel
o Weinig data over private regeldruk (gezin, organisaties, gemeenschap)
 Vaardigen hun eigen regels uit

o Welke eenheden telt men (wetten of ook ‘pseudowetten’?)


 Indien ze meetellen ook verantwoorden

Belangrijk punt: die private ordening (= private governance) wordt (over het algemeen) niet
meegeteld maar kan net zo een onderdrukkend effect hebben als die van de Staat. + steeds lastiger
om ze uit elkaar te halen omdat grenzen steeds meer vervagen (VB: tussen wat publiek is en wat
privaat is)

 Voornamelijk referentie aan recht van de staat


 SLblindheid voor ‘private governance’/ private ordening, die even onderdrukkend kan zijn.
 Vervagende grenzen tussen private en publieke sector
o (veel private INST met publieke financiering of taken, soft law, private rechtspraak)

13

Gedownload door: ayoubelamri99 | ayoubelamri99@outlook.be Wil jij €76 per


Dit document is auteursrechtelijk beschermd, het verspreiden van dit document is strafbaar. maand verdienen?
Stuvia - Koop en Verkoop de Beste Samenvattingen

Overregulering

Stelling dat de toegenomen regeldruk leidt tot excessieve regulering, waar we vanaf zouden
moeten.

1) Klachten
Klachten (niet van Huyse, maar van de mensen die willen dereguleren)

 Externe kosten (kosten voor de SL, het bedrijfsleven)


o Voorwaarden aan productieproces verhoogt lasten
 kosten drukken onevenredig hoog op kleinere ondernemingen (want zij
missen juridische competentie om rechtsregels adequaat op te volgen.) →
onevenwicht van machtsverhoudingen.

o Legislatieve interventie (OH) in economie verstoort marktproces


 Nadelig voor economische dynamiek en groeikansen bedrijf
 Kan om betere marktcoördinatie te willen
o Teveel wetgeving creëert onzekerheid en remt economie

 Interne Kosten (kosten voor de OH zelf)


o bestuurlijk apparaat: als je meer wetten moet maken betekent dat dat je meerdere
keren door die procedure van wetgeving en andere regels heen moet.
 = + procedure K, + ambtenaren, bureaucratie, complexere besluitvorming

o Gerechtelijk apparaat: meer belasting voor de rechtbanken, aangesproken worden


om over meer wetten uitspraken te doen

 hogere taaklast, wachtrijen vr de RB, rechtsonzekerheid (contradicties)


o Parlement: verschuiving van macht (door al die wetgeving)
 Weg van parlement naar uitvoerende macht

Argument: excessieve regulering leidt tot allerlei kosten die voorkomen zouden moeten worden.

We kunnen de klachten in 3 groepen neerzetten:


1) Volume van regelgeving: teveel regels
2) Kwaliteit van regelgeving (gering): inconsistent, slechte formulering
3) Gebruik van regelgeving (magistraten/ ambtenaren): te strikte toepassing volgens de letter/wet

14

Gedownload door: ayoubelamri99 | ayoubelamri99@outlook.be Wil jij €76 per


Dit document is auteursrechtelijk beschermd, het verspreiden van dit document is strafbaar. maand verdienen?
Stuvia - Koop en Verkoop de Beste Samenvattingen

Kritiek

Huyse gaat die klachten proberen te ontleden en problematiseren.


 Kritiek: Dereguleringsliteratuur gaat uit van:
o TE Simplistische kijk op gedrag van rechtssubjecten
 = Hoe mensen omgaan met wetten
o Feitelijke afdwingbaarheid van overheidsregels
 Hoe makkelijk het is om W ook daadwerkelijk te handhaven en af te dwingen.

Huyse zegt dat als je goed kijkt in dat dereguleringsparadigma: Te weinig aandacht vr/e # fenomenen:

 Schijnregulering: politiek afgezwakte symboolwetten die slecht afdwingbaar zijn


o Wetten komen tot stand binnen een politiek proces (in Landen zoals België waar vaak
een compromis gemaakt moet worden tussen verschillende partijen) is er het gevaar
dat sommige wetten in een nogal politiek afgezwakte vorm uitgevaardigd worden en
eigenlijk meer een symbolische functie hebben dan dat ze makkelijk afdwingbaar zijn.

 Wetsontwijking (= legitiem): juridische vluchtroutes binnen de wet


o Binnen het wettelijk kader zijn er een aantal paden waarin je juridisch correct nog
kan wegkomen met bepaalde acties.

 Wetsontduiking: straffeloze overschrijding (niet-recht)


o Overtreding van de wet, er wordt niet genoeg gehandhaafd. Men kan bepaalde
wetten overtreden zonder dat daar direct consequenties aan vast hangen.

Schijnregulering, wetsontwijking en wetsontduiking bekritiseren eigenlijk dat beeld, dat soort meer
simplistische idee achter de dereguleringsliteratuur.
De laatste 2 komen vaker voor in hogere sociale lagen + geven aan:

 Vereisen materiele middelen en vaardigheden die niet gelijkelijk verdeeld zijn.


o VB: mensen die zich laten adviseren door hun advocaten

 Striktere vervolging en bestraffing van geweld en vermogensdelicten, dan regulering van


economisch leven

Vormen van Deregulering


 Overregulering wordt beschreven als een maatschappelijke ‘ziekte’: hyperlexie
o Lex staat voor wet, hyperlex wilt dus zeggen: te veel wetten.
o Deregulering = medicatie

15

Gedownload door: ayoubelamri99 | ayoubelamri99@outlook.be Wil jij €76 per


Dit document is auteursrechtelijk beschermd, het verspreiden van dit document is strafbaar. maand verdienen?
Stuvia - Koop en Verkoop de Beste Samenvattingen

Huyse is hier kritisch op, als we dit stellen dan:


 Deregulering de ‘medicatie’
 Verschillende ‘therapieën’

1) Lichte chirurgie: kwaliteitsverbetering


2)Preventieve geneeskunde: reguleren met maat
3) Amputatie: ontmantelen van rechtsnormen

1.Kwaliteitsverbetering (lichte chirurgie)


 Codificatie en harmonisatie van wetgeving ter verbetering toegankelijkheid,
overzichtelijkheid & effectiviteit

 Regelmatige herijking wetten om discrepantie tussen wet en sociale werkelijkheid te


beperken
o Kijken “dit is wat we bedoelde met de wet oorspronkelijk maar wat voor een effecten
heeft die wet nu in de maatschappij en komen die overeen met wat wij
oorspronkelijk hebben bedoelt?”

o Regelmatig kijken of daar geen te groot gat is ontstaan tussen die wet en de sociale
werkelijkheid die die wet zou moeten reguleren.

HUYSE BRENGT NUANCE AAN:


 Wetstechnische oplossing: Herregulering geen deregulering
o Men gaat bestaande wetgeving in kwaliteit verbeteren.

2. Reguleren met maat (preventieve geneeskunde)


 Terughoudend Wetgevingsbeleid
o goed nadenken voor men überhaupt een wet gaat introduceren
o heel vroeg in het wetgevingsproces

 Nieuwe regels worden vooraf gegaan door kosten-batenanalyse (zowel direct als voor de SL)
en toetsingscriteria (Preregulering)

Ook hier brengt Huyse nuance aan: hij zegt eigenlijk er is hier wat men deregulering noemt maar het
is eigenlijk een soort préregulering, het gaat vooraf aan de regulering. Het is een soort check om te
kijken of we überhaupt een wet moeten uitvaardigen.

 Zowel gericht op kwaliteit als omvang van wetgeving

16

Gedownload door: ayoubelamri99 | ayoubelamri99@outlook.be Wil jij €76 per


Dit document is auteursrechtelijk beschermd, het verspreiden van dit document is strafbaar. maand verdienen?
Stuvia - Koop en Verkoop de Beste Samenvattingen

3. Ontmantelen van rechtsnormen (amputatie)


 Meest radicale vorm
 Gericht om volume van wetgeving drastisch te verlagen (Ontregulering)

Twee varianten:
1) Gericht op de inhoud van het recht:
 op gegroeide reguleringsdomeinen van arbeid, milieu, mensenrechten, welzijn, gezondheid
o Allemaal gebieden waar een OH is gaan ingrijpen
 Welzijnsregulatie staat haaks op belangen bedrijfsleven
o Zijn vaak een externe kost voor het bedrijfsleven
 Niet per se deregulering: afbraak bedrijfsvijandige regelgeving gaat samen met introductie
bedrijfsvriendelijke wetgeving (inhoud, niet volume)

2) Reductie van het recht van de staat ten voordele van andere vormen van ordening en niet-officiële
regelcomplexen met andere machtsverhoudingen (private normen)
 Wijzigt de rol van recht in de samenleving ten gunste van normenpluralisme

Maatschappelijke probleemstelling gepreciseerd

2 transformaties van reguleringsproblematiek:


 verschuiving in de aard van de rechtsorde
 wijzigingen in de maatschappelijke ordening

Regulering en Instrumentalisering van het Recht

Rechtssociologische bijdragen: historische wijziging in de functies van het recht


 Minimalistische visie
o 19e eeuw: recht een passieve registratie van een al aanwezige normatieve orde
(codificatie functie), Oh beperkte/terughoudende taak. = Nachtwakerstaat

 Maximalistische Visie
o 20e eeuw wordt recht een actief middel voor maatschappelijke verandering
(modificatie functie)

→ Recht wordt steeds meer gezien als instrument voor social engineering

17

Gedownload door: ayoubelamri99 | ayoubelamri99@outlook.be Wil jij €76 per


Dit document is auteursrechtelijk beschermd, het verspreiden van dit document is strafbaar. maand verdienen?
Stuvia - Koop en Verkoop de Beste Samenvattingen

Twee vormen van Juridisch Rationeel Denken

Formeel-rationeel
Weber beschrijft dit fenomeen (codificatie)
19e eeuw: Evolutie tot stelsel van universele regels. Daaromheen uitgewerkte rechtsleer en
gemeenschap experts (juristen)

 Empirische ontdekking van de relevante feiten van een casus


 Logische deductie van de toepasbare regels
 Oplossing zonder referentie aan externe waarden (ethisch, politiek, religieus, economisch)
 Recht heeft hoge vorm van autonomie
 Dam tegen overheidsinterventie (de OH moet gebonden worden aan de wet)
 Voorspelbaarheid stimuleert kapitalistische productiemodus.

VB: Recht en Maatschappij

 Recht en de Moderne Bureaucratische staat vormen een symbiotische eenheid.


 In Europa ontwikkelde recht een formele logische rationaliteit
o Algemene normen
o Consistent systeem
o Autonoom en Suprême. Mensen gehoorzamen allereerst aan dit systeem boven
andere vormen van sociale ordening

Dit systeem creëerde voorspelbaarheid, reduceerde economische onzekerheid en droeg bij tot
modern kapitalistische staat.

Materieel-rationeel
 Niet enkel de wijze en de procedure waarop dingen gebeuren maar vooral:
o Aandacht voor maatschappelijke inhoud van regels
o VB: contractenrecht
 Kijken naar sociale effecten van regels, blik komt meer op de SL terecht
 Wetgevend en rechtelijk optreden in functie daarvan.
 Externe overwegingen (ethisch, pol, eco) dringen binnen in rechtsvorming en rechtsvinding
 Deels verlies autonomie recht, primaat politiek
o Wordt als instrument gezien in de handen van beleidsmakers
 Juridisering van sociale relaties

18

Gedownload door: ayoubelamri99 | ayoubelamri99@outlook.be Wil jij €76 per


Dit document is auteursrechtelijk beschermd, het verspreiden van dit document is strafbaar. maand verdienen?
Stuvia - Koop en Verkoop de Beste Samenvattingen

Instrumentalisering van Recht

 Na WO II wordt het denken over recht materieel-rationeel


 Recht gezien als instrument om individuen en samenleving te sturen
 Van interne naar externe besluitvorming:
o geen logische toepassing van regels, maar politieke doelen
 Relatie met veranderende maatschappelijke orde en grenzen van de marktwerking
 OH treedt actief corrigerend op in de markt en grijpt in bij welvaartverdeling
(verzorgingsstaat)
 De verspreiding van overheidsrecht past binnen deze historische evolutie van de rechtsorde

Gevolgen:
 Nu geldt primaat van de politiek
 Juridisering van sociale relaties
 Moderne beleidswetgeving gekenmerkt door BVHoverdracht aan de exclusieve

Kritiek op Instrumentalisme

(visie dat recht een instrument is om bepaalde maatschappelijke doelen te bereiken door de OH.)

 Overschatting van instrumentele capaciteit van rechtsregels: relatieve ineffectiviteit van


rechtsregels
 Veronderstelling dat rechtsregels op automatische wijze hun doel van gedragsbeheersing
bereiken (spontane effectiviteit)
 Verwaarlozing aanvaardingsprobleem: regels zijn alleen effectief als deze aansluiten bij
sociale consensus

→ Dat causaal-deterministisch model is theoretisch en empirisch zwak. Recht van staat heeft als
techniek gedragsbeheersing dus eigenlijk grenzen en die realiteit kan men niet straffeloos negeren.

Grenzen en Risico’s aan Deregulering (Huyse)

Huyse gaat kritiek terugbrengen naar de deregulerignsgedachte. Hij grijpt terug naar de medische
metafoor, dus het idee dat er therapieën zijn.

 Hoe zwaarder de medicatie hoe groter de kans op neveneffecten


 Bekijk per hervormingsplan de voorwaarden, gevolgen en contra-indicaties

19

Gedownload door: ayoubelamri99 | ayoubelamri99@outlook.be Wil jij €76 per


Dit document is auteursrechtelijk beschermd, het verspreiden van dit document is strafbaar. maand verdienen?
Stuvia - Koop en Verkoop de Beste Samenvattingen

Deregulering als kwaliteitsverbetering, problemen

1. Kwaliteitsverbetering (kwaliteit van wetten verbeteren)


 Onderschatting juridisch-technische moeilijkheid, vaardigheden zijn niet altijd aanwezig.
 Weinig legistieke vorming ambtenaren
 Politieke compromissen compliceren opwaardering

2. Herijking wetgeving (strookt de wet wel nog met de sociale werkelijkheid)


 Registreren van sociale werkelijkheid is moeilijk en afhankelijk van consensus
 Doel: kloof tussen wet en levende werkelijkheid dichten

3. Regelen met maat


 Ontoereikende informatie voor kosten-baten weging
 Hoe weet men dat van te voren? Inschatting is lastig.

4. Selectieve Ontmanteling
 Ontmantelen bedrijfsongunstige rechtsnormen alleen na politieke confrontatie en moeizame
compromisvorming

Instrumentalisering van Recht (SLIDES NIET BEHANDELD)

Verschillende dereguleringstherapieën

 Kwaliteitsverbetering & reguleren met maat zijn instrumentalistisch


 Ontmantelen van rechtsnormen is fundamenteler: terugval naar formeel-rationale logica.

→ Minder overheid (nachtwakerstaat), meer private ordening

Grenzen en Risico’s aan Deregulering

Concluderend zegt Huyse: (deregulering sterk bekritiseerd)

Zwakke plekken
 Teveel ontworpen vanuit bedrijfseconomisch of juridisch-technisch perspectief. Daarmee in
politiek vacuüm tussen allerlei maatschappelijke belangengroepen

 Simplistische kijk op werking overheidsapparaat. Overschatting bereidheid om deregulering


uit te voeren. (Zo is twijfelachtig of ambtenaren wel in staat zijn om te dereguleren).

20

Gedownload door: ayoubelamri99 | ayoubelamri99@outlook.be Wil jij €76 per


Dit document is auteursrechtelijk beschermd, het verspreiden van dit document is strafbaar. maand verdienen?
Stuvia - Koop en Verkoop de Beste Samenvattingen

Wetgevingspraktijken (HULS)

1) Sociale werking van wetten


Wat is de invloed van recht op de maatschappij? (= belangrijke wisselwerking!)
2) Sociale Productie van Wetten
Hoe komen wetten tot stand + wat is de rol van de maatschappij in die totstandkoming?
Huls heeft het voornamelijk hierover, bezig met dit punt!

Sociale productie van wetten:

 praktijk van wetgeving: dynamische proces: waar politiek en recht elkaar direct beïnvloeden.
 Wetgeving op snijvlak politiek, recht en samenleving
 Juristen willen politiek meestal vermijden
 Politieke besluitvorming gaat aan wetgeving vooraf en is sociologisch essentieel
 Kritiek op Regulering
 Rollen bij totstandkoming van wetten
o Ambtenaren: niet alleen uitvoeren wat politici zeggen. 4 e macht met eigen inbreng
 Naast trias politica, zijn ambtenaren de 4e macht die inbreng hebben.
o Belangengroepen
 Effecten van Wetgeving

Wetgeving als product van nationale politieke besluitvorming


 Meerpartijensysteem: 1 partij haalt nooit de meerderheid in verkiezingen. (↔ VS/Spanje)
o Heeft consequenties voor de wijze waarop wetten tot stand komen
o Na de verkiezingen heeft nooit 1 partij de meerderheid → coalities.
 Complexe coalitiekabinetten
 Politieke meerderheid is bevoegd wetten te maken die bindend zijn voor de hele bevolking
(representatieve vertegenwoordiging).
 Politieke partijen willen allen hun maatschappelijke visie realiseren door wetten.
 Compromissen (kan effect hebben op de kwaliteit van wetgeving)

Semi soevereine staat


 Wetgever = het hoogste woord
 Is België nog wel soeverein? Veel recht is afkomstig van de Europese Unie
o Democratisch tekort: geen direct gekozen regering en beperkte macht parlement
o Rechtsorde buiten staten om: nieuwe dynamiek, maar ook legitimering

 Daarnaast lid van de Raad van Europa en EHRM (Europees Hof Rechten van de Mens).
Controle van nationale wetgeving aan mensenrechten

21

Gedownload door: ayoubelamri99 | ayoubelamri99@outlook.be Wil jij €76 per


Dit document is auteursrechtelijk beschermd, het verspreiden van dit document is strafbaar. maand verdienen?
Stuvia - Koop en Verkoop de Beste Samenvattingen

Kritiek op de Wetgever
Huls laat ons zien dat er verschillende vormen van kritiek zijn gekomen op de wetgever.

1. De wetgever ter discussie in de politiek


2. De wetgever ter discussie in het recht
3. De wetgever ter discussie in andere disciplines
4. De wetgever ter discussie door zichzelf

1) De wetgever ter discussie in de politiek (uitgebreid behandeld bij Huyse) ZIE


DEREGULERING
 Na WOII ontwikkelt zich een welvaartstaat
 Wetgeving wordt gezien als een instrument in de handen van de overheid.
 Overheid neemt veel taken ter handen
 In de Jaren 80 wordt politiek een dereguleringsoffensief ingezet
 Verlichting regeldruk op bedrijven en burgers
 Zoektocht naar alternatieven voor wetgeving

2) De wetgever ter discussie in het recht


 Klacht over gebrekkige juridische kwaliteit van wetgeving
o Gevolg: open vage normen als resultaat van consensuspolitiek
o Gevolg: Niet waarmaken sociale grondrechten
 Probleem: traagheid van de wetgever
 Handhavingsproblemen (wetsontduiking & wetsovertreding)
 Europees recht
 Ambtenaren domineren proces VS primaat politiek

3) De wetgever ter discussie in andere disciplines


Bestuurskunde: Herbezinning op verticale sturingsconcept waar OH centrale stuurinstantie is van
maatschappij (cockpit) (wat is de rol van de OH precies? Rol van de OH als soort van centrale
bestuursinstantie)
Economie: marktverstorende effecten van regelgeving. Regelgeving als efficiënte verdeling van lusten
en lasten in verschillende sectoren. Alternatieven voor OH interventies (emissierechten markt VS
milieuregels)

22

Gedownload door: ayoubelamri99 | ayoubelamri99@outlook.be Wil jij €76 per


Dit document is auteursrechtelijk beschermd, het verspreiden van dit document is strafbaar. maand verdienen?
Stuvia - Koop en Verkoop de Beste Samenvattingen

4) De wetgever ter discussie door zichzelf


 Inzicht dat wetgevingsproblematiek veelzijdiger is dan regelbestand
 Kritiek dat kwantiteit kwaliteit begint te beïnvloeden. Recht ingezet als instrument – door
overheid en burgers - voor gevestigde belangen
 Zorgvuldiger omgang met het recht: behoudt zicht op waarden in de samenleving
 Maatschappelijke functie: wetten die zinvol moeten zijn.

Niet pure deregulering of marktwerking, maar nieuwe figuren van wetgeving. Wetgeving schakelt
sociale dynamiek bij oplossing van problemen uit, omdat mensen wachten op de wettelijke regeling,
i.p.v. mensen te mobiliseren mee te werken met oplossingen vinden.

 Wet niet als instrument om problemen voor burgers op te lossen, maar als instrument voor
burgers om zelf hun problemen op te lossen met hulp overheid.
 3 rollen overheid: wetgever markt, toezichthouder, dienstverlener
 Nieuwe uitdagingen wetgeving
o Rol Europa
o Technologische ontwikkelingen
o Benodigde deskundigheid om deze problemen aan te pakken

Welke rol spelen ambtenaren achter de schermen


wetgevingsproces
Inleiding
Totstandkoming wetgeving is proces deels openbaar en deels achter gesloten deuren:

1) Openbare discussies kan je volgens in parlement en media.


2) Het besloten gedeelte door insiders in vergaderkamers van bestuurlijke/ambtelijke top van
ministeries, politieke partijen uit 2e kamer en achterkamer met lobbyisten en belangengroepen.

Wet meest bestendige vorm van overheidsbeleid vanwege legaliteitsbeginsel. Correct tot stand
komen , bevolking in representatieve democratie en verplicht zich aan de uitkomst te onderwerpen.

Interesse in wetgeving: staatsrechtgeleerden (in staatskundige vormgeving wetgeving ) ↔


praktijkjuristen (voor hen wetstekst en parlementaire stukken van belang als ze juridische adviezen
geven).

23

Gedownload door: ayoubelamri99 | ayoubelamri99@outlook.be Wil jij €76 per


Dit document is auteursrechtelijk beschermd, het verspreiden van dit document is strafbaar. maand verdienen?
Stuvia - Koop en Verkoop de Beste Samenvattingen

Theorieën over Wetgevingspraktijken

HULS gaat een analyse geven van de wetgevingspraktijken.

Sociale productie: Hoe komen wetten tot stand?


Concepten en inzichten om wetgevingspraktijken te analyseren.

 Synoptisch beleid fase


 Agendabouw
 Bureaupolitiek
 4 rationaliteiten

Modellen van het wetgevingsproces


Het synoptisch beleidsfasemodel (= bestuurdersperspectief)

1e Manier: Synoptisch beleidsfase (vergelijking met de wetgeving / juridische model)


Synopsis = term uit de film wereld
 Bestuurdersperspectief (top-down(begint van uit het bestuur)): beschrijving van buitenkant
van de productie van wetten, zoals model hiervoor
 Zoals wetgeving behoort te geschieden, goed geregisseerd, rationeel proces, waarin ieder
een duidelijke rol heeft
 Synopsis film, scenario’s, draaiboek
o Minister is regisseur
o Hoofdrollen: parlementsleden regeringsfracties
o Bijrol: leden oppositie

 Leiding politieke ambtsdragers bij formuleren wet ↔ Belangengroepen (zoals burgers)


deskundigen in de marge.
o focus op politici, Focus op primaat politiek
 Wetgeving = bindende besluitvorming dat richting wil geven aan SL
 Democratie: inspraak burgers + organisaties + adviezen deskundigen.
o Eindoordeel: aan politiek
 Leidende thema: primaat politiek (democratisch verkozenen hebben leiding) ↔ de andere
(invloed maar uitkomst accepteren als bindend)

 Juristen hebben een eigen rol (raad van state, wetgevingsafdeling ministerie)
o Inbreng verschillend in elke wet
o Nu: rekening houdend met EVRM + regelgeving EU

24

Gedownload door: ayoubelamri99 | ayoubelamri99@outlook.be Wil jij €76 per


Dit document is auteursrechtelijk beschermd, het verspreiden van dit document is strafbaar. maand verdienen?
Stuvia - Koop en Verkoop de Beste Samenvattingen

Agendabouwtheorie
Agendabouw (2e manier om te kijken naar wetgeving)
(hoe kunne we dit het best op de politieke agenda krijgen?)
 Model vanuit de samenleving (bottom-up)(we gaan hieruit van het levende recht i.p.v. alleen
het formele recht vanuit de instituties)

 Ontdekken wetmatigheid in hoe een probleem zich ontwikkeld en politieke erkenning krijgt
 Niet alle problemen worden door de politiek opgelost,
 Specifieke voorwaarden voor transformatie maatschappelijk tot politiek probleem
 Wetten zijn uitkomst van politiek proces, waarbij verschillende groepen met uiteenlopende
visies en belangen strijden

 Zowel binnen als buiten overheid

De weg van private problem tot wettelijk public issue

 Fases (verschillende fases op het pad van publieke herkenning)

 1e fase: waarop zo’n probleem belangrijk wordt: Diffuse onvrede over bepaald
maatschappelijk verschijnsel. Individuen proberen aandacht te vestigen.

o VB: Greta Thunberg (ging voor het parlement in Zweden staan, om te staken voor het
klimaat, ze deed dit in het begin helemaal alleen, schoolstaking)

 2e fase: Individuen proberen organisatie of beweging op te bouwen rond de kwestie.


Vereniging belanghebbenden en zoeken publiciteit. Verbreding & collectivering

o Pers werd erbij gehaald, krijgt landelijke aandacht op de tv, kranten, ..

 3e fase: Eisen stellen aan overheid. Wending tot politieke partijen en bestuurders

o We moeten probleem gaan formuleren, hierdoor komen we bij politieke instituties.

!!! Deze 3 fases komen nog voor ons hele stroomschema, voor ons hele synoptisch beleidsfase model
daarna komen we pas in de 4e stap als we succesvol zijn in de eerste 3 stappen. !!!

 4e fase: Opgenomen in partijprogramma, onderwerp van kabinetsformatie. Discussie over


definitie en wijzen van oplossing. Indienen wetsontwerp

!!! Stap 4 komt ook nog voor ons hele stroomschema en synoptisch beleidsfase model, pas dan stel
dat dat allemaal gedaan is dan kan er een wetsvoorstel worden ingediend om bijvoorbeeld
klimaatverandering tegen te gaan. !!!

25

Gedownload door: ayoubelamri99 | ayoubelamri99@outlook.be Wil jij €76 per


Dit document is auteursrechtelijk beschermd, het verspreiden van dit document is strafbaar. maand verdienen?
Stuvia - Koop en Verkoop de Beste Samenvattingen

We zien hier het radicale tegenbeeld van het synoptisch beleidsfase model (= top-down, hier
beginnen we bottom-up vanuit de maatschappij) Daardoor zien we heel andere aspecten van
wetgeving, het gaat over problemen in de SL zelf en de stappen hoe zo’n probleem getransformeerd
wordt naar een politiek probleem en naar iets dat uiteindelijk met een wet kan worden opgelost.

 Wetgever is niet 1 centrale juridische figuur die de wet maakt


 Uitkomst van complex transformatieproces waarin verschillende actoren en factoren invloed
hebben
 Veel problemen bereiken nooit alle fases.
 Overgang van de ene fase naar de volgende is niet vanzelfsprekend, Er moet aan bepaalde
condities worden voldaan (om al die stappen te doorlopen)

Laat zien dat wetgeving een veel complexer proces is, dat het niet alleen gaat om die ene centrale
figuur maar dat er veel meer maatschappelijke groepen komen bij kijken.

!Dit is de enige van de 4 die we gaan bespreken dat echt uitgaat vanuit de maatschappij!

De volgende 2 gaan ook uit van de instituties maar laten daarbinnen een heel
ander perspectief zien.

Bureaupolitiek model van Rosenthal

3e manier: Bureaupolitiek

= wetgeving als uitkomst van de strijd tussen verschillende werkonderdelen (bureaus) van de OH.
(ambtenaren en departementen)

 Organisatie OH als vertrekpunt


o OH in departementen met specifiek deelbeleid
o Departementen in bureaus met specifieke OHtaak: eigen versie algemeen belang
 Door complexiteit behoort elk maatschappelijk probleem tot competentie verschillende
departementen.
 Strijd met coalities, compromissen.

 Inhoud wet is niet eindproduct van ontworpen gewilde beleidsprocessen (VS synoptisch
beleidsfasenmodel(schetst ideaalbeeld van hoe wetgeving tot stand zou moeten komen), …
 … maar van krachtverhoudingen in het interdepartementale krachtenveld en bureaupolitieke
strijd
 Niet transparant of gereflecteerd in de stukken, die ademen sfeer van afgewogen constructief
overleg (purificatie)
 Niet consensus (Durkheim) maar strijd als motor van beleid
 Minder idealistisch, meer realistisch beeld op wetgeving

26

Gedownload door: ayoubelamri99 | ayoubelamri99@outlook.be Wil jij €76 per


Dit document is auteursrechtelijk beschermd, het verspreiden van dit document is strafbaar. maand verdienen?
Stuvia - Koop en Verkoop de Beste Samenvattingen

 Achter sluier van eenheid van regeringsbeleid, een strijd om de invulling van het algemeen
belang
 Nadeel: verkokerde, naar binnen gekeerde overheid, niet bezig met maatschappelijk
problemen

De 3 modellen hebben verschillende hoofdrolspelers:


 1e model: politiekers
 2e model: mensen in de maatschappij
 3e model: ambtenaren en departementen
 4e model: vanuit de instituties, maar er komen heel veel verschillende componenten bij
kijken.

De 4 rationaliteiten van Snellen

4emanier/model: Rationaliteitenmodel
 OHbeleid is opgebouwd uit 4 denksystemen die elk hun eigen logica en autonomie hebben
o Politiek
o Recht
o Economie
o Wetenschap
 Moeten in overheidsbeleid aan elkaar gekoppeld worden (we moeten door de 4 heen)
 Maakt inzichtelijk waarom wetgeving zo complex is.
 Moeilijk argumenten uit verschillende rationaliteiten tegen elkaar af te wegen

1) Rationaliteitenmodel: Politiek Systeem


 Wet is mogelijkheid voor een minister zich te profileren.
 Gepasseerde wetten als indicator voor succes
 Haast: 4 jarige verkiezingscyclus domineert tempo
 Voorwaarde: moet passen in programma partij en op Europees niveau

2) Rationaliteitenmodel: Juridische Systeem (recht)


 Nieuwe wetten moeten passen in het bestaande rechtssysteem (consistent? Geen strijd?)
 Geen strijd met EVRM, Grondwet, andere wetten, of aanwijzingen wetgevingstechniek
 Juridische kwaliteitstoets: de begripsvorming moet aansluiten bij bestaande juridische kaders
 Juridische discipline stelt eigen eisen op: gelijkheid, rechtszekerheid, geen terugwerkende
kracht, openbaarheid, behoorlijke regelgeving, etc.
 Wetgevingsdirecties moeten politieke beleidsmakers wijzen op de juridische rationaliteit

3) Rationaliteitenmodel: Economisch Systeem


 Dictaten van de overheidsbegroting
 Financiering van nieuw beleid gaat vaak ten koste van bestaand beleid
 Deregulering en bewustzijn van de kosten van wetgeving (van Huyse komt hier naar binnen)
 Burden for business omdat wetten kosten meebrengen

27

Gedownload door: ayoubelamri99 | ayoubelamri99@outlook.be Wil jij €76 per


Dit document is auteursrechtelijk beschermd, het verspreiden van dit document is strafbaar. maand verdienen?
Stuvia - Koop en Verkoop de Beste Samenvattingen

4) Rationaliteitenmodel: Technologische Rationaliteit


 Nieuw onderzoek leidt tot nieuwe uitvindingen, en tot technologische toepassingen
 Bron van innovatie en vernieuwing vraagt vaak om nieuwe wetgeving
o Nieuwe DNA techniek
o ICT

Wetgeving op de ambtelijke werkvloer


 Van het politieke niveau naar de werkvloer waar wetten worden gemaakt
 Centrale rol voor de figuur van de ambtenaar
o Beleidsambtenaren
o Wetgevingsjuristen
 Wetgeving als vak en ambtenaren als professionals die op hun eigen wijze uitvoering geven
aan wetgevingsbeleid

 Weinig onderzoek (vergeleken met rechtsvinding en advocatuur)

Deze 2 worden onderscheiden door Weber:

 Politiek als roeping


o Democratisch gekozen politici die voor de politiek gekozen hebben

 Politiek als beroep


o Deskundigen die politiek als baan uitoefenen

Politiek als ROEPING Politiek als BEROEP

Democratisch gekozen politici Deskundig ambtenaren


Maken beleid en keuzes (synoptisch beleidsfase
Voorbereiding en uitvoering
model)
Openbare uitoefening Besloten op burelen
Vast dienstverband en sterke rechtspositie
Om 4 jaar verkozen
tegen grillen politiek

28

Gedownload door: ayoubelamri99 | ayoubelamri99@outlook.be Wil jij €76 per


Dit document is auteursrechtelijk beschermd, het verspreiden van dit document is strafbaar. maand verdienen?
Stuvia - Koop en Verkoop de Beste Samenvattingen

Onderhandelend wetgeven

 Westerse culturen in overgang van bevels- naar onderhandelingshuishouding (De Swaan)


 Hiërarchische processen van onder & boven geschiktheid vervangen door onderhandeling op
basis gelijkwaardigheid en autonomie.
 Ook in politiek: OH van top-down model van eenzijdige machtsuitoefening, naar
participerend en interactief bestuur
 Ontstaan informele rechtstaat als tweede overheidscircuit waarin ambtenaren makkelijker
met de wet omspringen (Hoekema). Besturen als aangaan van contacten met
maatschappelijke organisaties via horizontale netwerken

 Consequenties voor de rol van de wet in de samenleving en de positie van ambtenaren


 Niet langer ‘zwoegers in het vooronder van het schip van de staat’
 Drukken hun stempel op beleidsvorming bij onderhandelingsprocessen met
belanghebbenden
 Zichtbaarder maken van wijze waarop wetten tot stand komen
o Huls zegt: het is belangrijk vanuit een sociologisch perspectief omdat veel
zichtbaarder te maken, nu weten we heel weinig over de rol van ambtenaren bij
wetgeving en het is belangrijk omdat juist heel goed te gaan begrijpen.

 Meer dan alleen aanhoren en formeel consulteren


 Komen tot een wederzijds voordelig resultaat, gezien de doelstellingen
 Besef van wederzijdse afhankelijkheid bij oplossing problemen

Definitie HOEKEMA
‘Overheidsbeleid dat wordt bepaald in een proces van overleg en onderhandeling binnen een min of
meer duurzaam netwerk van relaties tussen de overheid en groepen en particuliere organisaties
waarvan de activiteiten het belangencomplex in kwestie raken’ (Hoekema)

Uitvoering beleid wordt bepaald in proces van onderhandeling en overleg binnen een duurzaam
netwerk van relaties tussen openbare bestuursorganen en particuliere organisaties/groepen. Partijen
streven ernaar om specifieke doelen op bepaald terrein van OHzorg te bereiken via communicatie
hun relevante handelingen op elkaar af te stemmen en zich te binden een bereikte akkoord.

29

Gedownload door: ayoubelamri99 | ayoubelamri99@outlook.be Wil jij €76 per


Dit document is auteursrechtelijk beschermd, het verspreiden van dit document is strafbaar. maand verdienen?
Stuvia - Koop en Verkoop de Beste Samenvattingen

Begrippen en hun betekenis


Overheidsbeleid = Onderhandelend bestuur om algemeen beleid van de overheid.

Overleg en onderhandeling = Overleg= hebben gemeenschappelijk doel, wil, overtuiging dat een
bepaalde oplossing ten goede is. Onderhandeling bereiken van eigen
doelstellingen via eigen presentaties.

Duurzaam netwerk van relaties = Nood aan mate stabiliteit van verhouding tussen partners en
specifieke sociale structuur

Onderhandelingen tussen openbare bestuursorganen en particuliere groepen =

Overkoepelende maatschappelijke organisaties die belangen van de vertegenwoordigers adequaat


bespiegelen.

Activiteiten die belangencomplex raken = Nood aan wederzijdse afhankelijkheid tssn staat & SL

Specifieke doelen op terrein overheidszorg = Gaat om gezamenlijke verklaringen

Communicatie en zelfbinding = Op basis van vrije wil met elkaar onderhandelende en


overleggende partners.

Hebben partijen altijd vrije keuze om te handelen?

Neen, organen overheid en samenleving te afhankelijk van elkaar geworden. Overheid kan op aantal
beleidsterreinen niet meer eenzijdig optreden. Maatschappelijke organisaties ook nood aan
medewerking overheid om hun doelen te realiseren. Dus voor probleemoplossing afhankelijk. Zero-
sum-game waar verlies van de ene partij wordt gezien als winst andere kan worden omgezet in
winst-winst als je onderhandelt. Er is een cognitieve fase nodig waar uitwisseling info plaatsvindt en
partijen meer begrip tonen voor elkaars standpunt en consensus zoeken.

Is de belangenafweging feitelijk het laatste woord?

Bereikte resultaat is een package deal gebaseerd op win-win. Als gevolg van onderhandelingen wordt
op de duur een gemeenschappelijke taal gesproken en ontstaat wederzijds vertrouwen en
rechtszekerheid. Deze leiden tot een sterke politieke bestuurlijke binding en de maatschappelijke
binding is zeer sterk. Als de deal overeind blijft, wordt belangenafweging definitief volrokken in
onderhandelingsplatform.

30

Gedownload door: ayoubelamri99 | ayoubelamri99@outlook.be Wil jij €76 per


Dit document is auteursrechtelijk beschermd, het verspreiden van dit document is strafbaar. maand verdienen?
Stuvia - Koop en Verkoop de Beste Samenvattingen

Conclusies
Conclusies die Huls daarover geeft:
 Actieve en deskundige ambtenaren zijn onmisbaar voor de oplossing van maatschappelijke
problemen

 Er kan meer draagvlak verkregen worden voor wetgeving dan bij eenzijdig opgelegde
wetgeving (wetten worden veel beter nageleefd als ze worden gevormd door consensus)

 Democratische legitimiteit is niet sterk: ruimte voor politieke volksvertegenwoordigers wordt


kleiner (onderhandelingen vinden vaak plaats achter gesloten deuren, niet altijd duidelijk wie
verantwoordelijk is voor wat)

 Ontbreken van zwakke niet-georganiseerde gemeenschappen aan de onderhandelingstafel,


gevaar voor regelgeving door insiders

 Beslotenheid van de onderhandelingen via informeel bestuur, geen heldere


verantwoordingspatronen.

 Mate van binding van politici aan het resultaat. Horizontale en verticale betrouwbaarheid
naar eigen overheidsinstanties en onderhandelingspartners

Concluderende slide over wetgeving


 Wet niet alleen een instrument voor goed bestuur van rationele magistraat, …
 … maar een kunstwerk van allerlei zichtbare en onzichtbare montage-artiesten, ….
 … en van informele onderhandeling en bottom-up initiatieven

31

Gedownload door: ayoubelamri99 | ayoubelamri99@outlook.be Wil jij €76 per


Dit document is auteursrechtelijk beschermd, het verspreiden van dit document is strafbaar. maand verdienen?
Stuvia - Koop en Verkoop de Beste Samenvattingen

Handhaving
Vandaag: gaan we het meer hebben over de sociale werking van wetten.

Inhoudstafel: Handhaving

1) Handhaving als Thema van Rechtssociologie


2) Handhaafbaarheidstoetsen van Regelgeving
3) Publieke Handhaving
i. Handhaving door Ambtenaren
ii. Politie als Handhaver
iii. Bijzondere & Lokale Handhaving
4) Private Handhaving

Handhaving als Thema van Rechtssociologie


(1) discrepantie tussen wet en praktijk
(2) onderzoek naar effecten van wetten

Noorse wet op huishoudelijk personeel 1949 door Vilhelm


Aubert
Vilhelm Aubert: Noorse rechtssocioloog.
Heel bekend door volgend artikel: “Some Social Functions of Legislation”
= Wat sociale functies van wetgeving.

Hij gaat als 1 van de eerste daadwerkelijk kijken naar de effectiviteit van de wetten.

Wat zijn de effecten van de wetten? Langs de ene kant heb je de ideeën van de wetgever en
daarnaast is er de praktijk over hoe die wetten daadwerkelijk werken in de praktijk (hier is een groot
gat tussen).

 Waar doet hij precies onderzoek naar? Noorse Wet op het Huishoudelijk Personeel (1949)
 Ingevoerd ter bescherming van de positie van dienstbodes
o Hiervoor nauwelijks rechtsbescherming

 Voorheen contractvrijheid (amper regulering)


 Wet legt een aantal regels op omtrent arbeidstijden en loon. (Kan je vergelijken met Marx)
 Effectiviteitsonderzoek studies in 1952 en 1956
 Merkt op dat de positie van dienstbodes niet was verbeterd
o De wet voorzag niet in afdoende handhavingsmiddelen
o Geen externe controlerende instantie die toezag op de handhaving
 Wet alleen in werking na protest, protest gebeurde nauwelijks.
o Heel veel aanbod in die tijd

32

Gedownload door: ayoubelamri99 | ayoubelamri99@outlook.be Wil jij €76 per


Dit document is auteursrechtelijk beschermd, het verspreiden van dit document is strafbaar. maand verdienen?
Stuvia - Koop en Verkoop de Beste Samenvattingen

o Wet was heel juridisch technisch, amper te begrijpen voor dienstbodes


o Bode bazin patronale verhouding, weinig protest

Symboolwetgeving
(= vooral symbolisch op papier, maar weinig effect in de
realiteit → kritiek)
 Verschil tussen uitvaardigen en handhaven van wetten
o Wetgeving zonder handhaving = symboolwetgeving.
 Kritiek van symboolwetgeving: uitgevaardigde regels worden niet adequaat nageleefd.
 Niet goed uitgewerkt en weinig middelen voor handhaving + te weinig controle

Sociale functies van wetgeving


Artikel van Aubert heet: Sociale Functies van Wetgeving
 Communicatie van normen aan het algemene (leken-) publiek en aan groep juristen die
conflicten moet beslechten (terwijl litigatie (= het voeren van een proces) niet werd
verwacht in deze sector)

 Effectieve handhaving van, en conformiteit met uitgevaardigde normen in de maatschappij

 Bereiken van een succesvol compromis tussen verschillende politieke belangen (gevaar
symboolwetgeving)
o Wet tot stand in politiek proces

Handhaafbaarheid als aandachtspunt bij de totstandkoming van wetten

 Hoe kan de wetgever symboolwetgeving vermijden?


 Aandacht voor handhaafbaarheid
 Sociaalwetenschappelijke Inzichten
 Handleidingen en Checklists
o Vb Tafel van Elf
 11 criteria van handhaafbaarheid waar de wetgever over na moet denken bij
het maken van wetten.

Effecten van Wetten


Tafel van 11 als uitgangspunt
 Studie van de effecten van de wetten in de SL als ze eenmaal in werking zijn getreden
 Wet moet geen dode letter zijn.
 Burgers en bedrijven moeten er beroep op kunnen en willen doen
o Belangrijke factor waarom een wet minder effectief kan zijn
 Controleambtenaren belast met naleving van de wet

33

Gedownload door: ayoubelamri99 | ayoubelamri99@outlook.be Wil jij €76 per


Dit document is auteursrechtelijk beschermd, het verspreiden van dit document is strafbaar. maand verdienen?
Stuvia - Koop en Verkoop de Beste Samenvattingen

 Deze naleving is niet altijd strikt zoals de verwachting, deze kan variëren.
 Verschil tussen toezicht in books and in action

Sociaalwetenschappelijke Inzichten
Factoren van invloed op naleven van regels
Twee complementaire onderzoekstradities
 Top down handhaving: perspectief handhavingsinstanties (politie, ..)
o Jurisprudentie
o Wetsevaluatie
 Bottom up naleving: perspectief doelgroepen
o Sociale werking

Handhaving werkt beter als burgers en doelgroepen actief worden betrokken zowel in het proces van
wetgeving als in het proces van handhaving. Kans op Naleving wordt groter.

Jurisprudentie
 Zienswijze juristen: de effecten van wetten kunnen worden afgelezen uit de gepubliceerde
jurisprudentie.
 Advocaten verkennen de grenzen van nieuwe wetten en adviseren cliënten
 Zegt wat over juridische feiten niet over de sociale effecten
o een perspectief op handhaving, maar niet al omvattend.

Wetsevaluatie
1) Regulerings Impact Analyse (RIA): ex ante
 Integraal afwegingskader voor beleid en regelgeving & Tafel van 11

2) Regulerings Impact Evaluatie (RIE): ex post (Huyse had het hier over)

Effectiviteitsonderzoek
 Evaluatie van de maatschappelijke effecten van een wet door de wetgever (Aubert)
1) Informele check door ambtenaren bij insiders / belanghebbenden
2) Wetenschappelijk onderzoek door onafhankelijke deskundigen

 Interviews met relevante personen en organisaties, steekproeven dossiers rechtbanken


→ beleidsaanbevelingen
 Nadelen
o Beperkt tot perspectief doelstellingen van de wetgever
o Spanning wetensch onafhankelijkheid en objectiviteit en politieke wenselijkheid.

34

Gedownload door: ayoubelamri99 | ayoubelamri99@outlook.be Wil jij €76 per


Dit document is auteursrechtelijk beschermd, het verspreiden van dit document is strafbaar. maand verdienen?
Stuvia - Koop en Verkoop de Beste Samenvattingen

Top down handhavingsinstanties


Vaststellingen: zelden volgens letter (1), handhaving niet autonoom (2), verschillende stijlen (3)
 Zelden volgens de letter van de wet
o in de handhaving van de wet was verwachting ‘je hebt wet en het is aan handhaver
om ervoor te zorgen dat ze wordt nageleefd’  men ziet echter onderscheid tussen
geest en letter v/d wet (= twee types v handhavingsstijlen)

 geest van de wet: geeft meer keuzevrijheid voor handhavers om handhaving


in te richten zoals geacht in de context 

 geeft meer inzicht op eigenheid v handhavingsinstituties


 handhaver is instantie die contact heeft met doelgroepen  hebben
veel ervaring met materie en doelgroepen en bouwen expertise op
waarbij ze vrijheid hebben om wet toe te passen

 letter van de wet (= strikte handhaving): kan onbegrip oproepen en bijgevolg


legitimiteit van wet aantasten
o Volgen eigen agenda institutie in geest van de wet
 Toezichthouder heeft unieke kijk op de regulering van bepaalde materie
door ervaring en staat dichter bij doelgroepen dan regelgever

 handhaving niet autonoom


o moderne inzichten in de sociologie: zowel regelgeving als handhaving zijn processen
waarbij burger betrokken moet worden  geen proces boven de hoofden van
burgers

o zelfs bij strafrecht is burger onmisbare medewerker

 VB kliklijn (= burgers kunnen aangifte doen van dingen die ze hebben gezien)
 VB Boston bombing (Marx)
o handhaving is niet meer zoals vroeger volledig autonoom

“Citizens willingness to invoke the law …. To call upon it for aid in securing their private interests –
thus often seems essential to effective enforcement of law by state agencies.” (Cotterrell 1984)

 Gewijzigde Handhavingsstijl
o = verschil in stijlen v hoe gehandhaafd kan worden  v sanctie naar preventie
 VB: idee van wortel en stok
 vroeger werd gehandhaafd adhv ‘de stok’
 Echter ook adhv wortel handhaven  vooruitzicht v beloningen
 dreiging dat men stok zal gebruiken kan ook genoeg zijn om naleving
van de wet te bewerkstelligen

35

Gedownload door: ayoubelamri99 | ayoubelamri99@outlook.be Wil jij €76 per


Dit document is auteursrechtelijk beschermd, het verspreiden van dit document is strafbaar. maand verdienen?
Stuvia - Koop en Verkoop de Beste Samenvattingen

Empirisch Onderzoek: Drie dilemma’s voor de handhavers


Handhaver kiest handhavingsstijl afhankelijk van tijd, plaats, organisatie en medewerker:

 afschrikken VS verleiden: (VB van wortel en stok)

o een visie die uitgaat v repressie (gericht op stok)  hangt samen met strafrecht +
idee dat effectief en snel moeten worden ingegrepen

o visie die belonen vooropstelt (meer gericht op wortel)  niet zozeer repressie maar
eerder voorkomen dat wet niet wordt nageleefd

 principieel VS pragmatisch
o Dura lex sed lex (= de wet is hard, maar het is de wet)  wet moet principieel
toegepast worden op gestandaardiseerde manier

o toepassing variëren afhankelijk van de context  kosten-batenanalyse: afweging


maken tussen kosten + voordelen en nadelen nagaan

 overheid VS Nevenheid (doelgroepen)


o overheid: idee v staat die boven de burger staat en harde sturing wil  v bovenaf
regels opleggen Boven, hard, sturen, wantrouwen (OH)

o Nevenheid: staat moet naast burgers staan, overleg wordt gepleegd en is gebaseerd
op onderling vertrouwen

Bottum up Doelgroepen
analyse doelgroepen als centraal element
 onderscheid op basis van thema’s die doelgroepen belangrijk vinden
Indeling Doelgroepen, subgroepen
o heel veel ≠ doelgroepen, nl. 7
 geëngageerden, zorgzamen, behoudende, genieters, luxezoekers, zakelijke
en ruimdenkers

o = Normadressaten/rechtssubjecten (degene tot wie de normen zich richten)


 elke doelgroep past bij ≠ karakteristieken
 bij elk v die doelgroep hoort andere handhavingsstrategie  hebben elk ≠
motieven om wet na te leven

 bv. zakelijk: straf/boete motiveert naleving >< ruimdenkers: eigen


normen en waarden motiveren naleving

 als handhavingsstrategie aansluit bij motieven v doelgroep zal wet


beter werken in de praktijk

 elke doelgroep vereist een andere effectieve naleving met aangepaste


instrumenten

36

Gedownload door: ayoubelamri99 | ayoubelamri99@outlook.be Wil jij €76 per


Dit document is auteursrechtelijk beschermd, het verspreiden van dit document is strafbaar. maand verdienen?
Stuvia - Koop en Verkoop de Beste Samenvattingen

rechtsinstrumentalisme & semi-autonome sociale velden


rechtsinstrumentalisme: idee van social engineering

 door geschiedenis heen is er ontwikkeling v welvaartsstaat waar wet wordt


gezien als instrument om actief sociale verandering te bewerkstelligen 
wet is belangrijk instrument

 problemen met dit perspectief: veronderstelling dat regel gelijk is aan het
gereguleerde gedrag – wet gelijk aan naleving ervan  recht geïsoleerd vd
samenleving en voorgesteld als autonoom systeem

 sociale structuren zijn manipuleerbaar door gerichte menselijke. controle

o voorbeeld: uitwerking v idee v Roscoe Pound door Adam Podgorecki


 probeert binnen rechtspositivisme uitwerking te geven aan idee v hoe recht
beste gebruikt worden als sociaal instrument om sociale veranderingen te
bewerkstelligen

 rationeel beleid:
 wetenschap bestudeert oorzaak-gevolg relaties van wetgeving-
maatschappij
 zij kan via rationele beleidstechnieken de SL sturen door middel van
recht

Kritiek op rechtsinstrumentalisme (Sally Moore): zegt dat juridische normen worden onderscheidt
van
andere normen doordat ze gehandhaafd kunnen worden door middel van geweldsmonopolie van de
staat. Maar niet enkel juridische normen doeltreffendheid, ook normen nageleefd die nooit door
overheidsgeweldmonopolie worden afgedwongen.

Legal policy = wetenschap van Social Engineering: geen empirische studie maar programmatorische,
teleologische kijk op recht.

Legal realism & socio-legal studies (van binnen naar buiten)


 Recht is vertrekpunt en van daar wordt gekeken naar de productie en sociale werking.
o Voordeel: recht kan worden veranderd vanuit gewijzigde maatschappelijke
omstandigheden (social engineering)

o Nadeel: Veel juristen hebben een beperkt beeld van de sociale wetenschappen, als
hulpwetenschap. De studies zijn vaak pragmatisch, beleidsgericht, zonder veel
theoretische diepgaand of afstandelijkheid

37

Gedownload door: ayoubelamri99 | ayoubelamri99@outlook.be Wil jij €76 per


Dit document is auteursrechtelijk beschermd, het verspreiden van dit document is strafbaar. maand verdienen?
Stuvia - Koop en Verkoop de Beste Samenvattingen

Effecten van Wetten


 empirisch onderzoek binnen rechtssociologie toont aan dat wet zelden samenvalt met haar
uitwerking in de maatschappij en dat doelstellingen van de wetgever echter zelden (volledig)
gerealiseerd worden

o social engineering/instrumentalistische visie te simplistisch  in samenleving


bestaat meer complexiteit  hoe wet uitwerking krijgt in SL hangt af v ≠ factoren die
men vooreerst moet begrijpen
o samenleving is niet zo kneedbaar en mensen zijn geen atomistische rationele
individuen die top-down kunnen worden aangestuurd door unieke homogene
wetgever

Semi-Autonome Sociale Velden (SASV) (007!)

= Goed afgebakende velden die geobserveerd kunnen worden in termen van hun semiautonomie. Ze
brengen interne regels, symbolen en gewoonten voort, maar tegelijkertijd kwetsbar voor die regels
en beslissingen.

Kenmerken:
 bezit regelgevende capaciteiten (autonoom)
 bezit eigen middelen om naleving regels af te dwingen (autonoom)
 SAV in competitie met andere SAV en overheidsregelgeving (semi autonoom)
 groep mensen die georganiseerd kunnen zijn, maar hoeft niet (veld)

idee van SAVS is begonnen als kritiek op rechtsinstrumentalisme (Pound)


 gaat ervan uit dat wetgever moet beseffen dat in SL hij niet de enige is die normen aan het
stellen is

 wetgever maakt op formele wijze wetten maar ook andere instanties maken normen (=
SAVS)
 mensen leven in allerlei sociale verbanden  bv: gezin, familie, vereniging, sociale
netwerken
o elk van de sociale verbanden hebben eigen (vaak informele) normen en waarden 
mensen richten hun gedrag op normen die gelden in die verbanden

 begrip semi-autonome sociale velden


o Autonoom: de velden genereren zelf normen om gedrag te reguleren, en bezitten
eigen middelen en sancties om die regels af te dwingen (bv afkeuring, uitsluiting)

o Semi- : dergelijke velden bestaan niet op zich maar bv in een familie  sociale
verbanden hebben met elkaar te maken + sociale verbanden hebben te maken met
normen die wetgever uitvaardigt

 die interactie tussen normen van de wetgever en normen binnen de SAVS is


belangrijk in de rechtssociologie  SAVS kunnen werken als filter voor het
kennis nemen v wet

38

Gedownload door: ayoubelamri99 | ayoubelamri99@outlook.be Wil jij €76 per


Dit document is auteursrechtelijk beschermd, het verspreiden van dit document is strafbaar. maand verdienen?
Stuvia - Koop en Verkoop de Beste Samenvattingen

→ SAV geneert normen en voorziet de naleving ervan. Op spontane wijze maar ook volgen want
anders sociale afkeuring of uitsluiting. Rechtsnormen gehandhaafd door SAV zelf.

SASV & wetgeving

1) De boodschap van de wetgever bereikt het individu niet ongestoord


 wordt omgevormd en gefilterd door het SASV waar het individu deel van uit maakt
 Burger leest BS niet zelf, maar wordt geïnformeerd door SASV die gekleurd beeld
heeft van de wet

2) Verklaart strategisch gedrag van individuen en organisaties


 Primair belang burgers is deel uit te blijven maken van SASV
o Is ons directe leven

 Wetgever moet dus rekening houden met het bestaan en de werking van SASV’s. SASV’s beperken
haar sturingsmogelijkheden en kunnen wetgeving dan wel tegenwerken of ondersteunen.

2 MODELLEN:

CONFLICT model: Veld VS Staat CONSENSUS model: Veld & Staat

 Botsing van regulering met gewoonten,  Informele sanctionering kan essentiële


informele normen, maatschappelijke ondersteunende bijdrage leveren aan
belangen, … naleving van statelijke regelgeving

 Het veld biedt weerstand tegen de  Deze regels moeten dan


regulering door de Staat  kans op overeenstemmen met de waarden,
effectiviteit vd wet veel kleiner normen en verwachtingen van de
doelgroepen

 SASV als voorwaarde voor effectieve


regelgeving

Voorbeeld 1. Havendiefstallen (v rechtssocioloog Hofstede):


 onderzoek omtrent aan wie goederen toekomt indien ze van de boot vallen
 in havenwerk heerst saamhorigheid en solidariteit tussen havenwerkers  hebben daardoor
eigen systeem v normen
o normen gaan ook over ‘wanneer is het aanvaardbaar om producten die vd boot
gevallen zijn zelf te houden of terug te geven aan eigenaar’?

39

Gedownload door: ayoubelamri99 | ayoubelamri99@outlook.be Wil jij €76 per


Dit document is auteursrechtelijk beschermd, het verspreiden van dit document is strafbaar. maand verdienen?
Stuvia - Koop en Verkoop de Beste Samenvattingen

o als men zich aan de normen houdt kunnen ze bepaalde goederen voor zichzelf
houden maar wanneer ze die normen overtreden gaan ook binnen de groep v leiders
mensen dit melden aan de politie

Voorbeeld 2. Rookverbod V.S. (Kagan & Skolnick)

 observeren verandering in maatschappelijke opinie omtrent roken


o steun voor het rookverbod horeca
 gevolg: weinig handhavingscapaciteit nodig
 overtreders aangepakt door klanten, personeel
o regelgevende overheid als surfer op juiste maatschappelijke golf  je moet wachten
op wetgeving op de juiste maatschappelijke golf (wetgeving op het juiste moment
uitvaardigen)

 indien te vroeg: vloed golf van weerstand


 indien te laat: Weinig relevante tractie

Sociale Bereidheid tot Naleving


 effectiviteit van de wet hangt meer af van draagvlak bij bevolking dan haar intrinsieke
kwaliteit
 belangrijk inzicht (v Tyler): mensen gaan wet volgen omdat ze geloven in legitimiteit en
procedurele rechtvaardigheid van de wet, niet omdat ze bang zijn voor een straf
 sociale bereidheid om wet na te leven is cruciaal voor haar handhaving, anders is krachtige
handhaving nodig

 een wet verkrijgt pas effectiviteit als het door de SL aanvaard wordt/als legitiem wordt
gezien
 in sociologisch onderzoek kan legitimiteit onderzocht worden  men kijkt dan bv. naar:
o onder welke voorwaarden burgers wetten accepteren,
o hoe zij vertegenwoordigd worden in het regelgevingsproces
o Hoe waarborgen tegen overheidsgezag gegarandeerd worden

Naleving door Bedrijven

 ≠ doelgroep (dan burgers) en die ook ≠ motieven hebben om wet na te leven


 grote bedrijven opereren in publieke belangstelling  gevoelig voor regels en regeldruk
1) reden: schending v wetten zorgt voor een slechte reputatie van het bedrijf
 drie factoren voor naleving door bedrijven:
1) Cognitief: kennisniveau van de doelgroep (wat is de kennis die bestaat bij bedrijven
wetgeving?  mate waarin afwijkende regels gelden, taal en wijze van
bekendmaking

40

Gedownload door: ayoubelamri99 | ayoubelamri99@outlook.be Wil jij €76 per


Dit document is auteursrechtelijk beschermd, het verspreiden van dit document is strafbaar. maand verdienen?
Stuvia - Koop en Verkoop de Beste Samenvattingen

2) Economisch: kosten om aan regels te voldoen moeten passen in economische


mogelijkheden bedrijf  elke wet levert bepaalde kosten op voor bepaald bedrijf

 sommigen zijn makkelijker te incorporeren, voor anderen is dit moeilijker


en zal er meer weerstand zijn om de wet na te leven

3) Normatief: Draagvlak voor de normen + legitimiteit


TUSSEN-CONCLUSIE

Consultatie met doelgroepen (SASV, bedrijven, burgers) van beleid kan leiden tot betere wetgeving
Kan ofwel in het wetgevingsproces ofwel met andere burgers zelf omdat deze overheid kunnen
helpen met handhaven van de wet. Leidt ertoe dat wetten effectiever worden in de maatschappij 
effectieve wetten zijn dan ook legitiemer en hebben meer draagvlak .

Handhaafbaarheidstoetsen & handleidingen


 idee dat in maken van wet je tevoren gaat nadenken over handhaafbaarheid v/e wet  daar
zijn heel wat procedures voor
o bv. tafel v elf
 = elf criteria, elf vragen die de wetgever zich moet stellen over die
handhaafbaarheid

 totstandbrengen van wetten bestaat uit ≠ fasen


o Probleemanalyse
o Instrumentkeuze
 Rechtmatigheid, uitvoerbaarheid en doelmatigheid
 bepalen uitvoerbaarheid en toezicht (geschatte K, verwachte effecten?)
o Effecten wet

Tafel van Elf


 achtergrond: groeiend onbehagen over ineffectiviteit van regelgeving
o één van de redenen dat wetten ineffectief zijn: veel meer energie gaat naar het
bereiken van politieke compromissen dan naar onderzoeken van de effecten v
wetten

Oplossing: 11 criteria, 11 vragen die de wetgever zich moet stellen over handhaafbaarheid)

 vragen gesteld vanuit perspectief van doelgroepen


 (NIET vanuit handhavingsinstantie)
 Doel: betere W tot stand brengen, handhavingsinstanties helpen
Wat is de tafel van elf? Opsomming van factoren die van belang zijn voor de naleving van regels.

41

Gedownload door: ayoubelamri99 | ayoubelamri99@outlook.be Wil jij €76 per


Dit document is auteursrechtelijk beschermd, het verspreiden van dit document is strafbaar. maand verdienen?
Stuvia - Koop en Verkoop de Beste Samenvattingen

Sterke en zwakke kanten van de handhaving en naleving ervan kunnen met behulp van de tafel van
elf in kaart worden gebracht. Het is opgebouwd uit elf dimensies die met elkaar bepalend zijn voor
de mate van naleving van wetgeving. De elf dimensies zijn geformuleerd met het oog op een zo groot
mogelijke bruikbaarheid voor de praktijk van de beleidsontwikkeling en de rechtshandhaving.

Waarom de ‘Tafel van elf’ gebruiken? De OH wil verandering in de SL teweegbrengen door


beïnvloeding van het gedrag van burgers en bedrijven. Een van de beleidsinstrumenten die de OH
hiertoe ten dienste staat is regelgeving. Regelgeving veronderstelt ook een zekere mate van naleving
ervan door de doelgroep. Eveneens is regelgeving bedoeld om te worden nageleefd.
Dimensies van Naleving
Elf criteria kan men opdelen in 3 categorieën:

1) spontane naleving:
o Hoe zou de doelgroep zich gedragen zonder overheidsinterventie?
2) handhaving
a) controle
o Welke invloed gaat uit van de controlerende activiteiten van de overheid?
b) sanctie
o Welke invloed heeft sanctiedreiging op het nalevingsgedrag?

1) Spontane Naleving (perspectief van de doelgroep)


 kennis van regels = bekendheid en duidelijkheid van wet- en regelgeving bij de doelgroep

 kosten & baten = (im)materiële voor- en nadelen die uit overtreden of naleven van de
regel(s)volgen, uitgedrukt in tijd, geld en moeite

 mate van acceptatie = mate waarin het beleid en de regelgeving acceptabel worden
gevonden door de doelgroep

 normgetrouwheid doelgroep = mate van bereidheid van de doelgroep om zich te


conformeren aan het gezag van de overheid

 informele controle = de door de doelgroep ingeschatte kans op positieve of negatieve


sanctionering van hun gedrag door anderen dan de overheid

2) Handhaving (waar overheidsinstanties rol krijgen)


 Meldingskans = de door de doelgroep ingeschatte kans dat een overtreding die is
geconstateerd door anderen dan de overheid wordt gemeld aan overheidsinstanties

 Controlekans = de door de doelgroep ingeschatte kans dat men door de overheid


gecontroleerd wordt op het begaan van een overtreding

o VB: discussie omtrent corona  controleert politie wel daadwerkelijk?

 Detectiekans = de door de doelgroep ingeschatte kans op constatering van de overtreding


indien door de overheid gecontroleerd wordt

42

Gedownload door: ayoubelamri99 | ayoubelamri99@outlook.be Wil jij €76 per


Dit document is auteursrechtelijk beschermd, het verspreiden van dit document is strafbaar. maand verdienen?
Stuvia - Koop en Verkoop de Beste Samenvattingen

o VB: als gecontroleerd wordt zal doelgroep gedecteerd worden

 Selectiviteit = de (verhoogde) gepercipieerde kans op controle en detectie in het geval van


een overtreding door selectie van te controleren bedrijven, personen, handelingen of
gebieden.)

 Sanctiekans = de door de doelgroep ingeschatte kans op een sanctie indien na controle een
overtreding is geconstateerd

 Sanctie-ernst = de hoogte en soort van de aan de overtreding gekoppelde sanctie en


bijkomende nadelen van sanctieoplegging

Kritiek op de Tafel
1) geen onderscheid tussen normadressanten
o 1 checklist voor alle doelgroepen
 Altijd aan zelfde lijst refereren, maar lijst wordt onnodig lang
 Voor bepaalde groepen zijn sommige vragen niet relevant

o differentiatie doelgroepen:
 in lijst geen differentiatie tussen particulieren en bedrijven

2) te weinig aandacht voor legitimiteit


o niks over wijze van communicatie en vertrouwen in overheid in T11
o Onvoldoende aandacht legitimiteit in T11 (slechts 10%)

3) geen weging tussen dimensies en factoren


o T11 voorziet niet in wetgeving want capaciteit is beperkt.
o spontane naleving weegt zwaarder dan handhaving
 Legitimiteit als dominante factor
o Belang sanctiedimensie: naleving afhankelijk van mate zekerheid of zwaarte sanctie
o Effecten controleactiviteiten: soort controle bepaalt naleving of niet
 Als pakkans stijgt: naleving ook

Stakeholder Consultatie
 om draagvlak te creëren is consultatie belangrijk  wetgever moet doelgroepen (voor
meten en bevorderen v draagvlaak) en handhavers betrekken bij opmaak v wetgeving

Communicatiewijzen
Handhaver preventief te werk d.m.v. voorlichting en communicatie.
Vormen:
1) Normatief
 ingezet om bestaande sociale en persoonlijke normen van normadressanten te bevestigen en
nalevingsgedrag hiermee in verband te brengen

43

Gedownload door: ayoubelamri99 | ayoubelamri99@outlook.be Wil jij €76 per


Dit document is auteursrechtelijk beschermd, het verspreiden van dit document is strafbaar. maand verdienen?
Stuvia - Koop en Verkoop de Beste Samenvattingen

 Doel = internalisering regels → betere naleving

2) Dreigend
 vergroten perceptie van de pakkans waardoor naleving positief wordt beïnvloed

3) educatief, voorlichtend

CONCLUSIE:
(1)differentiatie in doelgroepen:
(2)aandacht legitimiteit door consultatie en communicatie
(3)betrek handhaver bij opmaak regels
(4)communicatie-analyse
(5)perspectief van normadressant en toezichthouder

Publieke Handhaving
= wet is tot stand gebracht en is een instrument geworden in handen v handhavers  hoe gaan
handhavers daar nu mee om en welke rol speelt wet in hun dagdagelijkse praktijk?

Handhaving door ambtenaren (Niet behandeld in de les)

Toezicht op handhaving wetten


Controle instanties houden toezicht op naleving wetten. 2 manieren:

TOP DOWN BOTTOM UP

1) controleambtenaren zijn primaire actoren


van beleid. Vaak enige contactpersoon tussen
OH en burger.
1) centraal: de vraag naar effectiviteit en
uitvoerbaarheid van het beleid
2) geen mechanisme toepassing maar direct
contact. Street level-bureaucrats: dagelijkse
2) controle ambtenaren zijn einde van
vertegenwoordigers OH op straatniveau, brug
beleidsteksten
formeel beleid en weerbarstige praktijk.
3) passen mechanisme regels toe
3) Niet alleen controle en sancties, maar ook
advies, informatie, onderhandelingen en
registratie geven.

44

Gedownload door: ayoubelamri99 | ayoubelamri99@outlook.be Wil jij €76 per


Dit document is auteursrechtelijk beschermd, het verspreiden van dit document is strafbaar. maand verdienen?
Stuvia - Koop en Verkoop de Beste Samenvattingen

Vier stijlen van regeltoepassing


(1) RECHTERLIJK
 Rechter kijkt naar regel, omstandigheden, bedoeling wetgever en rechtvaardige oplossing
 Model voor ambtelijke toepassing

(2) MECHANISTISCH
 Regels zijn regels, toegepast zonder onderscheid des persoons
o VB: Webers bureaucraat

45

Gedownload door: ayoubelamri99 | ayoubelamri99@outlook.be Wil jij €76 per


Dit document is auteursrechtelijk beschermd, het verspreiden van dit document is strafbaar. maand verdienen?
Stuvia - Koop en Verkoop de Beste Samenvattingen

(3) WILLEKEURIG
 Controleambtenaar maakt zelf uit of regels worden toegepast via eigen normen

(4) NON TOEPASSING


 Passieve controle, ambtenaar gelooft niet in controle

Verklaring voor verschillende stijlen


 Soort regulering
 kenmerken uitvoeringsveld
 politieke omgeving
 kenmerken controleorganisatie

Soort regulering
Verschillen tussen reguleringen wat betreft:

1. stringentere regels
2. bevoegdheden van ambtenaren
3. beroepsmodelijkheden burgers
4. specificiteit en details regels

Verklaart verschil tussen de oorspronkelijke bedoelingen en verwachtingen van de wetgever tijdens


de opstelling van de wet. De effecten als controleambtenaar in contact komt met burgers en
bedrijven

De politie als handhaver

De politie als handhaver


 politie gezien als handhavingsapparaat bij uitstek
o strafrecht als ultimum remedium
 politie: niet langer verbalisant van strafbaar gesteld gedrag van het strafbedelingsapparaat
 heeft eigen discretionaire ruimte om vorm te geven aan hoe ze wil handheven.
(keuzeopties)

Vervormde spiegels tussen politie en gemeenschap


onderzoek naar wederzijdse perceptie van burgers en politie en de wijze waarop deze tot
standkomt, in relatie tot Community Oriented Policing (COP)

o onderzoekers willen kijken naar hoe de politie kijkt naar de gemeenschap en hoe de
gemeenschap kijkt naar de politie + hoe komt die perceptie tot stand

o participerend onderzoek, interviews bij 5 Belgische politiekorpsen

 Community Oriented Policing (COP)


o = idee dat politie binnen gemeenschap moet staan, banden moet hebben met
mensen uit gemeenschap en daarmee moet samenwerken  gebaseerd op
vertrouwen

46

Gedownload door: ayoubelamri99 | ayoubelamri99@outlook.be Wil jij €76 per


Dit document is auteursrechtelijk beschermd, het verspreiden van dit document is strafbaar. maand verdienen?
Stuvia - Koop en Verkoop de Beste Samenvattingen

o vraag in het onderzoek: werkt dit ook daadwerkelijk?


 doordat politie in interactie staat met maatschappij, zal deze ervaring op doen en (gekleurde)
beeldvorming krijgen over gemeenschap + mensen in de gemeenschap die in interactie
staan met politie krijgen (gekleurde) beeldvorming van politie = vervormde spiegels

o vervormd want nooit helemaal neutraal, worden gekleurd door bepaalde percepties

Policing van groepen, niet buurten


 uit onderzoek ziet men dat politie op regelmatige basis in contact met delen van de
gemeenschap staat, vooral in “problematische buurten”  2 belangrijke fenomenen
o Overpolicing van ‘vaste klanten’
o Underpolicing van andere groepen

 in dezelfde gebieden is van beide sprake  2 zijden zelfde medaille


 geen geografisch kenmerk maar groepskenmerk binnen heterogene demografische buurten
 geen problematische buurten, maar problematische groepen
 de buurtgemeenschap bestaat niet, multiple buurtgemeenschappen

A. Overpolicing
 = idee dat veel politie aandacht uitgaat naar kleine groep individuen
o kleine groep = vaste klanten = daders + slachtoffers
 politie moet twee rollen spelen
o repressieve rol  daders
o zorg verlenende rol  slachtoffers
 groep met weinig sociale opvangmogelijkheden en weinig zelfredzaamheid
o kunnen enkel beroep doen op politie
o economische zwak, lage scholing, matige verbaliteit en taalbeheersing, geen stabiele
woonplaats, gerechtelijk verleden, soms etnisch.

47

Gedownload door: ayoubelamri99 | ayoubelamri99@outlook.be Wil jij €76 per


Dit document is auteursrechtelijk beschermd, het verspreiden van dit document is strafbaar. maand verdienen?
Stuvia - Koop en Verkoop de Beste Samenvattingen

Dader
 politie gebruikt eigen beoordelingsschema’s om categorieën v ≠ groepen te maken
o roepnamen (vb. Schelen, groseilles, snobs, eencelligen) voor ≠ groepen
o namen functioneel voor complexe maatschappelijke verschijnselen binnen
politiecultuur en laten interne communicatie toe

o politie heeft nl. gedetailleerd fijnmazig beeld van deze groepen en ook meer kennis
dan gangbaar in samenleving door dagelijkse confrontatie

 ≠ wijzen om, om te gaan met deze groepen


o deze praktijken weinig bekend bij leidinggevenden
o Pseudo-gedoogbeleid: toenemend gevoel van niet begrepen/gehoord te w bij politie
o leidt tot cynisme en onmachtsgevoel → Kloof tussen politie en GEMSCH wordt groter
 COP wilt dit tegengaan

B. Underpolicing
Onbekende groepen zonder problemen
 politie komt onvoldoende in aanraking met minder problematische en zelfredzame groepen
o veel positieve dynamiek in wijken ontgaat agenten daardoor
o perceptie die agenten hebben v deze wijken komt door kleine groep die ze
overpolicen

 verheffen uitzondering tot regel, waardoor kloof agent – gemeenschap groeit

Onbekende groepen met problemen


 groepen die nauwelijks beroep doen op politie en eigen oplossingsmechanismen gebruiken
 ≠ groepen
o mobiele groepen; wonen voor korte rijd in bepaalde wijk (bv. asielzoekers,
rondtrekkende mensen, studenten …)  minder zicht/slechter beeld door agenten
op die groep

o algemeen-cultureel gestereotypeerde groepen = groepen waarmee politie weinig in


aanraking komt waardoor politie teruggrijpt naar stereotypen die in de samenleving
leven over deze groep  relatie met criminaliteits/overlast profielen

o sedentaire groepen met eigen netwerken en eigen conflictbeheersing  het is


hierdoor moeilijk in te schatten voor agenten wat in die wijken leeft

48

Gedownload door: ayoubelamri99 | ayoubelamri99@outlook.be Wil jij €76 per


Dit document is auteursrechtelijk beschermd, het verspreiden van dit document is strafbaar. maand verdienen?
Stuvia - Koop en Verkoop de Beste Samenvattingen

Conclusie van het onderzoek:


 probleem van underpolicing belangrijker dan Overpolicing
 bij Overpolicing /vaste klanten zijn agenten bekend en hebben ze ≠ strategieën ontwikkeld
om met probleemgroepen om te gaan
o dit palet hebben ze niet bij groepen die underpoliced zijn  leidt vaker tot clichés
 deze clichés zijn zeer zichtbaar in intern discours onder collega’s 
negatieve beeldvorming
 in omgangspraktijk echter niet het geval

Aversie tegen COP-discours


 Weerstand tegen beleidspogingen om politiecultuur te veranderen volgens COP model
 gezien als: niet toepasbaar, te soft, extern opgelegd
 Observatie van praktijk leert echter dat wel veel COP elementen voorkomen
o ≠ aanpakken volgens bevolkingsgroep en context
o gebaseerd op specifieke kennis van problemen
o houden problemen werkbaar en onder controle

Factoren relatie intern vertoog en aanpak


 tendens straatagenten om eigen prioriteiten te bepalen (‘echt politiewerk’) los van
beleidsvoering  Bepaalde groepen worden aangepakt

 situationele aspecten beïnvloeden aanpak  als gedrag attributen vertoont van


categoriseringssysteem  Bevestiging vooroordelen

 Bij vaste klanten is keuze uit meerdere opties van vertoog en aanpak

o deze ontbreken bij onbekende groepen

 Cultureel en sociaal kapitaal agenten (attitudes, politieke voorkeuren, sociale vaardigheden


en ervaring)

o Beinvloeden harde of zachte aanpak v agenten

 Deze factoren zorgen er voor dat er een verschil is tussen beleidsniveau en straatniveau v/d
politie

Twee zijde van de medaille


Over- under twee kanten van dezelfde medaille. Multiculturele leiden vooral onder under. Politie
vaak te snel neigen naar pessimistische , argwanende, dominantie negatieve perceptie. Agenten
houden dus weinig rooskleurige beelden op na betreffende de bevolking en in bijzonder
multiculturele buurten.

49

Gedownload door: ayoubelamri99 | ayoubelamri99@outlook.be Wil jij €76 per


Dit document is auteursrechtelijk beschermd, het verspreiden van dit document is strafbaar. maand verdienen?
Stuvia - Koop en Verkoop de Beste Samenvattingen

Bijzondere & lokale handhaving

Handhaving van bijzonder strafrecht


 niet in strafcodex, maar bijzondere bepalingen in andere wetten
 Bijzondere inspectiediensten met ruime BVH: arbeidsvoorwaarden, milieu(inspectie), prijzen
producten

Handhaving op lokale niveau


 Gemeentelijke administratieve sancties  o.a. boete, vergunning, schorsing
 Gemeentelijke toezichthouders en politie handhaven

Private Handhaving (= handhaving door privé-instanties)


 veel sectoren in SL geen publieke handhaving door overheid meer, maar private actoren
o Beveiligingsdiensten
o Private speurders of netwerken
 (vb. Bellingcat = privé-instanties die privézaken willen opsporen)

1) handhaving v private regels in private domein


2) vermenging tussen private en publieke handhaving
3) privatisering van de politiefunctie

50

Gedownload door: ayoubelamri99 | ayoubelamri99@outlook.be Wil jij €76 per


Dit document is auteursrechtelijk beschermd, het verspreiden van dit document is strafbaar. maand verdienen?
Stuvia - Koop en Verkoop de Beste Samenvattingen

Formele Geschillenbeslechting
Kijken naar de rechtspraak

1) Civiele rechtspraak in België


Toename van casussen bij Belgische rechters in de laatste 150 jaar
 Maar weinig van de maatschappelijke problemen komen bij de rechtbank terecht.
 The “legal iceberg” (Galanter)
o Men ziet maar een heel klein deel, er is veel meer dan dat.

Rechtspraak kan geanalyseerd worden vanuit


 Aanbod perspectief: Instituties (zie college informele conflictbeslechting)
 Vraag perspectief: rechtssubjecten (degene die ernaar vragen, niet de aanbieders)

Vraag Perspectief
 Teveel aandacht is gegaan naar de rol van gerechten in conflictbeslechting
 Verdraaid beeld van conflicten: formele reductie van conflicten naar dossiers
 Omdraaiing: begin vanaf het perspectief van burgers en hoe zij met conflicten omgaan

Dispuut Pyramide
 We gaan het conflict volgen vanaf het moment dat het bestaat, en al de transformaties dat
het doormaakt en dan de vraag of het uiteindelijk bij de rechtbank terecht zal komen (=top
van de piramide, daar komen we helemaal op het einde.)

 Een analyse van het levenspad of carrière van een juridisch conflict en de fasen van
transformatie

 Rechtspraak is ‘the arrival in court of disputes which arose at other locations in society”
(Galanter) (= eindelijk komen ze aan in de rechtbank, maar ze zijn eigenlijk op andere
plaatsen in de maatschappij ontstaan.)

Model Wouters & van Loon


3 niveaus:
1. Baseline
2. Dispuut analyse proces
3. Dispuut omgang proces

51

Gedownload door: ayoubelamri99 | ayoubelamri99@outlook.be Wil jij €76 per


Dit document is auteursrechtelijk beschermd, het verspreiden van dit document is strafbaar. maand verdienen?
Stuvia - Koop en Verkoop de Beste Samenvattingen

1. Baseline
 Waartegen moet de dispuut Pyramide gemeten worden?
 Nodig voor vergelijking tussen verschillende landen
 De hoeveelheid voorkomende disputen als basis voldoet niet.
 De selectie van disputen is al bepaald door variabelen van analyse en omgang met disputen
 Basis: hoeveelheid sociale relaties waarin men zich bevindt
 Vergelijken tussen gevallen waarin problemen voorkomen en zonder problemen
 Laat disputen ook niet zien als een afwijking van normaal gedrag, maar als een bepaald soort
sociale relatie

2. Dispuut Analyse
 “all the steps involving the definition of an experience or relationship as problematic, and the
transformation of this subjective idea into a dispute, the situation where parties take
polarized positions” (p. 176)

 Naming
 Blaming
 Claiming

Probleem (naming)
Eerst bewust worden van iets als problematisch voor we kunnen spreken van een probleem.
o Gepercipieerde schadelijke ervaringen (Wouters & van Loon)
o Transformatie van niet-bewuste schadelijke ervaringen naar een bewust probleem
o De rol van het proces van benoeming

Actie initiatie (blaming)


 Na de bewustwording van een probleem, is de volgende stap wat er aan gedaan gaat worden
 Wordt het probleem als oplosbaar ervaren?
 Hangt af van complexiteit probleem en de vaardigheid van de persoon
 Zo niet dan wordt het opgegeven, waardoor de carrière van het conflict eindigt.

Vorderen (claiming)

 Communicatie v/d claim a/d andere partij die als verantwoordelijk w gezien vr/ h probleem.
 Hiervoor moet:
o Er een aanwijsbare partij gevonden kunnen worden
o De claim gezien worden als een mogelijke oplossing voor het probleem

Als 1 van deze vragen als negatief wordt beantwoord, dan gaat men het conflict niet verder oplosbaar
proberen krijgen, men gaat het laten gaan / niet meer nastreven.

52

Gedownload door: ayoubelamri99 | ayoubelamri99@outlook.be Wil jij €76 per


Dit document is auteursrechtelijk beschermd, het verspreiden van dit document is strafbaar. maand verdienen?
Stuvia - Koop en Verkoop de Beste Samenvattingen

Polarisatie
 De andere partij weigert geheel of ten dele in te gaan op de wensen van de claimende partij
 De analyse van het dispuut komt hiermee tot einde
 De twee partijen staan in een tegenovergestelde relatie tegenover het probleem
 Het probleem is getransformeerd naar een dispuut

3. Dispuut Omgang
Hulp Derde Partij
 Na polarisatie is de vraag wat de partij met het dispuut gaat doen.
 Eerste optie is om dispuut niet voort te zetten.
 Als wel, inroepen hulp van een derde partij
o Leden van de peer groep (familie, vrienden)
o Leden van verdere groepen (organisaties, officials)
o Juristen (advocaten, rechters)

 Dispuut eindigt met oplossing


 Hieronder vallen alternatieve vormen van geschilbeslechting
 Consultatie met derden biedt geen oplossing voor het conflict
o (dan volgende stap (rechtbank))

Rechtbank
 Geen oplossing wordt bereikt, kan besloten worden het dispuut niet voort te zetten
 Zo ja, indiening bij de rechtbank
 Geen heel verschillende wijze van conflictbeslechting
 Formeler door beroep op regels en procedures
 Een deel van de disputen wordt uiteindelijk beslecht door de rechter

Veel informele onderlinge overeenkomst (vorm informele conflictbeslechting) tussen partijen,


contact met andere is belangrijk.

Tussenconclusie: België heeft geen grote proceszoekende juridische cultuur ↔ andere landen
 Toont dat er een hoge barrière is om de stap naar de rechter te maken

Conclusie
Conclusie: Maar een heel klein deel (11%) van alle maatschappelijke problemen komen uiteindelijk
bij de rechter terecht als juridische disputen.→ toont hoge percentages van informele
conflictbeslechting.

Conclusie: Na elke fase v/d carrière v/h probleem is er uitval, waardoor het dispuut beëindigd wordt.
Dit wijst op vermijding van procesvoering in België.

53

Gedownload door: ayoubelamri99 | ayoubelamri99@outlook.be Wil jij €76 per


Dit document is auteursrechtelijk beschermd, het verspreiden van dit document is strafbaar. maand verdienen?
Stuvia - Koop en Verkoop de Beste Samenvattingen

2. Rechters werk in Theorie en Praktijk


Aan de hand van tekst: N. Huls – Niklas Luhmann: de vorm is belangrijk niet de inhoud.

Rechtvaardige procesvoering
Beeld van de rechter
Vrouw Justitia is blind en gebalanceerd
 Weegschaal: wegen van de argumenten en bewijs voor en tegen een zaak
 Blinddoek: onpartijdige en vooringenomen oordelen (= belangrijke concepten)
 Zwaard: bindende finale beslissing (er wordt niet op teruggekomen)

Waarom gehoorzamen mensen uitspraak van de rechter?


 Kan door geweld worden afgedwongen (orders ’backed by threats’) door een overheid met
geweldsmonopolie (Hobbes).
 Naleving door vrees voor sancties
 Luhmann: verbindende kracht oordeel ligt niet in inhoudelijke juistheid maar in
vormkenmerken van de juridische procedure.

Luhmann = functionalist. Alles heeft zijn eigen functie.

 Theorie van autopoetische systemen.


 Verschil systeem en omgeving, voortdurende uitwisseling tussen beiden.
 Bouwsteen: communicatie (niet sociale feiten, Durkheim, of mensen)
 Communicatie niet resultaat van individu handelen of uitwisselen boodschappen, maar een
zelfstandige werkelijkheid

In het spoor van Durkheim ’s Functionalisme


Wat is de functie van ‘x’ voor de samenleving?

Uitgangspunten:
 Gestandaardiseerde sociale activiteiten zijn functioneel voor het gehele sociale systeem
 Deze vervullen onmisbare positieve functies

Samenleving als organisme: 1 systeem waarin alle verbanden en onderdelen een bijdrage leveren
aan het voortbestaan van het geheel. Afwijking van bestaande verhouding is disfunctioneel die het
voortbestaan van de groep in gevaar brengt (Luhmann)

54

Gedownload door: ayoubelamri99 | ayoubelamri99@outlook.be Wil jij €76 per


Dit document is auteursrechtelijk beschermd, het verspreiden van dit document is strafbaar. maand verdienen?
Stuvia - Koop en Verkoop de Beste Samenvattingen

Theorie van sociale systemen


 Wereld als systeem dat uit allerlei subsystemen bestaat, met allen een eigen functie in het
geheel nodig voor het voortbestaan.
 In de premoderne tijd stond de mens centraal in eigen kleine wereld.
o Was zelfvoorzienend, zelf-belevend

 De moderne wereld is groot en de mens is aangewezen op anderen en sociale instituties


(boeren, politie, brandweer, artsen, leraren, juristen etc.)

OPLOSSEN OM DE COMPLEXITEIT IN HET DAGELIJKS LEVEN TE REDUCEREN:

 Door arbeidsdeling en specialisatie ontstaat functionele differentiatie in subsystemen (recht,


politiek, economie, religie, wetenschap)
 Deze hebben hun eigen autonomie, eigen rationaliteit en modus om waarheid te vestigen
 Daardoor kunnen we steeds complexere problemen oplossen

Rollen & Verwachtingen


 Elk subsysteem heeft eigen code van communicatie en eigen rollen voor mensen.
 Daardoor richten zij zich op een specifiek probleem dat opgelost kan worden.
 Elke rol: bepaald door een verzameling gedragspatronen die verwacht w van de rolvervuller
o Kennisverwachting/ cognitief (bijstelling bij deceptie)
 VB: als kind denk ik dat een kachel met een vuurtje er heel leuk uitziet en dat
dat het leukste is om eens vast te pakken, maar als ik de kachel eenmaal heb
aangeraakt en verbrand ben dan heb ik geleerd dat dat niet iets is wat ik heel
graag wil doen.
o normatieve verachting (verschil familie en school)
 VB: Diefstal, iemand steelt mijn fiets. Dat is een negatieve ervaring, maar niet
een negatieve ervaring die men normnoodzakelijk gaat bijstellen. Nu vind ik
misschien nog meer dat diefstal niet moet plaatsvinden.
 Deze verwachtingen moeten steeds worden aangepast
 (herstructurering van verwachtingen Luhmann)

Roldifferentiatie

 In moderne maatschappij is er een bepaalde vrijheid om andere rollen te kiezen.


o doel: in stand houden bestaande maatschappij
 Onze eigen ervaring is echter minder belangrijk in de bepaling van wat waar is en wat niet.
 Afhankelijkheid: we moeten ons voor veel keuzes verlaten op het oordeel van anderen.
o Specialisten
 Probleem: kan leiden tot vervreemding.

55

Gedownload door: ayoubelamri99 | ayoubelamri99@outlook.be Wil jij €76 per


Dit document is auteursrechtelijk beschermd, het verspreiden van dit document is strafbaar. maand verdienen?
Stuvia - Koop en Verkoop de Beste Samenvattingen

De Functie van Recht / juridische procedures


 Recht als subsysteem met eigen functie
 Kenmerk: beslissingen genomen door bevoegde instantie en via specifieke procedure hebben
een bindend karakter
 Waarom accepteren mensen deze beslissing?
 1 Klassiek antwoord: juridische procedure dient waarheidsvinding van juiste regels toepassen
op juiste feiten (Hobbes had ook een antwoord), rechts- bescherming/zekerheid bieden.
 Luhmann: systeem kan niet zowel op waarheid en bindende beslissingen gebouwd zijn.
Rechter moet beslissen, ook als hij de waarheid niet kent (bewijsplichten etc.)
o Verbod van rechtsweigering

VOORBEELD: Casus reparatie huurder van lekkend dak (niet besproken in les)
Take away punten

 Pure waarheidsvinding onmogelijk


 Rechter weet niet zeker of partijen liegen omtrent gebeurde
 Toch moet de rechter beslissen (verbod op rechtsweigering)
 Focus op procedures

Legitimiteit
= procedure is op een juiste manier tot stand gekomen en is gemotiveerd.
 Aanvaardbaarheid van de beslissing
 Niet gebaseerd op inhoudelijke juistheid van feiten en rechtsvragen
o (waarheidsvinding, natuurkundig model)
 Legitimiteit ligt in: de gegeneraliseerde bereidheid om inhoudelijk nog niet vaststaande
besluiten binnen bepaalde grenzen te gehoorzamen
 Speciale sociale omgeving waarbij mensen op voorhand beslissing accepteren
 Juridische institutie draagt bij aan stabilisering verwachtingen tussen mensen in samenleving
 Cruciale functies:
1) produceren van bindende beslissingen die algemeen geaccepteerd worden.
2) Uitkomst moet onzeker zijn, mag niet van tevoren vaststaan
3) Acceptatie is voor betrokkenen (verliezende partij) een leerproces waarin verwachtingen
geherstructureerd moeten worden
4) Rechtvaardige procedures maken dit makkelijker.

56

Gedownload door: ayoubelamri99 | ayoubelamri99@outlook.be Wil jij €76 per


Dit document is auteursrechtelijk beschermd, het verspreiden van dit document is strafbaar. maand verdienen?
Stuvia - Koop en Verkoop de Beste Samenvattingen

Sociale Bereidheid tot Naleving (thema dat we vorige week ook wel hebben gezien)
 De effectiviteit v/d wet hangt meer af van draagvlak bij bevolking dan haar intrinsieke
kwaliteit
 Mensen volgen regels omdat zij geloven in hun procedurele rechtvaardigheid, niet omdat zij
bang zijn voor bestraffing (Tyler)
 Gevoel van rechtvaardigheid na juridische procedure leidt tot betere naleving wetten
 Onschadelijk maken van teleurstelling door toegenomen participatie

Belang effectieve conflictbeslechtingsprocedure: Betekenis van de procedure


Legitimerend effect van procedures = isoleringstheorie:
1. ontoegankelijk = meer aanvaarding
2. isoleren van juridische relevante feiten
3. rechter alleen voor dat deel verantwoordelijk
4. sociale omgeving wordt gescheiden van partijen

Onvrede met vonnis heeft daardoor enkele effect op betrokken partijen: onschadelijk maken
teleurstelling betrokken partijen.

Juridische Procedure als Sociaal Systeem, wat zorgt voor legitimiteit


 Proces heeft eigen autonomie, uitkomst moet onzeker zijn
 Vereist invulling van bepaalde rollen (sociale acceptatie: geen beoordeling mens)
 Geen ritueel, verandering in gebeurtenissen mogelijk

Rol Differentiatie
 elk systeem = eigen rationaliteit + bepaalt zelf wat regels voor eigen waarheid zijn
premodern (godsbewijs) ↔ nu (afhankelijk van getuige, expert; is geen absolute waarheid).

 moderne procesrecht
1. proces niet helemaal autonoom, maar wel eigen zelfstandigheid t.a.v. omgeving
2. kan veranderen

 Rol procespartij gescheiden van andere maatschappelijke rollen van een persoon
 Gaat niet om hen als mens, maar om juridische hoedanigheid (huurder, werknemer, etc.)
 Verschil individu - juridische persona (masker) dienend om R&P toe te kunnen aan iemand
 Rol rechter ook gescheiden. Kent de partijen niet.
 Gespecialiseerd, toelegging op wat juridisch relevant is

Communicatie
 Recht heeft heel specifieke communicatie en taal om complexiteit te reduceren
 Partijen gestimuleerd om informatie te verschaffen aan rechter

57

Gedownload door: ayoubelamri99 | ayoubelamri99@outlook.be Wil jij €76 per


Dit document is auteursrechtelijk beschermd, het verspreiden van dit document is strafbaar. maand verdienen?
Stuvia - Koop en Verkoop de Beste Samenvattingen

Waarheidsmodus
 Recht heeft als subsysteem eigen wijze om waarheid te vestigen volgens eigen regels en ratio.
 Bewijs via deskundigen en getuigen.
 Vaak geen absoluut bewijs of echte waarheid
 Procesregels kunnen als nodig veranderen (via wetgever), bevordert legitimiteit

Autonomie
 Recht heeft als subsysteem bepaalde zelfstandigheid naar omgeving.
 Omgeving bepaalt niet wat er in rechtszaal gebeurt

!!! Dimensies creëren speelruimte voor betrokkenen, leidt tot acceptatie eindbeslissing !!!
Autonomie komt volgens Luhmann op 3 manieren naar voren:
1) Temporeel: recht heeft eigen tempo (bepalen procesdata, strikte termijnen, geen te lang uitstel)
2) Zakelijk: ruimte voor rechter en partijen om geschil vorm te geven. Creativiteit en risico nemen.
3) Sociaal: scheiding recht en feiten. Juridisch gelijk is anders dan sociaal gelijk krijgen

 Deze dimensies creëren speelruimte voor partijen en daarmee acceptatie


 Zakelijk. Partijen moeten keuzes maken, bepaalde mogelijkheden nemen/ andere elimineren.
 Partijen raken hierdoor steeds meer verstrikt in hun verhaal en gaan daardoor steeds meer
op in hun rol. = expressieve functie
 Tempo: Heet conflict koelt door trage procesgang af en opent daardoor andere
mogelijkheden
 Sociaal: Via het recht wordt een soms hard sociaal belangenconflict getransformeerd tot een
beheersbaar debat over een juridische vraag. (niet meer oplosbaar in eigen termen)

Beslissing (finaal, zwaard vrouw Justitia)


 Scheiding tussen vinder en ontvanger oordeel
 Finaal oordeel, geen discussie mogelijk
 Verliezer moet accepteren dat persoon er niet toe doet, alleen juridische hoedanigheid
 Functie proces: pacificering van onenigheid en absorptie verder protest
 Symboolfunctie voor derden via openbaarheid (expressieve functie recht)

2 Soorten overheidsactiviteit

Politiek & wetgever Rechtbank

Conditionele programmering: geen doelen, maar


Doelprogrammering: bereiken bepaalde sociale
regel toepassen (als, dan logica). Zo krijgt de
doelen.
rechter legitimiteit

58

Gedownload door: ayoubelamri99 | ayoubelamri99@outlook.be Wil jij €76 per


Dit document is auteursrechtelijk beschermd, het verspreiden van dit document is strafbaar. maand verdienen?
Stuvia - Koop en Verkoop de Beste Samenvattingen

Relevantie Luhmann
 Juridische procedures als specifieke manier om sociale conflicten op te lossen met specifieke
kenmerken
 Alleen met bepaalde kenmerken kan procedure legitimiteit krijgen (onzekere uitkomst, eerlijk
proces, openbaarheid)
 Legitimiteit van de uitspraak wordt niet bepaald door de inhoud, maar door de vorm van de
procedure
 Doel van de procedure is de uitkomst voor de verliezer aanvaardbaar te maken, perceptie van
rechtvaardige behandeling (fair trial)
o Weten dat hij eerlijk is behandeld, rechter onafhankelijk oordelen
 Procedures die niet meer werken kunnen aangepast worden via de wet. Dit is een
eeuwenlang leerproces (geen ritueel)
 ‘Politieke processen’ waarbij rollenspel en procedures worden betwist
 Openbaarheid van uitkomst = belangrijk

Politicologische analyse van rechtbanken door Martin


Shapiro
Is verbaast dat standaardbeeld van rechtelijke macht nog steeds zoveel aanhang heeft.
1. rechter onafhankelijk
2. neutrale regels toepassen
3. hij hoort beide partijen
4. wijst winnaar en verliezer aan

Shapiro heeft het over basale sociale logica van dit type geschilbeslechting. Als je probleem voorlegt
aan een 3e wordt je dyade een triade , d.w.z. een constellatie van 3 partijen.

NADEEL: (-) instabiliteit → triade kan instorten

59

Gedownload door: ayoubelamri99 | ayoubelamri99@outlook.be Wil jij €76 per


Dit document is auteursrechtelijk beschermd, het verspreiden van dit document is strafbaar. maand verdienen?
Stuvia - Koop en Verkoop de Beste Samenvattingen

GASTCOLLEGE WEEK 7:

Wat gaan we behandelen in deze les?

 Hoe spreekt een rechter recht?


o Vanuit welk idee? Welke context?
 Hoe onderzoeken criminologen de straftoemeting?
 Maatschappelijke analyse van de penaliteit (3BA)
o Komt kort aanbod bij Weber en Durkheim

Straffen als sociale praktijk

Benadering van het rechtspreken (= straftoemeting)

Rechtspreken als sociale praktijk in een veranderende sociale en penale context VS Rechtspreken ‘in
eer en geweten’ als een juridisch technische aangelegenheid.

Het rechtspreken (straftoemeting) als onderdeel van een breder maatschappelijk en penaal
systeem.

Rechter = onpartijdig en onafhankelijk (super belangrijk!)

PROBLEEMSTELLING (BEYENS, 2000)

 Vertrekpunt

o Stijging van de gevangenispopulatie, gevangenisoverbevolking


o Onderbenutting van de alternatieven
o Waarom / wanneer gevangenisstraf of alternatieven?
o ! Straftoemeting

 Centrale stelling
o Rechtspreken is een sociale praktijk, die vorm krijgt in interactie met andere actoren
en praktijken in een bepaalde penale en maatschappelijke context.

o Straffen als complexe sociale en betekenende praktijk versus rechtspreken als “in
eer en geweten” toepassen van de wet

o ! maatschappelijke en penale context van het rechtspreken

 Doelstelling
o Complexiteit van het rechtspreken begrijpen

60

Gedownload door: ayoubelamri99 | ayoubelamri99@outlook.be Wil jij €76 per


Dit document is auteursrechtelijk beschermd, het verspreiden van dit document is strafbaar. maand verdienen?
Stuvia - Koop en Verkoop de Beste Samenvattingen

WETTELIJK KADER
 Strafwetboek
 Wet van 1964 (1994) (1999)
o Opschorting, uitstel, probatie
o Dienstverlening en opleiding
 Wet van 17 april 2002
o Werkstraf als autonome straf

Hoe straf gaan bepalen?


 Selectie van misdrijven en daders bij rechter (zaken doorverwezen door OM)
 Legaliteitsprincipe (enkel bestraffen als misdaad omschreven is als straf)
 Oordelen over schuldvraag en straf
 Strafwetboek : strafminima en -maxima (ruimte rechter: zachte of verzwaarde straf?)
 Verzachtende omstandigheden (art. 79-85)
o Soevereine beoordeling rechter
 feit (schade, toevallig), slachtoffer (uitlokking, fouten, medeplichtigheid),
dader (leeftijd, gezondheid, gerechtelijk verleden, berouw, vergoeding
slachtoffer)
 Verzwarende omstandigheden : wettelijk en verplicht
o leeftijd, dood, arbeidsongeschiktheid van het slachtoffer (bende, valse sleutels,
braak)
o Gerechtelijk verleden dader (recidieven? Blanco of niet?)

Belangrijkste strafmodaliteiten in 1995 (hoofdstraffen) (Niet in detail


kennen)

 vrijspraak
 geldboete, al dan niet met uitstel
 effectieve gevangenisstraf al dan niet met geldboete
 uitstel van gevangenisstraf al dan niet met geldboete en/of probatie
 (dienstverlening) – opleiding
 combinatie effectieve gevangenisstraf en uitstel
 opschorting al dan niet met probatie

Straftoemetingsonderzoek

 “Toe-meten” suggereert een precisie die in de realiteit niet bestaat


 Brede waaier van mogelijkheden
o ! discretionaire beslissingsmacht van de rechter
 ! Ongelijkheden en verschillen tussen rechters en rechtbanken
o Dispariteiten, willekeur? (belangrijke focus van straftoemetingonderzoekers)

61

Gedownload door: ayoubelamri99 | ayoubelamri99@outlook.be Wil jij €76 per


Dit document is auteursrechtelijk beschermd, het verspreiden van dit document is strafbaar. maand verdienen?
Stuvia - Koop en Verkoop de Beste Samenvattingen

 Straftoemetingsonderzoek : blootleggen en verklaren van dispariteiten

Wettelijke factoren als krachtigste voorspellers


o ! aard en ernst misdrijf
 (omvang schade, verzwarende omstandigheden, opzet dader...)
o gerechtelijk verleden dader
↔ Buitenwettelijke factoren
o persoonlijke kenmerken dader
o kenmerken van de magistraat
o contextuele factoren : interne en externe

Buitenwettelijke factoren
 Persoon van de verdachte
o gerechtelijk verleden
o samenlevingspatroon (leven ze geïsoleerd, hebben ze kinderen, …)
o geslacht / gender → ridderlijkheidshypothese
(Wijst er op dat rechters zouden geneigd zijn (vanuit een ridderlijkheidsgevoel) om vrouwen minder
streng te straffen dan mannen voor gelijkaardige feiten)
o sociale positie - arbeidspositie
o etniciteit / nationaliteit

cumulatie van verschillende achterstellingsfactoren vergroot risico op gevangenisstraf


! kwalitatief onderzoek naar percepties, beeldvorming en “onzichtbare” achterstellingsmechanismen

 Persoon van de rechter


o persoonlijke en achtergrondkenmerken van de rechter
 selectiejustitie, overwegend conservatief milieu: ideologische bias
o rechter als informatieverwerker :
 Selectiviteit in opname van informatie : “belief perseverance”
• Openheid voor informatie die consistent is met de eigen opvattingen
 Interpretatie, inschatting van de informatie
 Strafdoelstellingen als richtinggever?

Strafdoelstellingen
• visie op straf?
o vergelden, afschrikken, verbeteren, re-integreren, herstellen, neutraliseren?
• visie op daders?
o verantwoordelijkheid, actief- passief, gedetermineerde versus rationele dader?

62

Gedownload door: ayoubelamri99 | ayoubelamri99@outlook.be Wil jij €76 per


Dit document is auteursrechtelijk beschermd, het verspreiden van dit document is strafbaar. maand verdienen?
Stuvia - Koop en Verkoop de Beste Samenvattingen

Persoon van de rechter


• Rechter als informatieverwerker : ! denkpatronen en penale doelstellingen (cf. Hogarth)
o Behandelingsgerichte rechter
 meer informatie bij beslissingsproces
 meer differentiëren, meer individualiseren
 meer belang aan socio-economische determinanten van criminaliteit

o “klassieke” denkers (vergelding, afschrikking)


 cognitieve simpliciteit (! ernst misdrijf)
 meer rekening houden met wettelijke en niet met buitenwettelijke

Context
 Intern : overige penale actoren (virtuele –of reële aanwezigheid)
a) voorlopige hechtenis: prejudiciërende werking
b) eis van parket
c) pleidooi advocaat
d) advies expert en maatschappelijke enquête
e) aanwezigheid dader versus verstek
f) inbreng en houding van de verdachte
g) slachtoffer, BP
h) socialiseringsproces op de rechtbank : penale cultuur
i) strafuitvoering en overbevolking

 Extern : maatschappelijke context


a) economische omstandigheden (cf. probleemgroepen)
b) publieke opinie, vermeende houding van burger ten aanzien van alternatieven en
gevangenisstraf

Straftoemetingspraktijk
Sentencing decisions are not generated by the application of legal rules but through the routine,
largely unreflective, day to day practices of judges working in a distinctive legal culture and in a local
court environment. (Hutton 2006)

63

Gedownload door: ayoubelamri99 | ayoubelamri99@outlook.be Wil jij €76 per


Dit document is auteursrechtelijk beschermd, het verspreiden van dit document is strafbaar. maand verdienen?
Stuvia - Koop en Verkoop de Beste Samenvattingen

Onderzoeksvragen
 Hoe verloopt het rechterlijke beslissingsproces?
o Aandachtspunten in beslissingsparcours?
o Gevangenisstraf versus alternatieven?
 Visie op dader?
 Visie op straf / strafdoelen?
 Visie op interactie met overige penale actoren (interne factoren)?
 Visie op interactie met externe actoren/factoren?

Bevindingen onderzoek
Persoon van de dader
 Opschorting en uitstel zijn sterk afhankelijk van persoonlijke kenmerken dader
o Gerechtelijk verleden : uitstel of niet, blanco strafblad : “gunstmaatregel”
o werk, sociale integratie, schuldbewustzijn, gedrag na de feiten en op de zitting, vaste
woonplaats (cf. nationaliteit)
o verstek (= recht) : zware straf (effectief)

 Individualisering het grootst bij minder ernstige misdrijven


 ! ‘Criminele ingesteldheid’
 ! Sociaal-economische afstand tussen rechter en verdachten

Daderbeeld en stereotypering
 “Echte” versus “occasionele” delinquent (‘sukkelaars’, berouwvolle, bekenners)
 Negatieve recidiveprognose
o ‘Liegende drugsverslaafden’, ‘ontkennende Marokkanen’
 Positieve recidiveprognose (denkt de rechter dat men gaat herbeginnen of niet)
o Studenten, goed ge-integreerden (cf. ouderlijk gezin), beroepsactieven, jonge
delinquent van goede huize:

Cumulatie negatief ingeschatte factoren : spiraal van gerechtelijke reactie

64

Gedownload door: ayoubelamri99 | ayoubelamri99@outlook.be Wil jij €76 per


Dit document is auteursrechtelijk beschermd, het verspreiden van dit document is strafbaar. maand verdienen?
Stuvia - Koop en Verkoop de Beste Samenvattingen

Rechter als informatieverwerker


 Straffilosofie: weinig manifest in discours : beslissing op niveau van “praktisch bewustzijn”

 Overwegend klassieke doelstellingen


o Vergelding : “hij moet het voelen”
o Afschrikking : “stok achter de deur”, “verwittiging”
o Neutralisering, beveiliging van de maatschappij
o Re-integratie : “nog een kans geven”
 Relatieve onbekendheid + afwijzende houding tav. probatie en dienstverlening bij de rechter

Interpretatie dader en straf (discours)


 Niet altijd consistentie tussen geuite visie en straf
 Werkloosheid eerder persoonlijk kenmerk dader dan structureel tekort in samenleving
 “Intellectuele” misdrijfplegers
o Hogere moraliteit en betrouwbaarheid
 Alleen effectieve gevangenisstraf beschikt over voldoende strafgehalte
 Opschorting, probatie en dienstverlening blijven gereserveerd voor een kleine, duidelijk
afgebakende groep delinquenten

Gevangenisstraf – alternatieve straf


Gunstmaatregelen’ voor de nog recupereerbare, behandelbare en kneedbare, nog voldoende in de
samenleving geïntegreerde dader, wiens misdrijf niet te veel afkeuring oproept, die verschijnt op de
zitting, een blanco strafblad heeft, bekent en zich schuldbewust gedraagt.

Werkstraf → wijziging

Een (lange) gevangenisstraf voor de gewelddadige, weinig schuldbewuste, intentionele, berekende


dader, die ‘de klappen van de zweep van justitie kent’, die ‘niet wil luisteren’ en bijgevolg ‘op de
blaren moet zitten’.

Maatschappelijke marginaliteit, een afwijkende levensstijl, luiheid en een gebrek aan verankering in
de burgerlijke samenleving zijn alle kenmerken die de kans op een (onvoorwaardelijke)
gevangenisstraf doen toenemen.

→ zie ook Engels onderzoek van Tombs & Jagger (2006)

Straftoemetingscultuur
 Schriftelijk (! dossiers)
 Weinig communicatie op de zitting (! tijdsdruk)
o rituelen, court room deafness, ‘dwangcommunicatie’
o >< participatieve en communicatieve straftoemeting

65

Gedownload door: ayoubelamri99 | ayoubelamri99@outlook.be Wil jij €76 per


Dit document is auteursrechtelijk beschermd, het verspreiden van dit document is strafbaar. maand verdienen?
Stuvia - Koop en Verkoop de Beste Samenvattingen

 ! Spijtbetuiging en berouw : morele benadering


 Tarifering >< individualisering
 Alternatieven : moeizame doorbraak, uitzonderlijk karakter, tijdrovend
o “elke bijkomende strafmodaliteit is welkom”
o Quid autonome straffen?

66

Gedownload door: ayoubelamri99 | ayoubelamri99@outlook.be Wil jij €76 per


Dit document is auteursrechtelijk beschermd, het verspreiden van dit document is strafbaar. maand verdienen?
Stuvia - Koop en Verkoop de Beste Samenvattingen

Structurele context: interne factoren


 Prejudiciërende werking van de VH
o VH wordt graag gezien als voorschot op de straf
 Pleidooi advocaat
o geloofwaardigheid, vrij negatieve visie, aanreiken van persoonlijke en situationele
elementen (cf. alternatieven)
 Klacht over te routinematige vordering van het parket
 ! Vroegere straftoemetingsbeslissingen (cf. strafregister, beeldvorming)
 Maatschappelijke enquête (door justitie assistente) : minder vertrouwen
 Strafuitvoering : ! niet-uitvoering KS, weinig kennis van de verschillende modaliteiten van
invrijheidstelling

Hedendaagse penale context


 Interactie met samenleving
 Toenemende mediatisering en belang van de publieke opinie
 Multiculturalisering samenleving
 Interculturele interactie in de rechtszaal
 Onveiligheidsgevoelens, onbehagen, crisis

Conclusie empirisch onderzoek (Beyens, 2000)


 Belang penale cultuur en routines, praktisch bewustzijn
 GS blijft de eerste straf
 ! Beeldvorming en perceptieprocessen
 ! Onderzoek naar visie en discours van de rechter over zijn straftoemetingspraktijk

Algemene conclusie
Rechter tussen rede en gevoel
 Rechter als middelpunt van vaak conflicterende belangen en rechtvaardigingen
 De rechter als ‘probleemoplosser’, die opereert vanuit een stilzwijgend, pragmatisch
utilitarisme (Robertson 1998)
o ‘! Instinct, ervaring, feeling, mysterie’
o ‘ST als kunst’
 Eclecticisme in discours en praktijk → ! Vergelding en afschrikking
 Rationalisering → verdrongen emotionaliteit, ! morele benadering
o Rationaliserende cosmetica, latent emotioneel aspect van de ST

67

Gedownload door: ayoubelamri99 | ayoubelamri99@outlook.be Wil jij €76 per


Dit document is auteursrechtelijk beschermd, het verspreiden van dit document is strafbaar. maand verdienen?
Stuvia - Koop en Verkoop de Beste Samenvattingen

Rechtspreken als betekende partij


 Complexe sociale werkelijkheid wordt gereduceerd en gereconstrueerd in abstracte
juridische categorieën

 ! Interpretatie en weging van de categorieën in wisselwerking met andere factoren en


actoren (>< atomistische visie op rechter)

 ! Manier van spreken over dader, visie op autoriteit van andere actoren, stereotypering

Straftoemetingscultuur
 Informeel socialisatieproces geeft betekenis aan handelen
 GEMSCH professionele kennis + vaardigheden (professionele common sense)
 ! Grote discretionaire beslissingsruimte
o Geïndividualiseerde beslissingen, casus gerichte aanpak, uniciteitsdenken
 Straftoemeting als het ‘libido’ van de strafrechtspleging (Melai)
o Quid stuurbaarheid van de ST? Quid stuurbaarheid van de rechter?
o ! Onafhankelijkheid en onpartijdigheid van de rechter?
 → ‘betrekkelijk vrije hand van de rechter’

Gelijkheid versus dispariteit


 ! Gelijkheidsstreven

 Eenheidsstraffen of individualiseren?

 Leidt individualisering tot ongelijkheid?


 Leidt individualisering tot willekeur?

o Wat is het meest rechtvaardig?

 Star gelijkheidsbeginsel versus geïndividualiseerde afhandeling?

o Rekening houdend met bandbreedte ‘valt het wel mee met de dispariteit’

Straffen is meer dan het bestrijden van criminaliteit


 Gevangenisstraf werkt niet!
 Waarom blijft ze dan de hoeksteen van de bestraffing?
 Verder kijken dan instrumentele doel-middel rationaliteit
 Welke maatschappelijke functies zou de bestraffing wel kunnen hebben?

o Latente aspecten van de bestraffing → ‘penaliteit’

 Interne complexiteit van de werking van het instituut

 ! Vertoog, ideeën, relaties tussen actoren, culturen, symbolen,


instrumenten, praktijken, rituelen, gebouwen….

68

Gedownload door: ayoubelamri99 | ayoubelamri99@outlook.be Wil jij €76 per


Dit document is auteursrechtelijk beschermd, het verspreiden van dit document is strafbaar. maand verdienen?
Stuvia - Koop en Verkoop de Beste Samenvattingen

Maatschappelijk aspecten van de penaliteit


Penologie en Penitentiair recht
 Straffen hebben een symbolische, morele en politieke betekenis (Durkheim)
 Bestraffing is een onderdrukkende en uitsluitende praktijk (Marxistische visie)
 De penaliteit is een disciplinerend instituut (Foucault)
 Gevangenisstraf is een geciviliseerde straf (Elias)
 Rationaliseerbaarheid van de bestraffing/penaliteit (Weber)

(Enkele belangrijke vragen om mee te nemen)


 Geef een aantal belangrijke kenmerken van de straftoemeting
 Waarom is het belangrijk om straftoemeting te bestuderen met kwalitatieve
onderzoeksmethoden

 Leg het debat tussen gelijkheid en individualisering uit. Wat zijn de valkuilen van elke
benadering?

 Welke buitenwettelijke factoren worden er onderscheiden in het straftoemetingsonderzoek?

 Welke factoren hebben de grootste ‘voorspellingskracht’ en waarom?

 Wat wordt er bedoeld met de rechter tussen rede en gevoel?

69

Gedownload door: ayoubelamri99 | ayoubelamri99@outlook.be Wil jij €76 per


Dit document is auteursrechtelijk beschermd, het verspreiden van dit document is strafbaar. maand verdienen?
Stuvia - Koop en Verkoop de Beste Samenvattingen

Informele Geschillenbeslechting

 Parmentier - Mediatie, de derde weg


 van Aeken - Civil court litigation and alternative dispute resolution

We gaan de teksten door elkaar heen gebruiken en bespreken.

ADR (Alternative dispute solution)


= geschillenbeslechting buiten de rechter om.

Model Wouters & van Loon


(proberen in België voor het eerst om het stuk onder water in kaart te brengen (ijsberg))

3 niveaus:

1) Baseline
2) Dispuut analyse proces
3) Dispuut omgang proces

Geschilbeslechtings-delta (2004, 2014)


 Afstand van beeld Pyramide met rechtspraak als top en sociale problemen als basis

 Statisch, lineair en teveel nadruk op rechtspraak als modus van oplossing.

 Doet geen recht aan de vele paden die in de SL bestaan

 Focus op alle wijzen van geschilbeslechting en de vertakking van beslissingen (‘rivierendelta’)

 Stroom van sociale problemen die worden ingedamd door juridische en buiten-juridische
infrastructuur

Functioneel perspectief

Van Aeken:
 Oppositie tussen rechtspraak en ADR maskeert wat ze gemeen hebben
 Niet kijken naar het recht, maar naar de functie van ADR en rechtspraak

70

Gedownload door: ayoubelamri99 | ayoubelamri99@outlook.be Wil jij €76 per


Dit document is auteursrechtelijk beschermd, het verspreiden van dit document is strafbaar. maand verdienen?
Stuvia - Koop en Verkoop de Beste Samenvattingen

Functionele Benadering
De Functies van het Recht:

 Ordenende functie. Recht is een manier om mensen de mogelijkheid te geven vooruit te lopen
op het gedrag van anderen.
 Instrumentele functie. Recht is een instrument om wenselijk geachte maatschappelijke
doeleinden te realiseren, zoals een optimale verdeling van goederen.
 Geschil-beslechtingsfunctie. Recht is een mechanisme om conflicten te beslechten.
o Belangrijkste functie voor vandaag.

 Normatieve functie. Recht is een uitdrukking van waarden en idealen.

 Allemaal potentieel juridisch

Achtergrond ADR
 Explosie van rechtszaken na WOII
Vrees voor ondermijnen van sociale cohesie tussen mensen en innovatie bedrijven.
o (VB: Italië: in 50 jaar aantal zaken verzevenvoudigd)
 Periode van culturele omwenteling en protest tegen veel sociale problemen op gebied van:
raciale discriminatie, seksuele vrijheid, milieu, armoede, anti-oorlog, Mensenrechtenbewegingen.

Onvrede met de Rechtsgang


 Procedure (traag, formeel, inflexibel)
o (regels staan vast daar heb je je aan te houden)

 Hoge kosten (rechtskosten, advocaatkosten, … moeten betaald worden)


 Complexiteit en techniciteit geschil
 Drempels voor sociaal zwakkere groepen
 Verzuring sociale relaties: conflict is beslecht, niet opgelost: winnaar & verliezer

Grondslagen
Informalisering & access to justice
Conflicten op kleinere schaal, informeler aanpakken om escalatie te voorkomen. Het is niet
antagonistisch zoals in de rechtbank. Ervoor zorgen dat zaak niet wordt opgezogen door justitie
anders kom je er niet snel vanaf. Het is meer duurzame aanpak van conflicten.

I. Access to Justice
Onderzoek sociale wetenschappen. Toegang tot recht niet evenredig verspreid in maatschappij
Barrières:
 Complexiteit, Kennis, Kosten (geld, tijd, moeite), Vrees verpesten sociale relaties

71

Gedownload door: ayoubelamri99 | ayoubelamri99@outlook.be Wil jij €76 per


Dit document is auteursrechtelijk beschermd, het verspreiden van dit document is strafbaar. maand verdienen?
Stuvia - Koop en Verkoop de Beste Samenvattingen

Verschil tussen partijen (Galanter)


 One-shotter: iemand die eenmalig met gerecht in contact komt.
 Repeat-player: iemand die regelmatig zaken doet bij het gerecht
 ‘Have-nots’ slechten minder barriers dan de ‘haves’.
 ‘Haves’ hebben gunstigere positie(Galanter)
o Eerdere juridische ervaring
o Toegang tot goede advocaten

Ontlasten gerechtelijk systeem


 Groter beroep op rechters
 Juridisering van maatschappelijke relaties en geschillen
 Rechtbanken slachtoffer van eigen succes (filevorming).
 Efficiëntie, doorstroomtijden verkleinen. Zaken weghalen uit gerecht.
 ADR als alternatief om efficiëntie te verbeteren.

Is ADR wel alternatief? Complementair niet competitief.


 Ja want ze staan naast justitie en voor justitie
 Nee, want recht is niet de norm en ADR de uitzondering. 10% van conflicten gaan naar justitie,
90% daarbuiten als standard

Filosofie van Participatie


 Conflicten vroeger gezien als extern aan partijen ↔ Christie: Conflict zijn eigendom van partijen
die het conflict tegen elkaar hebben ontwikkeld. Hebben zelf aandeel, beslissing om te
escaleren of oplossen

 Conflicten worden ‘gestolen’ van betrokken partijen. Geen inspraak meer. Schuldige
‘professional thieves’ onder aura van professionaliteit: Advocaten hadden belang bij
aanmoedigen en rekken conflicten.

 Conflicten opnieuw zien als eigendom van partijen.


 Betrokkenheid partijen bij afhandeling geschil verhogen
 Geen vervreemding van geschil door juridisch technische abstracties
 !!! Participatie en verantwoordelijkheidsgevoel van partijen en sociaal weefsel GEMSCH
versterkt

“een nieuw paradigma om juridische problemen


aan te pakken” volgens “twee assen” van waarden

72

Gedownload door: ayoubelamri99 | ayoubelamri99@outlook.be Wil jij €76 per


Dit document is auteursrechtelijk beschermd, het verspreiden van dit document is strafbaar. maand verdienen?
Stuvia - Koop en Verkoop de Beste Samenvattingen

Informele Geschillenbeslechting
Onderhandeling, Bemiddeling, Ombudsman, Arbitrage, Minnelijke Schikking

Onderhandeling
 De partijen proberen het dispuut te beslechten zonder beroep op een derde partij door
wederzijdse onderhandeling

Bemiddeling
 “het proces waarbij een derde partij de andere partij assisteert om zelf tot een overeenkomst te
komen in een probleemsituatie of een juridisch conflict.”

Contrast Bemiddeling & Rechtspraak


 Probleemeigenaar VS transformatie geschil tot juridische zaak
o Partijen bepalen zelf de definitie v/h probleem + verantwoordelijkheid voor oplossing
 Faciliteren VS structureren volgens juridische voorwaarden
o Uitwisseling info, aanpassen percepties, voorkeuren expliciteren
 Onderhandelen (de-escalatie) VS beslissen

Ombudsman
Stijgende betrokkenheid in Vlaanderen van ombudsman bij oplossing conflicten tussen burgers en
overheidsdiensten.
 Klachtbehandeling
 Aanbeveling

“Rechtssociologen zijn vooral geïnteresseerd in de ombudsman in vergelijking met de traditionele


conflictoplossing door de rechter” (B. Hubeau)

Rechter Ombudsman

bindend niet-bindend

opleggen overleggen

verticaal horizontaal

eenzijdig meerzijdig

reactief proactief

dwingend faciliterend

repressief reflexief

rechtsverhouding sociale verhouding

wetmatigheid behoorlijkheid

neutrale controle door toetsing positieve controle door verbeteringsadvies

73

Gedownload door: ayoubelamri99 | ayoubelamri99@outlook.be Wil jij €76 per


Dit document is auteursrechtelijk beschermd, het verspreiden van dit document is strafbaar. maand verdienen?
Stuvia - Koop en Verkoop de Beste Samenvattingen

Arbitrage
 Rechtspraak buiten de Rechter om (derde partij wordt betrokken). Partijen komen contractueel
overeen om hun heden of toekomstige conflicten door een arbiter te laten beslissen. Deze
uitspraak is bindend tussen partijen. = Private jurisprudence (soort privé-rechtspraak,
rechtspraak buiten de rechter om)

Minnelijke Schikking (Bij Rechtbank)


“het proces waarbij twee partijen ... een juridisch conflict bijleggen door middel van de betaling van
een geldsom”

Uitdagingen voor ADR


Proceduralisering
Machtsongelijkheid
 Ongelijke onderhandelingspositie (kennis, geld, toegang, vaardigheden)
o Vb individu VS bedrijf
 Nadruk op consensus, niet de conflict potentie tot verandering
o Conflict: Marx
o Consensus: Durkheim

Samenleving is veld van tegengestelde krachten


 Strijd tussen individuen en groepen over schaarse goederen zoals geld, macht, status
 conflicten onvermijdelijk omdat verschillende actoren soms verschillend belangen hebben
 het is nodig, het produceert verandering, innovatie
 conflicten kunnen functioneel of disfunctioneel zijn

Privatisering van jurisprudentie VS publieke rechtvaardigheid


 Vrijwilligheid, Evenwicht en wapengelijkheid, Ondoorzichtigheid, Geen precedenten, Geen
preventieve werking, geen controle van wetgever en bestuur

 Evenwicht tussen flexibiliteit van procedures en aanvaardbare graad van rechtsbescherming

Kritiek: Bindende Arbitrage


 Soms snakken mensen juist naar recht, i.p.v. arbitrage dat niet altijd dezelfde waarborgen heeft

Issues: Transparantie, Onafhankelijkheid en Aansprakelijkheid


 Vrijheid Keuze (machtsongelijkheid tussen partijen)
 Openbaarheid Sessies / Bewijs / Uitspraak
 Motivering Redenen Beslissing
 Mogelijkheid van Hoger Beroep
 Garantie Onafhankelijkheid Arbiter
 Aansprakelijkheid van Personen voor hun Acties

74

Gedownload door: ayoubelamri99 | ayoubelamri99@outlook.be Wil jij €76 per


Dit document is auteursrechtelijk beschermd, het verspreiden van dit document is strafbaar. maand verdienen?
Stuvia - Koop en Verkoop de Beste Samenvattingen

Onafhankelijkheid
“Onafhankelijkheid” wordt genoemd
 als een belangrijke eigenschap van de mediator,
 als een belangrijke “ethische standaard” waar de mediator zich aan moet houden.
 Hoe kan dit worden gewaarborgd? Bronnen:
o Ethische Overwegingen
o Code of Practice

Principes
 Onafhankelijkheid arbiter
 Transparantie van de procedure
 Hoor en wederhoor
 Doeltreffendheid toegang, prijs en snelheid
 Wettigheid en beschermingsniveau
 Vrijheid tot deelname
 Vertegenwoordiging

Institutionalisering
 Voorportaal van rechtsbedeling, integraal deel gerechtelijk system
o Gerechtelijk Wetboek. 7e DEEL: Bemiddeling (art. 1724 -1737), 2005
 Risico dat het zelf formeel & rigide wordt
 Herdefiniëren van kernfuncties van rechtsbedeling (vb. OM: vervolging)
o Delegatie non-kerntaken aan ADR

Professionalisering
 Bestaande beroepen ontdekken “markt” (advocaten, notaris)
 Criteria voor kwaliteit dienstverlening & deskundigheid, mechanismen vorming & opleiding
 Aanpakken belangenconflicten

Evenwicht tussen hoog kwaliteitsniveau en redelijke graad van formalisering die toegang niet beperkt

ADR en Recht 2 Wijzen van Conflictbeslechting


Maatschappelijke trends als achtergrond
gediversifieerde (multiculturele) samenleving
Eventueel naast juridische geschillen ook conflicten in verband met waarden en houdingen ten
aanzien van individu en samenleving

75

Gedownload door: ayoubelamri99 | ayoubelamri99@outlook.be Wil jij €76 per


Dit document is auteursrechtelijk beschermd, het verspreiden van dit document is strafbaar. maand verdienen?
Stuvia - Koop en Verkoop de Beste Samenvattingen

Internationalisering en globalisering
Toename conflicten door grensoverschrijdend karakter van optreden publieke overheden en
economische actoren: oplossingen vragen veel creativiteit

TECHNOLOGISERING (ICT)
 Hulpmiddelen bij oplossing geschillen (op afstand)
 Nieuw soort conflicten VB: producten en providers – providers en gebruikers
o phishing, intellectuele eigendom, ..

Nieuwe Oplossingen, Nieuwe Vormen van Conflictbeslechting?


 Online Dispute Resolution (ODR): een mix van ADR en ICT
 Geschillenbeslechting in Virtuele Werelden: een mix tussen ODR en virtual reality

Internationaal Vergelijkend Perspectief van Rechtspraak &


ADR (Van Aeken)
Verklaring voor het verschil in gebruik van rechtspraak en ADR:

 Socio-economisch
 Politiek-juridisch
 Culturele Verklaring: Litigiousness als het resultaat van rechtsbewustzijn en populaire houding
tegenover claimen (rechtsvorderingen)

Internationaal Vergelijkend Perspectief


Zie PWP: Japan -USA en China- Duitsland

 Statistiek is misleidend door verschil in institutionele achtergrond


 Kijken naar oorzaken
 Verschillende indicatoren

Culturele verklaring
 Vb De controverse over Amerikaanse ‘litigious society’, ‘Adversary culture’
 Zou in de Amerikaanse cultuur zitten om sneller naar de rechtbank te gaan

VB Japan:
 Vergelijkbare economische ontwikkelingsgraad
 Confucianistische moraal en schaamte cultuur.
o Oude geleerde wijze, grote invloed gehad op cultuur
 Nadruk op sociale harmonie en relaties
 Aversie tegen juridische conflictbeslechting die isoleert van het sociale.

76

Gedownload door: ayoubelamri99 | ayoubelamri99@outlook.be Wil jij €76 per


Dit document is auteursrechtelijk beschermd, het verspreiden van dit document is strafbaar. maand verdienen?
Stuvia - Koop en Verkoop de Beste Samenvattingen

Vergelijk Nederland & Nordrhein-Westfalen


 Geografie: zelfde grootte & bevolkingsomvang
 Socio-economisch: vergelijkbaar
 Politiek-juridisch: vergelijkbaar (Franse Code Civil)

Culturele Verklaring
 maar Nederland gebruikt veel minder rechtspraak
Cultuur of Mentaliteitsverschil?
 Nee, beide protestantse landen
 Nee, vergelijkbaar rechtsbewustzijn: hoog vertrouwen in recht & juridische instituties, tegenzin
om recht te gebruiken in privérelaties

Institutionele Verklaring
 Rechtszoeking wordt mede bepaald door institutioneel aanbod, meer dan door populaire vraag
(solutions, looking for problems):
 Aanbod van de proportie juridische VS extra-juridische oplossingen
 In Nederland meer avoidence infrastructure: meer alternatieve en pre-rechtbank vormen van
conflictbeslechting (schuldinning, bouwsector, huurzaken, arbeid)

Institutionele Verklaring
 Japan
 Aanbodsthese
 Bestaan van een serie van institutionele barrières tot rechtspraak (Civil Liberties Bureau)
 Ontmoedigend effect

Conclusie
 Derde weg is geen aparte weg (alternatief pad) maar steeds (meer) integraal deel van het pallet
van conflictbeslechtingswijzen

 Belang van benadering conflict in brede rechtssociologische visie, dan enge doctrinaire visie op
effectiviteit en efficiëntie

 Opletten voor de inherente gevaren van uitholling van de juridische procedure

77

Gedownload door: ayoubelamri99 | ayoubelamri99@outlook.be Wil jij €76 per


Dit document is auteursrechtelijk beschermd, het verspreiden van dit document is strafbaar. maand verdienen?
Stuvia - Koop en Verkoop de Beste Samenvattingen

Juridische Beroepen
 De Mensen van het Recht
o Hubeau et al - Het recht leidt tot alles, zelfs de balie

 Recht in het Bedrijfsleven


o van Houtte - Law in the world of business: lawyers in large industrial entreprises

De mensen van het recht


Juridisch beroep
‘een beroep waarbij juridische activiteiten worden uitgeoefend door een houder van een juridisch
diploma’ (Aubert, 1971)

Traditioneel Niet-traditioneel

Advocaten
Overheidsambtenaren
Notarissen
Bedrijfsjuristen
Gerechtsdeurwaarders
Magistraten

 Sommige traditionele beroepen zijn professies (notaris, advocaat)

NIET rechters: ze zijn gebonden aan hun ambt (de regels die daarvoor gelden) ↔
advocaten/notarissen veel meer marktvorming, kunnen zelf veel meer vormen geven aan hun eigen
professie.

o Berusten op bepaalde waarden en tradities,


o Zelfregulerend,
o Eigen opleiding,
o Toelatingsvoorwaarden
o Hoogstaande dienstverlening in vrije markt juridische diensten

Juridische beroepen in België een algemeen overzicht


Rechtssociologie van Juridische Beroepen
Diverse stadia in het proces om juridische professional te worden (VB: advocaat)
 Universitaire instroom en uitstroom
o Universitaire bachelor & master (toegang traditionele juridische beroepen)
o Hogeschool (paralegals)

 Werkkringen van juristen


 Advocaat stagiair
 Advocaat an sich

78

Gedownload door: ayoubelamri99 | ayoubelamri99@outlook.be Wil jij €76 per


Dit document is auteursrechtelijk beschermd, het verspreiden van dit document is strafbaar. maand verdienen?
Stuvia - Koop en Verkoop de Beste Samenvattingen

Studenten en afgestudeerden in rechten aan de universiteit


Historisch
19e eeuw wordt de maatschappij sterk door juristen bestuurd
 In regering en overheidsapparaat
 In handel en industrie

Algemene behoefte aan hogere opleiding (niet noodzakelijk aan specialistische juridische kennis)
20e Eeuw: wel grotere behoefte aan specifiek juridisch-technische kennis
 Door juridisering en verwettelijking van de samenleving en maatschappelijke verhoudingen.

Evolutie / oorzaken voor toename afgestudeerden


Geen oorzaak op groei:
 Internationalisering & Globalisering hebben weinig effect op de groei v/h
afstudeerpercentage
 Ook beperkte democratisering van onderwijs.

Sinds 1965 beleid voor hogere onderwijsparticipatie. Echter is er nog steeds een verband tussen het
opleidingsniveau en de tewerkstelling van ouders (werkloosheid) op starten in hoger onderwijs.

 Sociale ongelijkheid

Waar terecht:
 traditionele beroepen groot verschil
 Vanaf 1990 meer en meer naar niet traditionele

Behoefte aan juridische dienstverlening


 Vraag om advies
 Contractuele vastlegging van relaties (notariaat + bedrijfsjurist)
 Procedurele bijstand (advocatuur)

Ontwikkelingen in de Vraag
 Culturele ontwikkelingen
o Verkeer & echtscheiding
o Media-aandacht
o Geschooldheid (sneller naming, blaming, claming)

 Economisch-Politieke ontwikkelingen
o Innovaties in juridische vorm gieten
o Politieke territoriumtwisten: gewestelijk, nationaal, supranationaal

79

Gedownload door: ayoubelamri99 | ayoubelamri99@outlook.be Wil jij €76 per


Dit document is auteursrechtelijk beschermd, het verspreiden van dit document is strafbaar. maand verdienen?
Stuvia - Koop en Verkoop de Beste Samenvattingen

 Verwettelijking maatschappelijke relaties (Juridisering)


o Nieuwe gebieden binnen sfeer van het recht (consumentenbescherming, milieu)
o Andere bronnen regelgeving: secundair, quasi-wetgeving, soft law, paralegale normen

Veranderde functie Recht in de maatschappij


 Passieve registratie van aanwezige Normatieve Orde (codificatie)
 (vanaf 20e eeuw) Actief hulmiddel, instrument om samenleving te sturen en veranderen
 Jurist als sleutelfiguur in overheid en bedrijfsleven

De Diverse Beroepen
Advocatuur
HYPOTHESE (1970): (Vilhelm Aubert): De omvang van het werk van advocaten zal sterk afnemen
Hoeveelheid advocaten tussen 1840-1960 verzevenvoudigd, bevolking verviervoudigd (Noorwegen)

Bloei gerelateerd aan industriële revolutie & behoefte aan algemeen gevormd karakter juristen in
opbouw staatsapparaat (democratisering en emancipatie)

Speculatie over Neergang


 advocaat-generalist verliest terrein aan advocaat-specialist
 Juridische beslissingsmethode ruimt baan voor onderhandeling en bemiddeling

Hypothese gefalsificeerd
 Groei advocatuur niet bepaald door (afnemende) maatschappelijke vraag, ….
 Maar door stijgend aanbod (democratisering onderwijs & emancipatie vrouwen)
 Advocatuur diversifieert en incorporeert nieuwe ontwikkelingen
o (onderhandeling & bemiddeling) in plaats van delegatie

Organisatie / institutionele onafhankelijkheid


 Overkoepelende instellingen van balies geregionaliseerd (OVB, OBEG)
o Stages
o Permanente vorming (cursussen)
o Verlenen van bijstand
o Deontologische regels
o Tuchtrecht (onafhankelijke colleges)

80

Gedownload door: ayoubelamri99 | ayoubelamri99@outlook.be Wil jij €76 per


Dit document is auteursrechtelijk beschermd, het verspreiden van dit document is strafbaar. maand verdienen?
Stuvia - Koop en Verkoop de Beste Samenvattingen

Onderzoek naar Advocaat Stagiair (stage = 3 jaar) (Niet behandeld in de les)


Afleggen eed → Beroepsopleiding → Deelname aan tweedelijns juridische bijstand (pro deo)
Verplichtingen: baliebijdrage betalen, minimum aantal punten behalen, permanente vorming.

 Profiel advocaat stagair


o Hoge sociale afkomst
o Werklast en tijdsbesteding: > 45 u/week
o Financiele situatie: verschil internationale kantoren vs locale kantoren
o Van traditionele patroon-stagair relatie, naar advocatenkantoren

Ontwikkelingen met toename aantal advocaten


Ontwikkeling 1: Vervrouwelijking

 Sinds 1922 toegang tot balie


 1969: 10% vrouw ↔ 2014: meer dan helft vrouw

Moeilijkheden om door te groeien naar hogere functies (vennoot) (= glazen plafond)


Vrouwen werken meer deeltijds, combineren professionele en private aangelegenheden
Vooral in zachtere sectoren recht en werkzaam kleine kantoren

Ontwikkeling 2: Samenwerkingsverbanden

+: kostenbesparend, meer gespecialiseerde dienstverlening, aanvaardbare werkdruk


Graad van intensiteit & duurzaamheid
 Associatie (contractuele verdeling kosten & baten)
 Groepering (gemeenschappelijke diensten) (= meer duurzaam)
 Netwerk (verwijzing (bijvoorbeeld indien zelf te druk)
 Solo (geen samenwerking)

In solo-verband moeite met tijdsbeheer, vervanging, permanente vorming.


Eenmanszaak is niet langer de dominante praktijkvorm, uitoefening met anderen.
Nieuwe vaardigheden naast recht: communicatie & organisatie (team play).

 Vestigingen van internationale advocatenkantoren toegenomen


o Vooral uit U.K. & V.S en in Brussel
 Commercialisering van de advocatuur
 Belang Kostprijs
o Private relatie advocaat-client
o Publieke dimensie: toegang tot recht en rechtsbedeling

81

Gedownload door: ayoubelamri99 | ayoubelamri99@outlook.be Wil jij €76 per


Dit document is auteursrechtelijk beschermd, het verspreiden van dit document is strafbaar. maand verdienen?
Stuvia - Koop en Verkoop de Beste Samenvattingen

Specialisatie
Taboeonderwerp: advocaten horen in alle rechtstakken thuis te zijn.
 Onderzoek:
o Advocaten hebben wel degelijk specialisatieonderwerpen
o Verschil tussen mannen en vrouwen
o Hoe groter het kantoor hoe groter de kans op specialisatiegraad

 Advocaten wel degelijk specialisaitieonderwepren


 Hoe groter kantoor hoe meer specialisatie

Vlaamse Advocaat Stagiairs

 Burgerlijk recht
 Handelsrecht  Vreemdelingenrecht
 Familerecht

Mannen Vrouwen

 Administratief recht
 Familierecht
 Hadelsrecht
 Sociaal recht
 Fiscaalrecht
 Vennootschapsrecht

(Rationalisering (Weber)
 Op allerlei terreinen van de samenleving maakt de generalist plaats voor de specialist

 Ambtelijke organisaties zijn machines waarvan de onderdelen in elkaar grijpen.

 Doelstellingen in allerlei levensgebieden worden op een zo efficiënt mogelijk wijze gehaald op


basis van gespecialiseerde kennis, ook de rechtbank en advocatenkantoren)

Notariaat
 Aantal notarisstandplaatsen blijft relatief onveranderlijk
 Beperkte toename notarissen

Trends
 Samenwerking (kantoren) & Vervrouwelijking (31%)
 Associaties hebben beroep voor vrouwen opengesteld

82

Gedownload door: ayoubelamri99 | ayoubelamri99@outlook.be Wil jij €76 per


Dit document is auteursrechtelijk beschermd, het verspreiden van dit document is strafbaar. maand verdienen?
Stuvia - Koop en Verkoop de Beste Samenvattingen

Magistratuur
3 manieren om beroemd te worden:
1. via gerechtelijke stage
2. via examen inzake beroepsbekwaamheid
3. mondelinge evaluatie examen

 Beperkte stijging door gelimiteerde vacante plaatsen


 Meer vrouwen dan mannen (2013)
 Weinig vrouwen in Cassatie of GH (glazen plafond)
 Belang van vrouwen bij magistratuur: meer context & verzoenings-aanpak (female touch)

Gerechtsdeurwaarder
Weinig gerechtsdeurwaarders, Aantal blijft redelijk constant (numerus clausus)
 Tendens tot samenwerking
 Vervijfvoudiging van stagairs
 Weinig doorstroom (vergrijzing)
 Slechts 15% Vrouw

Bedrijfsjurist
 Grootste afnemer van afgestudeerde juristen na advocatuur
 Forse stijging, 48% vrouw (2015)

Recht in het Bedrijfsleven


Bedrijfsjuristen
 Bedrijfsjuristen zijn historisch een recent fenomeen in België (VS: eind 19 e eeuw)
 In Europa gebeurde dit na WOI
 In Nederland tot 1920 bij multinationals (Shell, Philips, Unilever)
 In 1930 Studiegenootschap juristen werkzaam bij handel en nijverheid
 In België na WOII na 1955
 In 1967 Belgische Vereniging voor Bedrijfsjuristen opgericht
 In 2000 Wet tot Oprichting Instituut voor Bedrijfsjuristen

Redenen
 Invloed van EC recht (EC recht = recht van de Europese commissie (EU-recht))
 Toenemende fiscale, sociale, economische regulering
 Transformatie van bedrijf van productie-eenheid tot socio-economische eenheid
 Complexiteit van bedrijfsleven en toenemend risico
 Onvermogen van de Balie om hieraan tegemoet te komen. (negatieve reden)

83

Gedownload door: ayoubelamri99 | ayoubelamri99@outlook.be Wil jij €76 per


Dit document is auteursrechtelijk beschermd, het verspreiden van dit document is strafbaar. maand verdienen?
Stuvia - Koop en Verkoop de Beste Samenvattingen

Het instituut voor bedrijfsjuristen kan de titel van bedrijfsjurist toekennen aan natuurlijke personen
die voldoen aan enkele eisen: het bezit van een diploma van doctor of licentiaat in de rechten of in
het notariaat of van een gelijkwaardig buitenlands diploma en van een arbeidsovereenkomst met een
Belgische openbare of particuliere onderneming. De aanvrager moet voor zijn werkgever, studies
opmaken, adviezen verstrekken, akten opstellen, raad geven en bijstand verlenen op juridisch vlak, en
in hoofdzaak verantwoordelijkheid dragen op juridisch vlak.

Onderzoeksontwerp
 Weinig aandacht binnen rechtssociologie.
 Vooral focus op de Balie

Methode
 Kwalitatief onderzoek (= diepte-interviews) (<1999) (↔ kwantitatief onderzoek (cijfers))
 20 Grote bedrijven in België (omzet, werknemers, sector)
 Onderscheid
o Bedrijfsjuristen: vervullen directe juridische functie in bedrijf
o Commerciële managers: verantwoordelijk voor een extern georiënteerd departement
in bedrijf

Onderzoeksontwerp
Onderzoeksprobleem
 De ambiguïteit van de positie van juristen bij bedrijven

Rol en Positie Bedrijfsjuristen


Wat verwacht men van bedrijfsjuristen?
 Kijk naar het economische beslissingsproces op verschillende plaatsen in het bedrijf en waar
juridische aspecten opduiken.
 Bedrijven functioneren volgens economische rationaliteit, maar moeten ook juridische logica
in acht nemen
 Bedrijfsjuristen werken volgens juridische rationaliteit, maar moeten ook kijken naar
economische rationaliteit

Onderzoeksontwerp
 Organisatietabel van bedrijf en rol juridisch departement
 Organisatie van juridisch departement
 Contact met externe juridische adviseurs
 Manier van contracteren
 Samenwerking juristen - managers

84

Gedownload door: ayoubelamri99 | ayoubelamri99@outlook.be Wil jij €76 per


Dit document is auteursrechtelijk beschermd, het verspreiden van dit document is strafbaar. maand verdienen?
Stuvia - Koop en Verkoop de Beste Samenvattingen

Organisatie juridisch departement


Bedrijfsjuristen zijn een gewoon verschijnsel in grote bedrijven, tussen 1 – 15, vaak (3-4)
 Vaak toch nog nood aan extern juridisch advies (bv. advocaten) in speciale gevallen
 Beeld: Advocaten zien bedrijfsjuristen vaak als inferieure competitie
o Nuance in studie
 Contact met advocaat vaak via bedrijfsjurist (tenzij bedrijfsjurist gecheckt moet w, dan direct)

Manier van Contracteren


 Hoe wordt een contract opgesteld en wat is de rol van de bedrijfsjurist?
 Contracteren is belangrijke economische & juridische activiteit
 Proces van contract voorbereiding geeft goed beeld van de samenwerking tussen
bedrijfsjuristen en managers
 Grote mate van vrijheid van beleid

Definitie van contract (Stewart Macaulay)

 Rationeel plannen van een transactie met een zorgvuldige provisie voor zoveel mogelijk
toekomstige contingenties als men kan voorzien
 Het bestaan of gebruik van een actuele of potentiële juridische sanctie om de uitvoering van
de uitwisseling te doen plaatsvinden of te compenseren voor niet-uitvoering

In contracten maakt men plannen voor:


 Wat elke partij moet doen of nalaten
 Wat te doen als contingenties optreden
 Wat er gebeurt bij niet-naleving
 De overeenstemming juridisch afdwingbaar te maken

Standaardcontract VS Specifiek Contract

 50-90% zijn standaardcontracten voor routine transacties


VB: Aard van G, Algemene praktijken v/d sector, Machtspositie bedrijf, Dwingend recht, Combinatie

Macht speelt een rol bij initiatief voor contract en onderhandelbaarheid. Grote bedrijven kunnen hun
termen afdwingen.

 Specifieke contracten (= tailor-made) worden gebruikt voor belangrijke, complexe of


exceptionele aangelegenheden

85

Gedownload door: ayoubelamri99 | ayoubelamri99@outlook.be Wil jij €76 per


Dit document is auteursrechtelijk beschermd, het verspreiden van dit document is strafbaar. maand verdienen?
Stuvia - Koop en Verkoop de Beste Samenvattingen

Rol van Bedrijfsjurist bij Contracten


 Precontractuele onderhandelingen worden gevoerd door managers
 Zij zien contracten als ondernemend (entrepreneurial) eerder als juridisch bureaucratisch
 Bij standaardcontracten: weinig samenwerking managers-juristen
 Bij specifieke contracten: onderhandelingen door managers tot de deal is beklonken, dan
wordt juridisch departement betrokken
 Onderhandelaars bepalen of juristen worden betrokken
 Voor belangrijke transacties weigeren algemene managers vaak handtekening als het
juridisch departement niet is betrokken

Belang van Contracten (Stewart Macaulay)


 Klassieker in rechtssociologie

Ondernemers hechten veel minder belang aan contracten dan juristen denken. Ze hebben vooral
commercieel inzicht en nood tot overstijgen juridisch muggenziften.

 Belang persoonlijke relaties tussen ondernemers. Contract van marginaal belang!

In België

 ‘Contracten hebben niet veel kracht, de relatie is belangrijk’


 ‘Niet het contract is het belangrijkst, maar de commerciële onderhandeling’
 ‘Het contract dient slechts als raamwerk’
 ‘Handel is veel informeler dan de wereld van recht’

Waarom komen niet-contractuele praktijken veelvuldig voor?

Contract is meestal niet nodig


 Voorkomen van problemen door andere technieken
 Technieken van risicomijding en spreiding (verzekeren)
 Niet-juridische sancties (interne & externe druk)

Contract is meestal niet wenselijk


 Vragen veel kostbare tijd
 Strikte contractualisering wekt wantrouwen (we houden ons bezig met wat er fout kan gaan)
 (dreiging met) conflicten verzuren zakenrelaties

Verschillende perspectieven op het belang van contracten naar gelang positie in


bedrijf
 Verkoop (sales), Aankoop (purchase), Financiële controle
 Externe juristen, Interne juristen

86

Gedownload door: ayoubelamri99 | ayoubelamri99@outlook.be Wil jij €76 per


Dit document is auteursrechtelijk beschermd, het verspreiden van dit document is strafbaar. maand verdienen?
Stuvia - Koop en Verkoop de Beste Samenvattingen

Toch veel aandacht aan contracten


 Dient interne organisatie doelen van bedrijf
 Voordelen (juridische zekerheid) boven nadelen (tijd en geld in contract)

Formeel-Juridische Rationaliteit
 In Europa ontwikkelde recht een formele logische rationaliteit
 Nadruk op vorm aspecten van het recht, niet de inhoud
 Dit systeem creëerde voorspelbaarheid, reduceerde economische onzekerheid en droeg
daarmee bij tot modern kapitalistische staat

‘(E)conomic exchange is quite overwhelmingly guaranteed by the threat of legal coercion …(A) stable
private economic system of the modern type …(would be) unthinkable without legal guarantees’
(Weber 1954: 29-30)

‘Longterm continuing relations, extended family ties and what Sally Moore calls ‘fictive friendships’ …
play a more important role in modern economies than Weberian formally-rational systems of contract
law’ (Macaulay 1996: 117)
→ Bekritiseert Weber!

Recht is niet zo belangrijk in het bedrijfsleven als gedacht, vergeleken met andere niet-juridische
ordeningssystemen.
!! Echter geen weerlegging van Weber !!

 Men houdt de andere partij niet aan contract op basis van andere economische rationaliteit:
het in stand houden van langdurige zakenrelaties of de hoge kosten van juridische
conflictbeslechting

 Economische doelrationaliteit: een rationele weging van doelen en middelen om een zo


efficiënt mogelijke uitkomst te verkrijgen

Manier van contracteren, 2e studie


Houding bedrijf veranderd
Wat we zien in het bedrijfsleven: we zien de 2 rationaliteiten naast elkaar. Bij de bedrijfsjuristen zien
we de juridische vorm van denken en bij de managers zien we die economische manier van denken.
Die staan vaak in een bepaalde spanning met elkaar.

 Houdingen in bedrijven echter radicaal veranderd sinds eerste studie (1963)


o Internationalisering van handel
o Sterkere competitie
o Groter gebruik financiele instrumenten

 Van stabiliteit naar instabiliteit


 Erosie/ verzuring persoonlijke relaties

87

Gedownload door: ayoubelamri99 | ayoubelamri99@outlook.be Wil jij €76 per


Dit document is auteursrechtelijk beschermd, het verspreiden van dit document is strafbaar. maand verdienen?
Stuvia - Koop en Verkoop de Beste Samenvattingen

groei van de advocatenkantoren en de rol van bedrijfsjuristen


 Investering in juridische diensten
 Veel meer juridische geschillen
 Grotere rol voor contracten

In België (studie van Houtte)


 Geen ondermijning persoonlijke relaties
 Geen toename in belang gehecht aan waterdichte contracten
 Wel indicaties dat contracteren steeds juridischer wordt
 Toenemende Amerikaanse invloeden

Rol & Positie Bedrijfsjuristen


Rol juristen volgens formele organisatietabel. Geven de verwachtingen aan van bedrijfsdirectie.
 Europese bedrijfsjuristen behoren vooral tot het personeel niet tot het bestuur
 De taak is voornamelijk geven van advies en niet actieve deelneming in besluitvorming
 Wantrouwende houding naar bedrijfsjurist

Samenwerking juristen - managers


Bedrijfsjuristen denken volgens managers te technisch en formuleren zaken in te complex
taalgebruik. Ze vertragen de zaken. Maar ook acceptatie van andere restrictieve constraints van
juristen.

Ook evolutie naar meer pragmatische houding van juristen

Van Curatief naar Preventief?


rol van bedrijfsjuristen was eerst beperkt tot het omgaan van geschillen (handling company slip-ups).
 Bredere appreciatie van de rol van recht binnen bedrijven
 Niet alleen geschillen, maar bredere preventieve actie (verzoenend)
 Nog weinig zichtbaar in de praktijk

Onderzoeksprobleem
 De ambiguïteit van de positie van juristen bij bedrijven
 Discrepantie tussen economische rationaliteit in de bedrijfswereld en juridische rationaliteit
waar bedrijfsjuristen aan voldoen?
 (Weber’s over formeel-juridische rationaliteit en economische doelrationaliteit)

88

Gedownload door: ayoubelamri99 | ayoubelamri99@outlook.be Wil jij €76 per


Dit document is auteursrechtelijk beschermd, het verspreiden van dit document is strafbaar. maand verdienen?
Stuvia - Koop en Verkoop de Beste Samenvattingen

Conclusies
 Managers denken te makkelijk zelf zaken te regelen
o gebrek aan juridisch inzicht

 De helft van de juristen geeft aan te laat geïnformeerd te worden


o Alleen als er al problemen zijn

 Belang van bedrijfsjurist positie en rol is beperkt.


o Vaak extern advies voor management, pas als dit gevraagd wordt
o Vaak de juridische uitvoerder van contracten, laat betrokken
o Vaak curatieve rol in een late fase bij conflicten

Bedrijfscultuur & Juridisch Bewustzijn


Langzame toenadering van beide kanten

 Juridisch bewustzijn bij bedrijven neemt toe


 Juridisch advies is onontbeerlijk in complexe bedrijfswereld
 Minder neiging risico’s te nemen
 Andere bedrijfscultuur factoren: bedrijfsethiek, samenwerking departementen
 Juridisch departement is meer pragmatisch geworden

89

Gedownload door: ayoubelamri99 | ayoubelamri99@outlook.be Wil jij €76 per


Dit document is auteursrechtelijk beschermd, het verspreiden van dit document is strafbaar. maand verdienen?
Stuvia - Koop en Verkoop de Beste Samenvattingen

Legitimiteit van Recht. Burger & Justitie


!! Kijken vanuit het perspectief van de burger zelf !!

Relatie Burger – Recht (rechter en wetgever)

 Top down: thema van gehoorzaamheid ( topdown )


o Rechter (Luhmann)
o Wetgever (Tyler)

 Bottom up: thema van participatie en deelname aan rechtsvorming (van onder naar boven)
o Rechter (Luhmann: nadruk op het proces, legitimiteit door proces)
o Wetgever (agendabouwtheorie (Nick Huls)

Legitimiteit

 Sociologische grondwerk: Weber: 3 Vormen van Legitimiteit van Gezag


 Luhmann: Legitimiteit van Rechterlijk Oordeel door het Proces
 Tyler: Legitimiteit en Naleving van Recht

Publieke Opinie & Recht

 Ideeën over publieke opinie over het recht bestonden voornamelijk in theorie
o Weinig kennis over hoe dit in de praktijk speelde

Wat weten en vinden burgers van recht


Knowledge & Opinion about Law
Internationale onderzoeksgroep
Onderzoek naar rechtsbewustzijn en maatschappelijke opvattingen over het recht van mensen (KOL)
(1973) (gedaan door Vinke en Van Houtte)

Eerste generatie onderzoek naar de mate waarin mensen het recht kennen
 Maatschappelijke kennis van rechtsregels is gering, ook doelgroepen (vgl. Aubert)

Aubert past perfect in deze traditie, omdat hij ging onderzoeken of de wet op het huishoudelijk
personeel gekend is door het huishoudelijk personeel, hij komt tot de bevinding dat dit vrij matig het
geval is.

 Kennis wordt bepaald door de inhoud en verschillende achtergrond variabelen:


o leeftijd, geslacht, opleiding

1973: maatschappelijke bundel leverde 3 conclusies:


 Maatschappelijke kennis rechtsregels = gering
 Opvattingen over R niet alleen bepaald door inhoud regels ook door achtergrondvariabelen

90

Gedownload door: ayoubelamri99 | ayoubelamri99@outlook.be Wil jij €76 per


Dit document is auteursrechtelijk beschermd, het verspreiden van dit document is strafbaar. maand verdienen?
Stuvia - Koop en Verkoop de Beste Samenvattingen

 Onderzoekers verwachten dat ze met de gegevens voorspellingen zouden kunnen doen over
ontstaan crimineel gedrag.
Rechtsregels in ervaringswereld van verschillende bevolkingslagen
Onderzoek met een steekproef resultaten:

 Minder voorstanders voor het willen straffen van overtredingen op soc-eco vlak
 Houding burgers t.o.v. bepaalde rechtsregels samenhangt met aantal achtergrondkenmerken

Rechtsgevoel en Rechtsbewustzijn
Kenmerken 2e onderzoek
 Jaren 80: nieuw type onderzoek naar maatschappelijke opvattingen over recht
 Tweede generatie onderzoek naar de mate waarin mensen zich in het recht herkennen

gaat niet om de vraag of mensen het recht kennen/ of ze kennis hebben van het recht maar eerder
om de vraag herkennen zij zichzelf in het recht/ hoe beleven zij het recht, kunnen zij zichzelf daarmee
vereenzelvigen.

 Recht niet gezien als apart van de samenleving (law and society (= eerste generatie
onderzoek)), maar integraal onderdeel daarvan (law in society)
 Meer gericht op de beelden van recht en rechtsinstituties waarop mensen handelen.
 Van meten naar interpretatie: geen grote bevolking enquêtes (kwantitatief)(= eerste
generatie onderzoek) maar kwalitatieve diepte-interviews en observaties.

3 Voorbeelden
Merry = onderzocht manier waarop mensen tegen het recht aankijken als de manier waarop
juridische profesionals het recht beschouwen. De juridische professionals waren deze “garbage cases”
ontdoen want ze zien deze als triviaal en lastig. (bv burenruzie)

Nielsen= onderzocht of mensen vinden dat wettelijk verboden zou moeten worden om iemand op
straat uit te schelden.

Ewick en Silbey= onderzoeken dagelijks leven en de rol van het recht die hierin speelt. Recht wordt
gezien als objectief gegeven. Sommige mensen accepteren recht zoals het is (with the law) ↔ recht
moeilijk te accepteren want vinden niet legitiem (against the law).

91

Gedownload door: ayoubelamri99 | ayoubelamri99@outlook.be Wil jij €76 per


Dit document is auteursrechtelijk beschermd, het verspreiden van dit document is strafbaar. maand verdienen?
Stuvia - Koop en Verkoop de Beste Samenvattingen

Benadering Amerikaans en Europees


VS Benadering: Hoe ervaart men het Recht?
(2e generatie onderzoek, onderzoekt de beleving van het Recht, kan men zich in het recht herkennen)
Roscoe Pound: maakte het verschil tussen law in the books and law in action.
 Recht in de boeken: geeft een bepaald ideaal aan, hoe de dingen ongeveer horen te zijn.
 Recht in actie: laat zien hoe dingen daadwerkelijk zijn.
→ Tussen die 2 ideaalbeelden van het recht: grote kloof. Deze rechtssociologie, wil die kloof
aantonen, wil laten zien dat die kloof er is.

Beïnvloed door Pound’s law in action:


 Aantonen van de kloof tussen idealen en werkelijkheid van het recht
 Primair gericht op overheidsrecht
 Recht behandeld als onafhankelijke variabele: definitie van recht wordt voor het onderzoek
bepaald en maakt geen deel uit van onderzoek

Mensen spreken op drie manieren over het recht


 Voor het Recht: Recht als objectief gegeven, overtuigd van juistheid
 Met het Recht: Recht als instrument om eigen voordeel te halen
 Tegen het Recht: Moeilijke acceptatie van recht. Twijfel aan legitimiteit

(Recht = iets dat al vaststaat, recht is onafhankelijk)

EU Benadering: Wat ervaart men als Recht?


( Wat recht is staat nog niet vast, is afhankelijk, men moet onderzoeken wat men als recht ervaart)
Beinvloed door Ehrlich ’s concept van ‘levend recht’ van onderen (manier waarop mensen invulling
geven aan recht onafhankelijk van officiële recht)

 Rechtsbegrip: Kritiek op VS benadering met nauwe focus op officiële recht. Ook kijken naar
rechtsbewustzijn van onderen (vgl. de normen in Moore’s semi-autonome Velden)

 Recht is afhankelijke variabele. Definitie niet vooraf bepaald, maar in het onderzoek

Voorbeeld bouwfraude (Nederland)

Prijsafspraken in Nederlandse bouwsector in strijd met Europese kartelwetgeving


 Hoe ervaart men het recht? Negatieve houding tegenover aanbestedingsrecht
 Wat ervaart men als recht? Eigen stelsel van interne normen (semiautonoom sociaal veld)
Wijze waarop sector deze beleeft is cruciaal voor effectiviteit van officieel recht:
Voor wat hoort wat, gelijkheid, vereffenen tot je erbij neervalt

92

Gedownload door: ayoubelamri99 | ayoubelamri99@outlook.be Wil jij €76 per


Dit document is auteursrechtelijk beschermd, het verspreiden van dit document is strafbaar. maand verdienen?
Stuvia - Koop en Verkoop de Beste Samenvattingen

Rechtsvervreemding
Definitie
Arbeider is verplicht voor iemand anders te werken en is niet langer in staat via zijn eigen acties de
wereld zelf vorm te geven en zich de waarde van eigen arbeid toe te eigenen. Hij vervreemdt van:

 Het product dat hij voortbrengt (wat niet van hem is)
 Het arbeidsproces (waarin hij een schakel is)
 Zijn medemens (die concurrenten zijn geworden)(kunnen hun gaan vervangen)
 Uiteindelijk van zichzelf

Friedman: intern en extern rechtssysteem:


intern: recht in de ogen van rechters, advocaten ↔ extern: recht in ogen rest bevolking
4 vormen:
 Juridische machteloosheid: idee zelf niks te kunnen doen om proces te beïnvloeden
 Juridische onoverzichtelijkheid: perceptie van willekeur (beslissingen: willekeurig gemaakt)
 Juridische anomie: gevoel dat schending van recht regel ipv uitzondering is
o Anomie gezien bij Durkheim
o Anomie = normenloosheid
 Juridisch waardeconflict: non-identificatie met waarden die recht uitdrukt

Twee vragen bij rechtsvervreemding


Cognitief:
 Kent men het Recht?
o In hoeverre is men op de hoogte van de geldende rechtsregels?
Normatief
 Herkent men zich in het Recht?
o In hoeverre komt de inhoud van het rechtsnormen overeen met de eigen normen?

4 Burgerschapsstijlen

+ kennen -

Juridische actieven Gezagsvertrouwen


Geïnformeerde vereenzelviging Ongeïnformeerde vereenzelviging
Herkennen
Cynici Buitenstaanders
Geïnformeerde vervreemding Ongeïnformeerde vervreemding

Juridisch actieven (linker bovenkant)

 Goed op de hoogte van het recht + rechtsnormen komen overeen met de eigen normen
 Actieve burgerschapsstijl (homo juridicus) ↔ (homo economicus)

93

Gedownload door: ayoubelamri99 | ayoubelamri99@outlook.be Wil jij €76 per


Dit document is auteursrechtelijk beschermd, het verspreiden van dit document is strafbaar. maand verdienen?
Stuvia - Koop en Verkoop de Beste Samenvattingen

Gezagsgetrouwen

 Herkent zich goed in het recht maar niet goed op de hoogte van de inhoud van rechtsregels
 Respect voor autoriteit

Cynici

 Kent het recht goed, dit is juist een reden om er niet mee eens te zijn (geen herkenning)
 Weinig vertrouwen in functioneren rechtssysteem

Buitenstaanders

 Kent het recht niet goed, herkent zich niet ook niet in het geldende recht
 Mensen met marginale positie in de SL: vreemdelingen, mensen in getto’s, maar ook
achterban van ‘nieuw-rechts’(mensen die ontevreden zijn en zich buitenstaander voelen van
het systeem).

Legitimiteit in rechtssociologisch onderzoek


De manier waarop recht wordt ervaren en de mate van rechtsvervreemding is van belang voor de
legitimiteit van recht.

Dat er een algemene bereidheid bestaat om de voorschriften van de heersende macht te aanvaarden
ongeacht of deze corresponderen met de eigen waarden

Weber legitimiteit
Het politieke wordt gekenmerkt door capaciteit om anderen te doen gehoorzamen maar macht is
niet voldoende, gezag moet legitimiteit hebben.

3 Vormen van Legitimiteit:


 Traditioneel: berust op traditie en sacrale waarden daarin (vb. paus, stamhoofd)
 Charismatisch: berust op charisma van de persoon van de leider (Hitler, Gandhi)
 Wettelijk-rationeel: berust op geldigheid van geheel van wetten die door ambtenaren op
rationale wijze worden toegepast

De drie aspecten van legitimiteit


Vertrouwen: Alle vormen van zowel toenemend of afnemend vertrouwen binnen organisaties,
vertrouwen burgers in elkaar, mate vertrouwen in politiek, media,..

2 vormen:
 sociaal – vertrouwen : vertrouwen waarbij onbekende elkaar ontmoeten
 institutioneel – vertrouwen : vertrouwen in sociale, culturele, eco. en pol. instituties

Tevredenheid:

94

Gedownload door: ayoubelamri99 | ayoubelamri99@outlook.be Wil jij €76 per


Dit document is auteursrechtelijk beschermd, het verspreiden van dit document is strafbaar. maand verdienen?
Stuvia - Koop en Verkoop de Beste Samenvattingen

Tevredenheid burgers met de politie en de rechtspraak. Meer contact en ervaring met politie en
rechters leidt tot meer ontevredenheid omdat ze andere verwachting hadden dan hoe het in
werkelijkheid is.

Acceptatie:
Legitimiteit : de mate waarin gezag een duurzaam patroon heeft omdat het kan rekenen op
vanzelfsprekende volgzaamheid en vrijwillige gehoorzaamheid.

Uitkomst moet onzeker zijn + acceptatie is voor betrokkenen een leerproces zijn waarin
verwachtingen geherstructureerd worden. Rechtvaardige procedures maken dit makkelijk. Dus
cruciale functie: produceren van bindende beslissingen die als algemeen geaccepteerd worden
geacht.

Legitimiteit betwist
Legitimiteit is niet meer vanzelfsprekend onder burgers, het wordt betwist. De burgerlijke
aanvaarding van optreden in justitie geschiedt pas als verantwoording wordt afgelegd. Partijen die in
het ongelijk worden gesteld leggen zich niet meer automatisch neer bij vonnis, maar gebruiken pers
of politiek om gelijk te krijgen.

Van Beleving naar Naleving van Recht


 Derde generatie onderzoek
 De manier waarop mensen het recht beleven is van invloed op de naleving van het recht.
 Mensen volgen regels omdat zij geloven in hun procedurele rechtvaardigheid, niet omdat zij
bang zijn voor bestraffing (Tyler)
 Gevoel van rechtvaardigheid na juridische procedure leidt tot betere naleving wetten

Conclusie: kennis burger is gering: onderlinge verschillen in mate waarin burgers zich herkennen:
legitimiteit echt voor veel mensen niet langer vanzelfsprekend.

Publieke Opinie & Recht


Met Dicey link terug te maken tussen publieke opinie en wetgeving!
→ (social engineering vs social expression).

Legitimiteit van de Rechter (Huls)


 Na WOII veel vertrouwen in rechterlijke macht
 Verandering rond 1968 (vertrouwen begint te kantelen)
o Grote sociale omwentelingen in de SL, protest bewegingen
o Bestaande structuren worden in twijfel gesteld
 Rechters gezien als gesloten conservatieve GEMSCH met weinig oog voor wat er speelt i/d SL
 Verwijten van klassenjustitie + Weinig vertrouwen

95

Gedownload door: ayoubelamri99 | ayoubelamri99@outlook.be Wil jij €76 per


Dit document is auteursrechtelijk beschermd, het verspreiden van dit document is strafbaar. maand verdienen?
Stuvia - Koop en Verkoop de Beste Samenvattingen

Hoogconjunctuur van vertrouwen


Voorbeeld Nederland: 1970-1990. Rechtelijke macht reageert op kritiek en oogst meer waardering
 Openstelling voor nieuwe groepen (eg. vrouwen) om een betere afspiegeling te krijgen van
samenleving en zo meer vertrouwen te winnen
 Meer open, responsief en maatschappelijk betrokken
 Domeinuitbreiding juridische sfeer:
o Open normen, Rol EVRM (supranationaal recht), Redelijkheid & billijkheid
 Beoordeling van private en publieke machtsuitoefening door rechter
 Toenemende cyclus van vraag en aanbod van recht
Twee metaprincipes of algemene verwachtingen
1. Rechtvaardigheid: voor ieder probleem is er een oplossing (revolutie der stijgende
verwachtingen). Hoort niet bij het leven
2. Herstel en vergoeding van schade

Omslag Jaren 90: Kritiek


 Bestuurders: Rechter is op de stoel van de bestuurders gaan zitten
 Ontevredenheid burgers
→ Populistische kritiek op de rechter van politici en media

 Ministerie van Justitie: rechter als slechte manager. Lange wachttijden, meer budget.
Modernisering van het juridische ‘bedrijf’. Reorganisatie en profesionalisering

 Wetenschap: rechters zijn onpraktisch en ondeskundig


o Rechtssociologen: Rechtsvorming bij hoogste rechter vaak gebaseerd op onjuiste
verdraaide feiten (rol advocaten & verzekering)

o Rechtspsychologen: gebrek deskundigheid rechters (oordeel betrouwbaarheid


getuigen en controle verhoortechnieken politie). Pleidooi voor
deskundigheidsbevordering

Aanvaarding van de Rechtsnorm (Van Houtte)


Publieke Opinie
= tegen klassieke politieke theorieën, verenigde wil volk, gebaseerd op geïdealiseerde beelden van
het individu en gemeenschappen)

 Opinie met een publiek karakter


Rechtstheorie= publiek: veronderstelde gemeenschappelijke belangen of doelen van een
meerderheid van de bevolking. (= openbaar, algemeen, volks-, staats-)

1 publieke opinie= publieke opinie van de meerderheid van de bevolking

96

Gedownload door: ayoubelamri99 | ayoubelamri99@outlook.be Wil jij €76 per


Dit document is auteursrechtelijk beschermd, het verspreiden van dit document is strafbaar. maand verdienen?
Stuvia - Koop en Verkoop de Beste Samenvattingen

 Opinie van een publiek (Van Houtte)


Empirische Sociologie
Opvattingen gedeeld door een geheel van individuen dat als dusdanig sociaal te onderscheiden is
 Veelheid en pluriformiteit van opinies van publieken
 Publieke opinie heeft geen werkelijkheid los van deze individuen
 Inhouden van publieke opinie hebben geen eigen bestaan
 Valt niet gelijk met uitlatingen in massamedia.
 Studie van publieke opinie in de sfeer van het politieke: de wederzijdse beïnvloeding tussen
publieke opinie en publiek beleid.

Meting van Publieke Opinie

Opiniepeiling: Representatieve steekproef van doelbevolking: Vragenlijst; Proporties van


respondenten die hetzelfde antwoord geven; In relatie gebracht met sociologische kenmerken.
Andere manifestaties van publieke opinie (niet noodzakelijk representatief)
 Demonstraties & revoltes
 Pers en communicatiemedia
 Betrekken van andere relevante categorieën burgers via andere, + participatieve methoden

Justitie onder de Loep: De publieke opinie ten aanzien van


justitie
De kloof tussen de burger en justitie is groot en de toegang tot justitie is moeilijk.

Meningen verschillen naargelang de sociale-economische achtergrond, leeftijd en opleidingsniveau


van de respondenten.

Kort overzicht bestaande onderzoeken in BE – kloof recht en burgers


VB:
(1) Affaires Bende van Nijvel: rapport parlementaire onderzoekscommissie
(2) Dutroux : spaghetti-arrest, onbegrip en verontwaardiging, witte marsen
 Kritiek op het functioneren van de politie en rechtelijke macht en nood tot hervorming
 Imago van ouderwets, onredelijk en wereldvreemd instituut
 Onderzoek naar legitimiteit van recht en gerecht komt weer centraal te staan
 Media – gebeurtenissen: invloed op vertrouwen bevolking, maar: geen oorzaken
wantrouwen

Vertrouwenscrisis
Vertrouwen in het gerecht is heel laag in vergelijking in het onderwijs en politie. Het wordt
beïnvloedt door de maatschappelijke gebeurtenissen (VB: Dutroux), alsook door de negatieve
berichten over het gerecht in de media. (onderzoek door Elchardus en Smith 1998)

97

Gedownload door: ayoubelamri99 | ayoubelamri99@outlook.be Wil jij €76 per


Dit document is auteursrechtelijk beschermd, het verspreiden van dit document is strafbaar. maand verdienen?
Stuvia - Koop en Verkoop de Beste Samenvattingen

Legitimiteit & Vertrouwen


Werking van justitie:
 Effectiviteit: justitie moeten adequate oplossingen bieden voor maatschappelijke problemen
 Legitimiteit: ‘Erkenning van de autoriteit van politie en justitie door de burger’. Justitie dient
vertrouwen te genieten van de bevolking voor:
o bereidheid politie te gehoorzamen,
o wetsgetrouw te handelen
o medewerking te verlenen
 VB: Aangifte doen, bereidheid tot getuigen, niet zelf recht in handen nemen)

Kloof burgers en recht


 1997: rapport parlementaire onderzoekscommissie
 1998 – 2001: politiehervorming
 2014: hervorming rechtelijke macht

Justitiebarometer
Opgang vertrouwensonderzoek van (juridische) instituties. Nuttig voor beleidmakers om een idee te
hebben van de mening v/d bevolking over het recht.
 Ontwikkeling van een instrument om deze te meten

Methodologie van het onderzoek


In de psychologie “ houding”: geheel van neigingen , gevoelens, vooroordelen, vooropgezette noties,
ideeën, bekommernissen, bedreigingen en overtuigingen aangaande een specifiek onderwerp, het
attitudeobject.

Houding: (1) cognitief (2) emotioneel (3)gedragscomponenten

Theoretisch model als uitgangspunt voor de houding van burger t.a.v. justitie door middel van
focusgroepsinterviews.

98

Gedownload door: ayoubelamri99 | ayoubelamri99@outlook.be Wil jij €76 per


Dit document is auteursrechtelijk beschermd, het verspreiden van dit document is strafbaar. maand verdienen?
Stuvia - Koop en Verkoop de Beste Samenvattingen

Opinies zijn verbale expressies van specifieke attitudes met een gevoel van voorkeur of afkeur

99

Gedownload door: ayoubelamri99 | ayoubelamri99@outlook.be Wil jij €76 per


Dit document is auteursrechtelijk beschermd, het verspreiden van dit document is strafbaar. maand verdienen?
Stuvia - Koop en Verkoop de Beste Samenvattingen

Nationale resultaten van de eerste juridische barometer


Vertrouwen in justitie - algemeen
 42% heeft wel of eerder wel vertrouwen in justitie
 Meer vertrouwen in politie dan justitie
 Mogelijke oorzaken:
o Grotere vertrouwdheid met politie (zichtbaar, eigen ervaring)
o Informatie justitie vaak via media
o Negatieve associatie (belasting, straf vs. redder in nood)

Relatie Achtergrond Factoren

 Vlamingen positiever dan Walen en Brusselaars


 Vertrouwen neemt af met leeftijd
 Vertrouwen neemt toe met inkomen en betaald werk
 Vrouwen hebben minder vertrouwen dan mannen
 Mensen met ervaring met het gerecht hebben minder vertrouwen
 Mensen die kwaliteitskranten lezen hebben meer vertrouwen

De Werking van Justitie


 informatie van justitie over haar werking 75.5% (eerder) negatief
 tevredenheid over werking van justitie 50% (eerder) negatief
 evolutie van de werking van justitie 46.5%: geen evolutie
 toegankelijkheid van justitie 56.3%: (eerder) gemakkelijk
 verwachting van een eerlijk proces 64.6%: (eerder) wel eerlijk proces
 duidelijkheid van juridische taal 72.4%: juridische taal is onvoldoende
duidelijk

Afhandeling van een zaak en procedures


!! algemeen weten of men positief of negatief is over deze items, de concrete percentages niet !!

Algemeen Deel
Rechtszaken duren te lang, meerderheid is niet akkoord met vrijspraak bij procedurele fouten. Iets
meer als de helft vindt dat uitspraken rechtvaardig zijn

Burgerrechtelijk Deel
Meerderheid positief over inschakelen deskundigen, lekenrechters, meerderheid positief over vragen
mening van kinderen boven 12 in familiezaken, meerderheid + over burgerrechtelijke bemiddeling.

Strafrechtelijk Deel

100

Gedownload door: ayoubelamri99 | ayoubelamri99@outlook.be Wil jij €76 per


Dit document is auteursrechtelijk beschermd, het verspreiden van dit document is strafbaar. maand verdienen?
Stuvia - Koop en Verkoop de Beste Samenvattingen

Meerderheid voorstander van strengere bestraffing, voor alternatieve straffen en voor strafrechtelijke
bemiddeling

Gevangenisstraf

- 40%: vervroegde vrijlating


- 52%: uitzitten straf tot einde

Volksjury + snelle rechter


- 73%: eerder voorstander

Rechter moet rekening houden met:


 AKKOORD: aard feiten, vroegere veroordelingen, schade slachtoffer
 NIET AKKOORD: ervaring jeugd, mening bevolking, geestentoestand, sociale situatie

101

Gedownload door: ayoubelamri99 | ayoubelamri99@outlook.be Wil jij €76 per


Dit document is auteursrechtelijk beschermd, het verspreiden van dit document is strafbaar. maand verdienen?
Stuvia - Koop en Verkoop de Beste Samenvattingen

Ervaring met Justitie


Van de bevraagde groep, welk percentage heft in de afgelopen 10 jaar eerdere ervaringen met
justitie opgedaan?

Ervaring Opinie Resultaat Opinie Proces

(laatste 10jr) (positief) (positief)

Burgerlijke zaken 22% 51% 57,3%

Strafzaken 6,6% 36,3% 36,6%

Conclusies
 Minderheid Belgen heeft vertrouwen in justitie
 Werking van justitie kan verbeteren
o dure processen, procedurefouten, negatieve ervaring met zaken,
juridisch taalgebruik, te lage straffen

 Positieve houding over betrekken burgers aan gerechtelijk apparaat


o Bemiddeling (strafrecht & burgerrecht), Lekenrechters, Volksjury

!!! Men verwacht geen verwachting gelijke behandeling, maar wel rechtvaardige uitspraken !!!

Het nut van Opinie onderzoek


Onmisbaar Instrument VS Overbodige Luxe?
Sommige vinden het een onmisbaar instrument, andere een overbodige luxe.

De soorten publieken
Vier soorten publieken met verschillen in kennis en belang

1) Beroepsmatig betrokken bij justitie, vb. rechters, griffiers, parket ..


2) Niet rechtstreeks beroepsmatig betrokken, maar goed geïnformeerd
a. VB: advocaten, gerechtsjournalisten, beleidsmakers justitie, ..

3) Gebruikers. Burgers die zelf ervaringen hebben gehad met justitie


4) De potentiële gebruikers met nog geen ervaring.

Justitie dient vertrouwen te genieten van alle burgers. Betrekken van deze groepen.

Justitie in 4 categorieën:
1. justitie weinig efficiënt of effect gekenmerkt, maar toch hoge legitimiteit geniet

102

Gedownload door: ayoubelamri99 | ayoubelamri99@outlook.be Wil jij €76 per


Dit document is auteursrechtelijk beschermd, het verspreiden van dit document is strafbaar. maand verdienen?
Stuvia - Koop en Verkoop de Beste Samenvattingen

2. justitie heel efficiënt en effectief , maar weinig vertrouwen in hebben

3. justitie zowel als efficiënt/effectief en legitiem beoordelen

4. geen van beide, vervreemd van justitie

Moet beleid zich richten op de publieke mening?

‘Politicians use public opinion surveys in a manner that a drunk uses a lamppost, for support rather
than illumination’ (Flanagan & Longmire 1997) = Politici gebruiken publieke opinie onderzoeken vaak
zoals een dronken een lantaarnpaal gebruikt, voor ondersteuning meer dan voor verlichting.

 Er is kritiek op de politiek omdat opinieonderzoek heel vaak selectief wordt gebruikt, als het
iemand uitkomt dan wordt het gemobiliseerd en als het niet uitkomt wordt het genegeerd.

 Opiniepeiling om te weten wat leeft bij doelpubliek


 Geeft richting en peilt sociale temperatuur, maar niet direct volgbaar en vrees voor misbruik

(Als ze communicatie verbetert, zal justitie een betere positie krijgen en op die manier haar imago
verbeteren. De burger verwacht dat hij op de hoogte gehouden wordt.)

103

Gedownload door: ayoubelamri99 | ayoubelamri99@outlook.be Wil jij €76 per


Dit document is auteursrechtelijk beschermd, het verspreiden van dit document is strafbaar. maand verdienen?
Stuvia - Koop en Verkoop de Beste Samenvattingen

Recht in de risicomaatschappij
Recht in de maatschappij – NICK HULS
Leven in de risico maatschappij
Inleiding
Moderne samenleving ontwikkelt zich in de richting van een risicomaatschappij, een nieuwe fase van
het industriële tijdsperk. Succesformules van de economische groei zijn oorzaken van risico’s.

Modernisering in 3 fasen
1. Pre-Moderniteit
2. Moderniteit = breuk natiestaten met feudale maatschappij

SUCCESFORMULES MODERNE TIJD=


 kapitalistische productiewijze= ondernemingsgerichte productie, arbeidsdeling,
schaalvergroting en consumptie

 wetenschappelijke vooruitgang= expansie technologie in maatschappij, via


consumentenproducten, elektronica, vervoer

RISICO’S =
 Tsjernobyl Ramp 1986, Milieuvervuiling door gif, pesticide, gezondheidsprobleem
 leidt tot groeiend bewustzijn van techno-wetenschappelijke oplossingen, begin
reflexieve moderniteit

3. reflexieve moderniteit = breuk Industriële revolutie en risicomaatschappij


 SL niet langer georganiseerd rond creatie & verdeling goederen/welvaart , maar productie
en verdeling risico’s.

 Risico= moderne benadering van het voorzien en voorspellen van de gevolgen van
menselijk handelen.

 Nu mondiale schaal i.p.v. nationale schaal.

Rationalisering en modernisering
 generalist wordt specialist
 meer efficiënte, rationeler, meer arbeidsdeling

Onderscheid met industriële risico’s


 niet waarneembaar met menselijke zintuigen
 onomkeerbaarheid van VB: beperkte voorraad grondstoffen + onherstelbaar
 aarde en omvang schade groot dat verzekering en aansprakelijkheidsinstelling tekortschieten
 overstijgen van nationale staten

104

Gedownload door: ayoubelamri99 | ayoubelamri99@outlook.be Wil jij €76 per


Dit document is auteursrechtelijk beschermd, het verspreiden van dit document is strafbaar. maand verdienen?
Stuvia - Koop en Verkoop de Beste Samenvattingen

Modernisering in 3 historische fasen


 Pre-moderniteit
 Moderniteit
 Reflexieve Moderniteit

Kritiek op Beck ↔ Wildavsky

BECK WILDAVSKY

Risico’s nadelig, gevaar dat mensen bang maakt geen risico nemen is grootste risico

Risico= een voorwaarde voor maatschappelijke Risico’s nemen o.b.v. trial without error
dynamiek en modernisering
Maatschappij is wars van risico’s en als je niet
Te veel nadruk veiligheid en zekerheid =
deelneemt aan risico’s , heb je geen
verhindert nieuwe ontwikkelingen en zo geen
moderniseringsproces
winst

Veiligheid paradox= mensen tolereren minder


Objectieve toename gevaar de resterende onveiligheid naarmate de
maatschappij veiliger is

Lekenkennis
Verschil waarop experts en leken de risico’s inschatten. Mensen bang voor risico’s die ze niet kunnen
zien of voelen.

Recht in de risicomaatschappij
Algemeen
Duitsers: grondwettelijke veiligheid burgers kan niet worden gegarandeerd door deze maatschappij.

Amerikanen: minder vertrouwen in overheid als oplosser voor problemen. Ze hebben kritiek op de
gebrekkige wetenschappelijke onderbouwing van veiligheidsregulering en willen een kosten-baten.

Hoe kijkt Nick Huls


Combineren veiligheid en oog voor de veerkracht van recht. Hij kijkt naar de ontwikkelingen in
maatschappij.
 levende rechtsstaat ontwikkelt zich in interactie met samenleving
 technisch-wetenschappelijke ontwikkelingen kunnen zich niet geheel buiten recht plaatsen

105

Gedownload door: ayoubelamri99 | ayoubelamri99@outlook.be Wil jij €76 per


Dit document is auteursrechtelijk beschermd, het verspreiden van dit document is strafbaar. maand verdienen?
Stuvia - Koop en Verkoop de Beste Samenvattingen

De vier wachters van risico maatschappij: kansen en uitdagingen


Deze 4 zijn zelf ook geëvolueerd dus daar ook rekening mee houden.
a. de internationale voorhoede van de advocatuur
Commercialisering beroep advocaat, het beroep ontwikkelt zich tot een big business, het ‘officum
nobile’ verliest zijn betekenis als kenmerk.

Er zijn grote internationale Amerikaanse law firms. De belangrijke juridische innovaties komen uit
Amerika zoals VB: de class action.

 Door specialisering is er veel ervaring met complexe risico’s.


 Geen eenmanszak meer; maar uitoefening met andere
 vestiging internationale advocatenkantoren
 vooral uit UK en VS in Brussel

Klassiek liberale beginsel dat ieder zijn eigen schade draagt, vervangen door ‘pech moet weg’ de
verzorgingsstaat. Alsook van een passief vertrouwen op overheid naar actievere houding waar men
beroep doet op privé middelen voor schadevergoeding nl. aansprakelijkheidsrecht.

Deze professionele letselschade advocatuur die zich ontwikkelt is geen idealistische , maar sociale
beweging.

b. technische normenstellers

Er zijn technocratische comités in Europa. Veel van deze wetenschappelijke comités beslissen over
risico’s.

EU niet langer systeem van consensusvorming in proces waarin nationale overheden, bedrijfsleven, in
dialoog met de commissie, langzaamaan de richtlijnen produceerden.

Er zijn niet langer gedetailleerde veiligheidsvoorschriften voor producten , maar enkel essentiële
veiligheidseisen die moeten worden ingevuld door Europese normalisatie-instituten. Bedrijven
hebben een zeer gespecialiseerde expertise.

Voordelen Nadelen

Hoge veiligheid Uitsluiten kleine bedrijven

Weinig politiek spel en ambtelijke gepalaver Kleien rol voor milieu en consumentenbelangen

Deskundige besluitvorming Gebrek aan democratische legitimiteit

c. reflexief bestuur, reflexief omgaan met risico’s


Overheid niet langer top down/ command and control: wetgeving maken en handhaving. Maar
overheid moet horizontaal onderhandelen met belangengroepen.

 deregulering geen optie als veiligheid burgers in geding is


 niet dereguleren maar proceduraliseren: eerlijk proces waarborgen & open communicatie

Er is ruimte voor zelfregulering maar die moet aan bepaalde wettelijke voorwaarden voldoen.
 normalisatie en certificering

106

Gedownload door: ayoubelamri99 | ayoubelamri99@outlook.be Wil jij €76 per


Dit document is auteursrechtelijk beschermd, het verspreiden van dit document is strafbaar. maand verdienen?
Stuvia - Koop en Verkoop de Beste Samenvattingen

 standaardisering
 beroepscodes
 ethische codes

Nieuwe vorm inspraak: “Constructive Technologie Assessment”: overheid in discussie gaan met
experts en belanghebbenden.

d. actievere rol rechter: bewijs en risico


Rechter heeft zich ontwikkeld van een wetstoeppasser naar een politieke actor.
 rechter beslist, niet de deskundige = rechterlijke onafhankelijkheid
 motiveren hoe hij technisch of wetenschappelijke bewijs waardeert
 nood aan wettelijke bewijsmiddelen en rechtelijke overtuiging

Amerika ↔ Benelux

Amerika:

Cross-examination van deskundigen door advocaat om zo wetenschappelijke veronderstelling op


het bot te deconstrueren. Nadeel is dat het geleidt heeft tot een markt voor ‘junk science’.
Wetenschappelijke huurlingen vertellen uitsluitend deel dat voor hun opdrachtgever goed is.

Benelux:

Hier vertrouwen in wettenschappelijke autoriteit groter. Er is vaak maar 1 deskundige. Het recht
met de expert zijn plaats wijzen. Rechter zo meer inzicht in wetenschappelijke en technische
kennis. Er moet contra-expertise zijn.

Uitdagingen rechtsstaat
Wetgever ervaart dat Europa meer marktwerking eist. De advocatuur ontwikkelt zich tot een
bedrijfstak waar commercie een grotere rol speelt. De rechter wordt gemoderniseerd via een
management van denken. (verwijt van te lage straffe, procedurefouten,..)

De centrale waarde rechtsstaat komen in het gedrang. (beschermen minderheden en zwakkeren


alsook eerlijke procedures)

Balans
 in een open democratie = communicatie + onderhandeling
 beslissen op grensvlak van wetenschap, politiek, economie, gezondheid en recht.
 geen enkel van deze geeft absolute zekerheid meer
 samenleving individualistischer
 reflectie rol recht
 risico’s: tegenmoet gestreden met breed scala aan harde en zachte reguleringsvormen.

107

Gedownload door: ayoubelamri99 | ayoubelamri99@outlook.be Wil jij €76 per


Dit document is auteursrechtelijk beschermd, het verspreiden van dit document is strafbaar. maand verdienen?
Stuvia - Koop en Verkoop de Beste Samenvattingen

Vande Walle – conflicthandeling bij collectieve schade: op


zoek naar herstel
opkomst slachtofferschap
Slachtsoffers zijn een ingecalculeerde prijs in de inrichting en functioneren van de risicomaatschappij.
(vliegtuigongelukken, voedingsindustrie)

Continentaal systeem: individueel slachtofferschap ↔ nu collectief.

Collectief slachtofferschap
In criminologie weinig aandacht voor collectief slachtofferschap, omdat in strafrechtelijk context het
allemaal individuele slachtoffers zijn.

Maar door verbreden gezichtsveld criminologie meer studie naar collectief slachtofferschap.

Voorbeelden:
 Tsjernobyl , andere schadelijke bedrijfsactiviteiten
 oorlog en terrorisme dat leidt tot massaschade
 victimologie = slachtofferschap aftoetsen aan mensenrechten
 herstelbeweging= herstel gemeenschap

Diversiteit in collectief slachtofferschap


Gezamenlijk kenmerk: groot #burgers schade oploopt door 1 gemeenschappelijke oorzaak.
Maar er is een grote diversiteit aan schade en slachtoffers. Verschillende oorzaken:
a. natuurkrachten zonder directe tussenkomst mens
VB: aardbevingen, overstromingen,.. toename door klimaatopwarming en klimaatveranderingen.

Grote aandacht voor collectiviteit door massa slachtoffers door de rol van media. Pas bij afhandeling
dat slachtofferdossiers een individuele afhandeling krijgen.

Aangetoond wel degelijk indirecte band mens en klimaat: klimaatverandering en veel slachtoffers
door slecht preventiebeleid.

VB: tsunami Thailand: gebrek werkende communicatiesysteem

Probleem= er is geen directe aansprakelijkheid.

b. politieke conflicten
VB: burgeroorlog, terroristische aanslag, internationale conflicten. Aandacht door massa slachtoffers
genocides Rwanda.

Hier ligt klemtoon sterk op collectiviteit slachtoffers. Collectiviteit slachtoffers = doel een
bevolkingsgroep te raken. Bij de state-based crimes heeft de media een cruciale rol.

Speciale conflictbehandeling: tribunalen. Het is moeilijk verantwoording te dagen.

108

Gedownload door: ayoubelamri99 | ayoubelamri99@outlook.be Wil jij €76 per


Dit document is auteursrechtelijk beschermd, het verspreiden van dit document is strafbaar. maand verdienen?
Stuvia - Koop en Verkoop de Beste Samenvattingen

c. private bedrijven
Aandacht voor slachtofferschap ontstaan in 1960 door ontstaan milieubeweging en
consumentenbeweging. Burgers gingen zich vragen stellen over de risico’s die gepaard gaan met de
ongebreidelde productie.

Hier wel verantwoording bij organisaties en personen aanwijzen.


 poging tot buitengerechtelijk afhandelen weg van alle publieke inmenging en om
collectiviteit slachtoffers zoveel mogelijk in te perken tot individuele zaken

 buitengerechtelijke afhandeling is flexibel en aan te passen aan context, maar gerecht biedt
meer garanties op correcte afhandeling.

Specifieke kenmerken collectieve schade

Zware schade ↔ strooischade Gefixeerde massaschade en sluipende schade

Veel burgers schadelijk schade door 1


1 individueel aanzienlijke schade
gebeurtenis met 1 verantwoordelijke

Veel burgers schade door reeks


Individueel beperkte schade, maar gebeurtenissen, met onduidelijke oorzaken en
collectief aanzienlijk. Er is collectieve verantwoordelijken. (bv kanker, DES-pil ) Hier
2
afhandeling nodig. (vooral kan je moeilijk de collectiviteit aantonen.
consumentenzaken) Media hier collectief slachtofferschap
zichtbaar maken.

Mechanisme voor afhandeling collectieve schade


 gerechtelijke groepsvordering
 buitengerechtelijke collectieve afhandeling
 waarheids- verzoeningscommissies

Voordelen collectieve actie: erkenning slachtofferschap & vereenvoudiging conflictafhandeling

Gevaren: slachtofferschap geneutraliseerd + erkenning individueel slachtoffer onmogelijk

Erkenning slachtoffer
Meest voorkomende vormen van conflictafhandeling = vermijdingsgedrag en tolerantie. Het zijn
vormen van eenzijdige beslechting of inactiviteit aangezien 1 van de partijen zich neerlegt bij situatie
of conflict vermijdt.

Conflicten opgelost omdat 1 van de partijen het wilt vermijden of doordat slachtoffer de oorzaak van
schade negeert of tolereert. In sommige gevallen slachtoffer vrije keuze, maar meestal zijn
vermijdingsgedrag en tolerantie gevolg van machtsonevenwicht tussen verantwoordelijke en
slachtoffer, waardoor slachtoffer kans niet heeft of niet ziet om conflict op te lossen.

109

Gedownload door: ayoubelamri99 | ayoubelamri99@outlook.be Wil jij €76 per


Dit document is auteursrechtelijk beschermd, het verspreiden van dit document is strafbaar. maand verdienen?
Stuvia - Koop en Verkoop de Beste Samenvattingen

Er zijn technieken om conflict te neutraliseren door schade te ontkennen of verantwoordelijkheid


door te schuiven.

Het is eenvoudiger om conflict te neutraliseren zolang slachtsoffers als individuelen handelen. Anders
bij collectief afhandeling zijn er machtsongelijkheden.

(Zie one shotter, repeat play hoorcollege8)

Vereenvoudigde conflictafhandeling
Slachtoffers van schadelijke bedrijfsactiviteiten zijn weinig vertrouwd met de werking van rechtsgang
omdat er veel barrières zijn. Maar door collectiviteit kunnen leed en ervaringen worden gedeeld. Dus
one-shotters en repeat player ongelijk wegwerken door de collectief afhandeling.

Gevaar bij te collectief slachtofferschap


(1) aantal slachtoffers te groot

 biedt mogelijk om slachtofferschap te neutraliseren tot slachtofferschap van ramp of ongeluk


o VB: hongersnood Afrika

 te grote collectiviteit is dat het een vorm van conflicthandeling met directe erkenning van
slachtoffer onmogelijk maakt (Holocaust, Rwanda) → alternatieven: waarheid en
verzoeningscommissies

(2) andere mechanismen voor afhandeling collectieve schade: de groepsvordering

 VS: grote class action in VS tegen bedrijven (dit is groepsvordering)

 EU: druk om tot wettelijke regeling te komen wanneer we worden geconfronteerd met een
zaak waarin groepsvordering kan worden toegepast.

Bv in Nederland:

Dexia zaak door gedupeerden aandelenleasecontacten. (aandelen kopen met geleend geld bij
bank)

Collectivering: coalitie topadvocaten, consumentenorganisaties, TV programma’s; internet,..

Tegenmacht tegen grote bedrijven (banken)

 steeds meer verankering van groepsvordering bij wet in Europa. (Nederland: wet op
collectieve afwikkeling massaschade 2005 & EC’s New Consumer Deal 2018)

110

Gedownload door: ayoubelamri99 | ayoubelamri99@outlook.be Wil jij €76 per


Dit document is auteursrechtelijk beschermd, het verspreiden van dit document is strafbaar. maand verdienen?
Stuvia - Koop en Verkoop de Beste Samenvattingen

Groepsvordering in BE:

Zaak Lernhout en Haupsie. Class action door Deminor en Test Aankoop in 2000.

Wet tot invoeging van ‘Rechtsvordering tot collectief herstel in wetboek van economisch recht’
2014

Kanttekeningen
1) voor wie is de groepsvordering aangewend

 alleen slachtoffers financiële schade met procesmiddelen, of ook zwakkeren?


 alleen consumenten met financiële schade, of ook andere oorzaken zoals natuurramp en
soorten schade VB: fysiek of emotioneel

2) betekenis slachtsoffers: niet altijd erkenning van verantwoordelijkheid door daders bij
groepsvordering

 route via strafrecht


 route via herstelrecht (verzoeningscommissies)

111

Gedownload door: ayoubelamri99 | ayoubelamri99@outlook.be Wil jij €76 per


Dit document is auteursrechtelijk beschermd, het verspreiden van dit document is strafbaar. maand verdienen?
Powered by TCPDF (www.tcpdf.org)

You might also like